You are on page 1of 277

PSAT

The PSAT

1 1
PSAT

Revolution Prep
PSAT Workbook

This workbook is for the exclusive use of Revolution Prep, LLC and its students and is not legal for resale.
2015 Revolution Prep, LLC. All rights reserved. Version 8-15.

While every effort has been made to ensure the accuracy and correctness of the content in this manual,
Revolution Prep assumes no responsibility for errors or omissions, or for damages resulting from the use of
the information contained herein.

This product is not endorsed by the College Board., nor any other agency or school. Any similarity to other
printed material is purely coincidental.

2 2
PSAT Table of Contents

PSAT Introduction (Page 4)

Critical Reading Writing Mathematics


(Page 8) (Page 59) (Page 138)

3 3
PSAT Introduction

PSAT Introduction

4 4
PSAT Structure and Scoring

PSAT LENGTH

Test 1: Reading = 60 Minutes


Test 2: Writing and Language = 35 Minutes

Test 3 and Test 4: Math = 70 Minutes


o Test 3: No calculator math section = 25 Minutes
o Test 4: Calculator permitted math section = 45 Minutes

Total = 165 Minutes (2 hours and 45 minutes)

PSAT ORDER OF DIFFICULTY

Math questions generally increase in difficulty as you move through a section.


o Obvious answers are usually correct earlier in a section and incorrect later in
a section.

Reading and writing questions are of randomly varying difficulty throughout each
section.

PSAT SCORING

The Math and the combined Reading and Writing tests are each scored from 160 to 760.

Your COMPOSITE SCORE is the total of these two scores (320 to 1520).

PSAT SCORING: NATIONAL MERIT SCHOLARSHIP CONSIDERATION

Only juniors taking the PSAT are eligible for National Merit Scholarship consideration.

o For COMMENDED status, a junior must receive a PSAT score in the top 96th
percentile nationally (~34,000 students each year).

o For SEMIFINALIST status, a junior must receive a PSAT score in the top 99th
within his or her state. (~16,000 students each year).

o For FINALIST status, there are additional criteria that a student must meet
beyond his or her PSAT score.

5 5
PSAT Content

READING

If you are reading this, then its already clear you can read boring
material. Now, you just need to learn how to answer the questions
correctly.

This has a lot less to do with knowledge than with your ability to survive
mind-numbing reading passages.

WRITING AND LANGUAGE

Whats scary: it measures your understanding of standard written


English.

Whats not-so-scary: theres no spelling, no vocabulary, and no complicated


terminology.

MATH

All the math you were trying to forget: Pre-Algebra, Algebra, Coordinate &
Plane Geometry, and Trigonometry.

**THE BEST NEWS**


You will learn what the test will be like EVERY TIME.
No surprises on test day.

6 6
PSAT Guessing and Pacing

GUESSING ON THE PSAT

There is NO PENALTY for wrong answers on the PSAT and Every question is
worth the SAME POINT VALUE.

FIRST: Answer all the questions you KNOW HOW TO DO.


NEXT: Make educated guesses on questions for which you can ELIMINATE ANSWERS.
LAST: Guess randomly on questions about which you HAVE NO CLUE.

PACING ON THE PSAT

DONT RUSH to answer every question.

If youre regularly making careless mistakes, you must SLOW DOWN.


Spend test time working on the problems you KNOW HOW TO DO.
Spend homework time working on the problems you DONT KNOW HOW TO DO.

DONT HESITATE to cut loose.

If youre unsure whether youve wasted a lot of time on a question, you already have!
o If you stare at a question and CANT figure out anything, MOVE ON!
The LONGER a question takes, the LESS it can help your score.

DONT WORRY about timing for every single question.

GOOD MOVE: Staying aware of how much time is left in a test.


BAD MOVE: Calculating how much time is left per question in a test.
Develop a sense of test pacing from your practice tests and homework.
Do the math on the test, not on the clock.

7 7
PSAT Writing

PSAT Reading:
Table of Contents

8 8
PSAT Reading: Table of Contents

PSAT Reading: Table of Contents

PSAT Reading: Introduction (Page 10)

Determining Central Ideas, Themes, and Meaning (Page 12)

Command of Evidence (Page 28)

Analyzing Multiple Texts (Page 35)

PSAT Reading and Writing: Table of Contents


Evaluating Quantitative Information (Page 44)

Words in Context (Page 52)


Words in Context (Reading)
Words in Context (Writing)

9 9
PSAT Reading

PSAT Reading:
Introduction

10 10
PSAT Reading: Introduction

PSAT Reading Introduction

Test 1: Reading

60 Minutes for 48 Questions


Four single passages and one multiple texts passage

Concepts covered on PSAT Reading:

Understanding main ideas and purpose


Applying quantitative information
Defining words in context
Utilizing textual evidence

Quick Reading Tips


Read for main ideas.
Save time by skimming details.
Always anticipate your answer.
Always eliminate choices to find the correct answer.
Dont spend too much time on any one question.
Dont make stupid mistakes stay focused.

11 11
PSAT Determining Central Ideas, Themes, and Meaning

PSAT Reading:
Determining Central Ideas,
Themes, and Meaning

12 12
PSAT Determining Central Ideas, Themes, and Meaning: Reading for Main Ideas

Reading for Main Ideas

When reading on the PSAT, it is important to read for an understanding of what the text
is about (as opposed to what happens).

We understand what a text is about through its main ideas:

Why did the author write this sentence, paragraph, or passage?


If I had to quickly describe this sentence, paragraph, or passage to someone
else, what would I say?

Facts, lists, and details have little importance for understanding the meaning of a
text. Think big picture!

Paragraph Structure: Where Do Main Ideas Live?

Main ideas are usually located at the beginning and the end of a paragraph. Details
tend to be found in the middle of a paragraph.

13 13
PSAT Determining Central Ideas, Themes, and Meaning: Effective Reading

Effective Reading: The Two-Track Mind

Whenever you read, there are two parts of your brain at work:

1) The part of your brain that reads the words on the page.
2) The part of your brain that thinks about other stuff.

These two parts of your brain do not necessarily work together!

To ensure focus, periodically stop and ask yourself questions while you read!

At the end of each paragraph, STOP! What is the main idea?


At the end of the passage, STOP!
o Who was the author? (Was the author part of the story or just telling
the story?)
o What was the authors overall purpose in writing the passage?
o How did the author feel about this topic? (What is the tone of the
passage?)

Effective Reading: The Two-Track Transmission

Once we know how to identify main ideas, we can focus our time and attention on the most
important parts of a text.

When to accelerate (read more quickly) when reading a text:

When the author lists facts to support a main idea


When the author repeats a point that has already been made
When the author uses unfamiliar terms/phrases that are unrelated to a
main idea

When to slow down (read more slowly) when reading a text:

At the beginning and end of paragraphs


Whenever the author introduces a new idea
When the authors uses an adverb (adverbs show how the author feels)

14 14
PSAT Determining Central Ideas, Themes, and Meaning: Effective Reading

Effective Reading: Sample Paragraph

Critics argue that since the Apollo


missions, little substantial progress has
been made in space exploration. NASA
Line missions have been criticized as ineffective
5 and costly, and more than one respected
pundit has suggested that the U.S.
government drastically reduce funding for
the International Space Station and other
NASA ventures. Against this backdrop of
10 hostility, policymakers would do well to
remember that trailblazing scientific
exploration has been the subject of
misplaced scrutiny throughout history.
Centuries before the founding of NASA,
15 Christopher Columbus faced similar
skepticism in funding his maiden and in
retrospect, historic voyage across the
Atlantic.

15 15
PSAT Determining Central Ideas, Themes, and Meaning: Effective Reading

Effective Reading: Sample Paragraph


NEW MAIN IDEA (SLOW
Critics argue that since the Apollo DOWN): NASA programs
missions, little substantial progress has have recently been the
been made in space exploration. NASA subject of ridicule.
Line missions have been criticized as ineffective
5 and costly, and more than one respected
DETAILS (SPEED UP)
pundit has suggested that the U.S.
about the ridicule that
government drastically reduce funding for NASA has received.
the International Space Station and other
NASA ventures. Against this backdrop of
10 hostility, policymakers would do well to
remember that trailblazing scientific NEW MAIN IDEA (SLOW
exploration has been the subject of DOWN): Columbus faced
misplaced scrutiny throughout history. skepticism similar to
what NASA is currently
Centuries before the founding of NASA,
facing.
15 Christopher Columbus faced similar
skepticism in funding his maiden and in
retrospect, historic voyage across the
Atlantic.

16 16
PSAT Determining Central Ideas, Themes, and Meaning: Solving Questions

Solving Reading Questions on the PSAT: There are four steps to solving reading
questions on the PSAT.

1) Cover the answer choices. Do not look at the answer choices!

2) Anticipate an answer to the question. How would YOU answer the question? Use the
notes you took while reading!

3) Eliminate answers that are clearly incorrect. Answers are clearly incorrect because
they do not align with your anticipation!

4) Make a decision among whatever choices remain. Avoid answers with absolute
language and/or non-relevant information.

Solving Reading Questions on the PSAT: Out of Scope Answers

What makes wrong answers wrong can generally be reduced to a few words the words
that make a wrong answer Out of Scope.

An Answer Choice Is Out of Scope if it

1) brings in subjects that are not related to the passage


2) talks about something from a part of the passage outside of the part the
question is referring to
3) is too extreme (unless the author is very extreme)
4) assumes too much about the authors viewpoint

17 17
PSAT Sample Reading Passage #1

Questions 1-10 are based on the following Dr. Richard Seed, even offered to set up cloning
passage. 45 clinics, surely jumping the gun by several decades
since very few scientists have the necessary
This passage is adapted from Ian Wilmut, Keith expertise, and even in the best hands, human
Campbell, and Colin Tudge, Dolly and the Age of cloning at this stage would be absurdly risky. I
Biological Control. 2000 by Ian Wilmut, Keith fielded many of the telephone calls that flooded
Campbell, and Colin Tudge 50 into Roslin Institute in the days after we went
public with Dolly, and quickly came to dread the
Line Dolly seems a very ordinary sheep just an pleas from bereaved families, asking if we could
amiable Finn-Dorset ewe yet as all the world has clone their lost loved ones. I have two daughters
acknowledged, if not entirely for the right reasons, and a son of my own and know that every parents
she might reasonably claim to be the most 55 nightmare is to lose a child, and what parents
5 extraordinary creature ever to be born. Mammals would give to have them back, but I had and have
are normally produced by the sexual route: an egg no power to help. I suppose this was my first sharp
joins with a sperm to form a new embryo. But in intimation of the effect that Dolly could have on
1996 Keith Campbell and I, with our colleagues at peoples lives and perceptions. Such pleas are
Roslin Institute and PPL, cloned Dolly from a cell 60 based on a misconception: that cloning of the kind
10 that had been taken from the mammary gland of that produced Dolly confers an instant, exact
an old ewe and then grown in culture. We fused replication a virtual resurrection. This simply is
that cultured cell with an egg from yet another ewe not the case. But the idea is pervasive and was
to reconstruct an embryo that we transferred into reflected in articles and cartoons around the world.
the womb of a surrogate mother, where it 65 Der Spiegels1 cover showed a regiment of
15 developed to become a lamb. This was the lamb we Einsteins, Claudia Schiffers, and Hitlers the
called Dolly: not quite the first mammal ever to be clever, the beautiful, and the not very nice.
cloned, but certainly the first to be cloned from an The prospect of human cloning causes us grave
adult body cell. Her birth overturns one of the misgivings. It is physically too risky, it could have
deepest dogmas in all of biology, for until the 70 untoward effects on the psychology of the cloned
20 moment in February 1997 when we made her child, and in the end we see no medical
existence known through a brief letter in the justification for it. For us, the technology that
scientific journal Nature, most scientists simply produced Dolly has far wider significance. As the
did not believe that cloning in such a way, and decades and centuries pass, the science of cloning
from such a cell, was possible. 75 and the technologies that may flow from it will
25 Dollys impact was extraordinary. We expected affect all aspects of human life the things that
a heavy response, but nothing could have prepared people can do, the way we live, even, if we choose,
us for the thousands of telephone calls (literally), the kinds of people we are. Those future
the scores of interviews, the offers of tours and technologies will offer our successors a degree of
contracts, and in some cases the opprobrium, 80 control over lifes processes that will come
30 though much less of that than we might have effectively to seem absolute. Until the birth of
feared. Everyone, worldwide, knew that Dolly was Dolly, scientists were apt to declare that this or
important. Even if they did not grasp her full that procedure would be biologically impossible
significance (and the full significance, while not but now that expression, biologically impossible,
obvious, is far more profound than is generally 85 seems to have lost all meaning. In the twenty-first
35 appreciated), people felt that life would never be century and beyond, human ambition will be
quite the same again. And in this they are quite bound only by the laws of physics, the rule of logic,
right. and our descendants own sense of right and
Most obviously and unfortunately, because it wrong. Truly Dolly has taken us into the age of
is certainly not the most important aspect 90 biological control.
40 commentators the world over immediately
perceived that if a sheep can be cloned from a body 1Germany's biggest and most influential weekly
cell, then so can people. Many hated the idea. But magazine; the title is German for The Mirror
others welcomed human cloning, and some, like

18 18
PSAT Sample Reading Passage #1
1. The passage is written from the perspective of 6. Which choice does the author cite as a
someone who is limitation to the future of scientific research?

A) actively involved in conducting biological A) Government funding.


research. B) Laboratory techniques.
B) a participant in a recent debate in the field of C) Imagination with regard to clonings
biology. possibilities.
C) knowledgeable about recent scientific D) Morality and logic.
advancements.
D) an advocate for further cloning research. 7. Which choice provides the best evidence for the
answer to the previous question?
2. The author indicates which of the following
about cloning research? A) Lines 57-59 (I supposeperceptions)
B) Lines 67-68 (The prospectmisgivings)
A) Its impact should be thoroughly studied. C) Lines 78-81 (Thoseabsolute)
B) Its benefits should be widely promoted. D) Lines 85-89 (Inwrong)
C) Its effects have been fully understood.
D) It will answer previously unconsidered 8. As used in line 70, untoward most nearly
questions means

3. The phrase beginning and unfortunately . . . A) detrimental.


(lines 38-39) reflects the authors response to the B) long-lasting.
fact that C) malicious.
D) unforeseen.
A) the successful cloning of Dolly did not prove
that people can be cloned. 9. The author suggests that Dollys cloning is
B) in the wrong hands, cloning could bring about
dire consequences. A) the pinnacle of his professional career.
C) most people do not grasp the full significance B) an epoch-defining event.
of cloning. C) most important for its implications for human
D) critics sensationalized the Dolly breakthrough. cloning.
D) completely misunderstood.
4. As used in line 48, absurdly most nearly
means 10. Which choice provides the best evidence for the
answer to the previous question?
A) surreally.
B) unreasonably. A) Lines 25-31 (Dollysfeared)
C) ignorantly. B) Lines 38-42 (Mostpeople)
D) purposelessly. C) Lines 48-53 (I fieldedones)
D) Lines 89-90 (Trulycontrol)
5. The author would most likely agree that the
example of the Der Spiegel cover described in lines
65-67 represents

A) how the world would be different had cloning


existed a century ago.
B) a quintessentially German worldview.
C) a present-day impossibility.
D) the likely negative effects of cloning.

19 19
PSAT Sample Reading Passage #2
Questions 11-19 are based on the following 50 stimulated efforts at research in depth. Modern
passage. Swedish research on Linnaeus started in the
decades about the turn of this century, during the
The following passage is an excerpt from an essay period of the Linnaean jubilee festivals. An
on the renowned Swedish scientist Linnaeus. inventory of the extant manuscripts was made and
55 texts were published. In 1903 there appeared
Line Without doubt, Carl von Linn, known in the Thore Magnus Friess monumental biography, and
English-speaking world as Linnaeus, is the just fifteen years later the Swedish Linnaean
greatest figure in Swedish natural science. Even Society was founded. Linnaeuss life, his
during his lifetime he was acclaimed by the whole environment, and aspects of his work were
5 world, and his writings became peerless books of 60 becoming known in the minutest detail. But still,
law. Within the domain of natural history, he the traditional attitude of awe and wonder was
reigned supreme, just as Newton did within the maintained. Linnaeuss saintly halo shone with
exact sciences. Certainly there existed impenitent greater clarity than ever. He belonged to the
foreigners who refused to bow before the authority Swedish people, and it was not fitting to make a
10 of the Swedish botanist, especially in France, but 65 critical evaluation of his person and his scientific
his influence only increased with the years, and his accomplishment. Since then, matters have
reformation of the most lovable of the sciences improved. Through the research of the last decades
became general property. Even today, several of we know that the man Linnaeus was other than
Linnaeuss works remain indispensable tools for the one of the Romantic legend the sweet and
15 the biological investigator. 70 sentimental elements have vanished. His inner
But what is remarkable and unique about world also contained dark tones of depression and
Linnaeuss posthumous reputation is the influence despair, at least in his later years. Linnaeus now
he has exercised on his native country. It is emerges as more complicated than had previously
without parallel. We know of no other example been suspected. This applies also to his work,
20 where a great scientist has become part of a whole 75 primarily his biological view of nature and his
peoples national consciousness, a patriotic symbol. religious outlook, which have been the object of
There exists, or at least has existed, a Swedish increasingly penetrating studies.
Linnaeus legend, piously tended by those who Thus the revision of the current picture of
considered themselves to be the guardians of the Linnaeus proceeds. But there still remains much to
25 memory of Linnaeus. Its founder, it could be 80 be done; one is never really finished with a man
claimed, was Linnaeus himself. With his nervous like Linnaeus. There are great and important
and overwrought self-awareness, he described and questions that research has scarcely touched. And
commented on the extraordinary destiny of his life perhaps the Romantic Linnaeus legend still lives
and his incomparable greatness as a scientist. But on in certain respects, preventing an impartial
30 it was not until a little way into the nineteenth 85 interpretation of Linnaean botany and the man
century, with the full development of Romanticism, behind it. We must have the right to invite
that Linnaeus first became the object of a national Linnaeus to step down from his pedestal and to put
cult. His person and work were regarded with some searching questions to him; his limitations
reverent awe, and under the sign of the new and failures cast a clearer light over the
35 sensitivity Linnaeus became the Prince of Flowers, 90 incomparable quality of his genius. To the best of
pointing the way to the sanctum of the mysteries of ones ability, and with his works as a guide, it is a
Nature. This picture of Linnaeus was bequeathed matter of separating the light from the shadows.
to coming generations. The delicate Linnaea was
made his symbol, soon to be embroidered on the
40 collars of the uniforms worn by members of the
medical profession. It was as an incarnation of all 11. The author indicates that Linnaeus was
the loveliness of the short Nordic summer that
Linnaeus gained his prestige. Meadows and groves A) self-effacing.
sang his praises, and Linnaean botany, the science B) a model for all scientists.
45 of herbaria, has remained down to our own C) superior to all scientists.
generation a sort of Swedish national asset, a joy D) a conscious cultivator of his own myth.
or a vexation to generations of schoolboys.
The cult of Linnaeus, the native reverence for
Linnaeus and things Linnaean, has naturally

20 20
PSAT Sample Reading Passage #2
12. Which choice provides the best evidence for the 16. The author would most likely agree with which
answer to the previous question? of the following statements?

A) Lines 16-18 (Butcountry) A) Linnaeuss theses will be questioned as more


B) Lines 25-29 (Its founderscientist) comes to light about his character.
C) Lines 48-50 (The cultdepth) B) His stature in Sweden will be adversely
D) Lines 78-79 (Thusproceeds) affected by new findings.
C) Linnaeus the man and Linnaeus the scientist
13. The sentence beginning Certainly there should be considered separately.
existed . . . (lines 8-13) primarily serves to D) Our admiration for Linnaeus will increase as
illustrate how we better understand his limitations and
flaws.
A) the French disliked the Swedes.
B) factors unrelated to science can impede 17. The authors attitude toward current Linnaean
scientific appreciation. scholarship is one of
C) Linnaeus never transcended national A) pride and nostalgia.
boundaries. B) criticism and doubt.
D) the opponents of Linnaeus were small in C) approval and encouragement.
number. D) realism and resignation.

14. The author would most likely characterize the 18. Which choice provides the best evidence for the
scholarly work described in lines 44-52 (Modern answer to the previous question?
Swedish . . . detail) as
A) important but partial. A) Lines 16-18 (Butcountry)
B) antiquated and foolish. B) Lines 22-25 (ThereLinnaeus)
C) superficial but seminal. C) Lines 58-62 (Linnaeussmaintained)
D) necessary and complete. D) Lines 86-90 (We mustgenius)

15. As used In line 73, complicated most nearly 19. The main purpose of the passage is to
means A) describe the legend of an influential figure.
A) elaborate. B) provide an overview of a scientists work.
B) confused. C) conduct a search for truth.
C) unfathomable. D) explore the personality flaws of a famous
D) complex. individual.

21 21
PSAT Determining Central Ideas, Themes, and Meaning: Notes

22 22
PSAT Determining Central Ideas, Themes, and Meaning: Review
Questions 1-10 are based on the following following a raw food diet can make it difficult to get
passage. adequate amounts of calcium, vitamin D, vitamin
B12, iron, zinc, and protein. In fact, many raw
The following passage discusses some aspects foodists have to rely on supplements in order to
both positive and negative of a raw food diet. 55 maintain good nutritional balance. Additionally,
raw food may contain harmful bacteria and
Line In recent years, many alternative diets have parasites, and certain products, such as kidney
emerged, each touting itself as the paragon of beans and buckwheat, are unsafe to consume in
healthy eating. Many of them vegan, vegetarian, their raw form. Ignorance on the matter could
and fruitarian diets, among others restrict the 60 result in poisoning. Finally, opponents criticize raw
5 ingredients that they claim comprise healthy food advocates as promoting supporting data that
nutrition. Raw foodism, on the other hand, does is anecdotal, rather than scientific in nature.
not necessarily restrict ingredients. Instead, it Indeed, many raw foodists claim the best support
restricts the methods of food preparation. As its for a raw food diet is personal experience.
name suggests, raw foodism consists entirely of 65 While mainstream scientists have generally not
10 foods that have not been heated above a certain supported the raw food movement, there is some
temperature, usually 116F, though sometimes as scientific evidence that can be seen to support raw
low as 104F. Its followers generally opt for food claims. Proponents of raw food believe that it
unprocessed, uncooked fruits, vegetables, nuts, contains enzymes that aid its own digestion and
seeds, sprouts, herbs, grains, and legumes, which 70 help populate the digestive tract with beneficial
15 must comprise at least 70% of their diet in order florae that stimulate the human immune
for them to be considered raw foodists. Often, but system. Raw food enzymes include amylases,
not always, raw food dieters are also vegans, who proteases, and lipases, which are enzymes that
exclude meat, fish, poultry, dairy, and eggs. Both break down starches, proteins, and fats,
raw foodists and those who oppose the practice 75 respectively. Keeping these and other biological
20 agree that applying heat to a food usually, though components, such as antibodies, intact helps
not always, chemically transforms it, thus prevent degenerative diseases, slow the effects of
changing its flavor, texture, appearance, and aging, enhance energy, and boost emotional
nutritional properties. What they disagree on is balance. Raw foodists point to several scientific
whether the practice of cooking is positive or 80 studies that have linked high temperature meat
25 negative. preparation to increased cancer risks. Researchers
Reasons for practicing raw foodism are as at the National Cancer Institute have found that
diverse as the individuals who support it. Some human subjects who ate beef rare or medium-rare
raw foodists are pragmatists, claiming that the had less than one-third the risk of stomach cancer
pursuit reduces food and energy costs. They also 85 than those who ate beef medium-well or well-done.
30 say it saves time, though some raw food recipes Other research has indicated significant benefits in
can require considerable advance preparation. reducing breast cancer risk when large quantities
Others have been swayed by health arguments. of raw vegetables are included in the diet.
They claim that raw food is the optimal fuel for the One striking point of contention has involved
human body, since the digestive system evolved 90 pasteurization. Depending on the process used,
35 before humans started cooking with fire. Benefits pasteurization heats milk and other liquid
are thought to range from lower weight and blood products to between 145F and 280F in order to
pressure to reduced allergies and cancer risks. reduce or kill dangerous bacteria, such as
Mainstream scientists who oppose raw foodism Salmonella, E. coli, and Listeria. The U.S. Food
cite anthropological evidence showing that human 95 and Drug Administration (FDA) notes that
40 beings have been cooking their food for over pasteurization has been highly successful in
250,000 years. Food cooked accidentally, by reducing the prevalence of diseases, including
lightning strikes or brush fires, has been consumed listeriosis, typhoid fever, tuberculosis, diphtheria,
for even longer. These critics claim that and brucellosis. The FDA strongly recommends
widespread use of cooking has changed human 100 that children, the elderly, pregnant women, and
45 anatomy, especially around the jaw, and that the persons with weakened immune systems avoid raw
resulting improved nutrition has even led to milk products and other raw liquids, as these
increased brain size. They recognize that the diseases can cause serious injury or even death.
structure of some foods, like tomatoes and carrots, Raw foodists do not discount the effect of
makes it virtually impossible to access the 105 dangerous bacteria, but they claim that the real
50 available nutrients without cooking. Moreover, threat lies in unsanitary dairy and other

23 23
PSAT Determining Central Ideas, Themes, and Meaning: Review
production conditions. If the milk, for example, is 5. As used In line 32, swayed most nearly
collected and stored cleanly, they believe milk- means
borne diseases will not be a problem. They believe
110 the same holds for other commonly pasteurized A) authorized.
products, such as apple juice. More to the point, B) controlled
raw foodists hold that the risk is worth the benefit. C) influenced.
Raw milk is claimed to have antibodies that D) undermined.
provide resistance to many viruses, bacteria, and
115 bacterial toxins and may help reduce the severity 6. Which choice best summarizes the second
paragraph of the passage (lines 26-37)?
of asthma symptoms. They discount studies that
show no significant nutritional difference between A) Most raw foodists cannot fully comprehend the
pasteurized and nonpasteurized milk. Ultimately, value of a healthy diet.
until more hard research is done, mainstream B) Some raw foodists have better reasons for
120 science and raw food advocates will continue to be practicing raw foodism than others.
C) There is a diversity among practitioners of raw
at odds. foodism.
D) Raw foodism is an excellent diet for weight
1. The main purpose of the passage is to loss.
A) highlight how scientific research 7. It can reasonably be inferred from the passage
overwhelmingly supports raw foodism. that raw foodists believe which of the following
B) explain that no scientific research has been regarding pasteurization?
found to support raw foodism.
C) explain how the benefits of raw foodism are a A) Nonpasteurized nondairy products, like apple
topic of intense scientific debate. juice, do not pose the same risks as
D) allow raw foodists to renounce the dangerous nonpasteurized milk.
effects of bacteria. B) Pasteurization is a crutch used by the FDA to
avoid enforcing cleaner production conditions.
2. The passage indicates that raw foodists view C) There are enough benefits from consuming
traditional science with nonpasteurized products to justify any
potential dangers from unsanitary production
A) scorn. conditions.
D) There has not been enough scientific research
B) respect.
regarding the costs and benefits of
C) admiration. nonpasteurized liquids.
D) contempt.
8. Which choice provides the best evidence for the
3. Which choice provides the best evidence for the
answer to the previous question?
answer to the previous question?
A) Lines 33-35 (They claimfire)
A) Lines 18-25 (Bothnegative)
B) Lines 65-68 (Whileclaims)
B) Lines 38-41 (Mainstreamyears)
C) Lines 99-103 (The FDAdeath)
C) Lines 65-68 (Whileclaims)
D) Lines 111-116 (Moresymptoms)
D) Lines 86-88 (Otherdiet)

4. The author places the words Raw foodism in 9. The conclusion that can reasonably be inferred
quotation marks (line 6) most likely to from the fifth paragraph (lines 89-103) is that

A) pasteurization is the only known method of


A) indicate that she is not quite sure what they
mean. killing E. coli bacteria.
B) mark them as a technical term with a specific B) raw milk is the most dangerous food for
children and pregnant women.
definition.
C) force the reader to pay more attention to the C) extreme heat can be an effective means of
term. destroying harmful bacteria.
D) cast aspersions on raw foodisms legitimacy as D) pasteurization at a lower temperature would
a scientific movement. make pasteurized milk acceptable to raw
foodists.

24 24
PSAT Determining Central Ideas, Themes, and Meaning: Review
10. When the passage says that raw foodists
discount studies that show no significant
nutritional difference between pasteurized and
nonpasteurized milk (lines 116-118), it most
nearly means that

A) debate over pasteurized milk is irrelevant


because most raw foodists do not consume
dairy products.
B) raw foodists always disagree with FDA
findings regarding food processing.
C) FDA studies are frequently suspect and cannot
be relied upon when making nutritional
decisions.
D) at times, raw foodists ignore scientific data
that does not support their beliefs.

25 25
PSAT Determining Central Ideas, Themes, and Meaning: Answers

Workbook Answers Review Answers

Determining Central Ideas, Themes, 1) C


and Meaning
2) B
1) A
3) A
2) A
4) B
3) D
5) C
4) B
6) C
5) C
7) C
6) D
8) D
7) D
9) C
8) A
10) D
9) B

10) A

11) D

12) B

13) B

14) A

15) D

16) D

17) C

18) D

19) A

26 26
PSAT Command of Evidence

PSAT Reading:
Command of Evidence

27 27
PSAT Command of Evidence

Command of Evidence

Identifying Command of Evidence Questions on the PSAT: Command of Evidence


questions ask for which line(s) provide the best evidence for the answer to the
previous question.

On Command of Evidence questions, pick the lines with the main idea that most-closely
aligns with the answer to the previous question.

If unsure, eliminate down to as few choices as possible before choosing an answer.

The Nellie, a cruising ship, swung to her anchor


without a flutter of the sails, and was at rest. The tide
had come in, the wind was nearly calm, and being
Line bound down the river, the only thing for it was to come
5 to and wait for the turn of the tide. 1. The passage suggests that the narrator
The Director of Companies was our captain and our
host. We four affectionately watched his back as he A) regards his fellow sailors with benevolence.
stood in the bows looking toward the sea. On the whole B) is concerned that he has been at sea too long.
river there was nothing that looked half so nautical. He C) believes that the upcoming journey will be
10 resembled a pilot, which to a sailor is trustworthiness unbearable.
personified. It was difficult to realize his work was not D) eagerly awaits the stories told by his fellow sailors.
out there in the luminous estuary, but behind him,
within the brooding gloom.
Between us there was, as I have already said
15 somewhere, the bond of the sea. Besides holding our
hearts together through long periods of separation, it
had the effect of making us tolerant of each others
storiesand even convictions. The Lawyerthe best of
old fellowshad, because of his many years and many 2. Which choice provides the best evidence for the
20 virtues, the only cushion on deck, and was lying on the answer to the previous question?
only rug. The Accountant had brought out already a
box of dominoes, and was toying architecturally with A) Lines 2-5 (Thetide)
the pieces. Marlow sat cross-legged, leaning against B) Lines 6-8 (Thesea)
the mast. He had sunken cheeks, a yellow complexion, C) Lines 15-18 (Besidesconvictions)
25 a straight back, and, with his arms dropped, the palms D) Lines 29-32 (Afterwardsstaring)
of his hands outwards, resembled an idol. The Director,
satisfied the anchor had good hold, made his way
forward and sat down amongst us. We exchanged a few
words lazily. Afterwards there was silence on board
30 the yacht. For some reason or another we did not begin
that game of dominoes. We felt meditative, and fit for
nothing but placid staring.

28 28
PSAT Command of Evidence

Command of Evidence

Identifying Command of Evidence Questions on the PSAT: Command of Evidence


questions ask for which line(s) provide the best evidence for the answer to the
previous question.

On Command of Evidence questions, pick the lines with the main idea that most-closely
aligns with the answer to the previous question.

If unsure, eliminate down to as few choices as possible before choosing an answer.

I spent most of my childhood summers down the knew to desire was physical splendor, in which
road from home at Big Mommas house. We began category I was sorely lacking. I was the tallest girl
each day with the morning ritual she referred to as in my eighth-grade class, and when I tried to walk
Line her labor of love combing my hair. I would sit on 45 in dress shoes, my heels would slide out, causing
5 the porch floor with my feet swinging over its edge me to trip over myself. Naturally, my only concern
while my head bobbed back and forth between Big was ridding myself of awkwardness. Beauty was
Mommas legs as she tugged, parted, and braided something I saw only in others. A womans even-
my long, thick, nappy hair. Big Momma always sat colored skin and bright white teeth made her
perfectly upright, sucking in her breath with each 50 beautiful, never the inner peace that sparkled in
10 drag of the comb, then releasing the air from her her eyes. I greatly admired the little girls sunny
hollow Cherokee cheeks, never once bending her Easter dress, adorned with white bows and
back. After she finished the job, shed pat me on the ribbons, but gave no thought to the mother
head and say, Now you beautiful. Id rush to the needle in one hand, iron in the other, creating this
bathroom, stand on the toilet seat, and peer over the 55 lovely vision. And Big Mommas front lawn with its
15 sink into the mirror, eager to view this new and velvet violets, deep purple grape suckers, and
beautiful me. Of course, she never materialized. All yellow sunflowers floating in the air like balloons
I ever saw was my chubby face with a crown of was beautiful, but never once did I consider the
lopsided plaits and a mouth full of what my momma care they were given even as the flowers first
teasingly called beaver teeth because they looked 60 petals danced indiscriminately in the sunlight. I
20 large enough to saw wood. had always focused on my plainness, and it was
Besides our grooming, Big Momma and her band this sorry image of myself that I took with me to
of swearing sopranos made sure their offspring got a Youth-En-Camp that summer. Only later would I
proper Christian upbringing. Every Sunday there understand that real beauty emanates from the
was morning church school and Baptist Training 65 heart.
25 Union. And for one week every August the young
ones were herded to Grambling, Louisiana, a small 3. The author characterizes her time at Youth-En-
college town, for a gigantic statewide revival called Camp as a period of
Youth-En-Camp. Although the drive took only a few
A) adventure.
hours, it had the feel of a great adventure. This was
B) transition.
30 due in part to the parcel of sheets, dresses, and fried
C) learning.
chicken that always accompanied me but also
D) ease.
because the decreased supervision allowed me to
experience free will. It was during one of these
4. Which choice provides the best evidence for the
revivals that I became hopeful that I would one day
35 answer to the previous question?
look into the mirror and see beauty in myself.
I was thirteen at the time too old to be in one of A) Lines 16-20 (Allwood)
the crayon classrooms but still too awkward to be B) Lines 33-35 (It wasmyself)
cool. Before that summer Id never thought that I C) Lines 36-38 (I wascool)
could be beautiful perhaps cute, on a good day, but D) Lines 60-63 (I hadsummer)
40 never glamorous, radiant, or enchanting. Of course,
up to that point, the only form of beauty I

29 29
PSAT Command of Evidence Notes

30 30
PSAT Command of Evidence Review

Questions 1-10 are based on the following passage. between classes, a new social hierarchy, and a
broader view of the world. Critics of revisionism
This passage discusses the revisionist theory of the also argue that while the French Revolution did
origins and outcomes of the French Revolution. 50 not create capitalism, the power struggles and
structure of pre-revolutionary France were not
Edmund Burke first attacked the French conducive to capitalism. They argue that it was
Revolution in 1790 with his work the Reflections on the elites resistance to this new order that
the Revolution in France. In his writing, Burke caused the Revolution.
Line sharply criticized those who claimed that the 55 Revisionists maintain that the French
5 French Revolution was a necessity. Thomas Paines Revolution completed a process of centralization
famous work The Rights of Man was written in and reconstruction for France that was already
reply to Burkes analysis. The Rights of Man was materializing prior to the Revolution. They
both enormously influential and exceedingly speculate that the process of modernization and
popular in the English working class movements. 60 reconstruction could have occurred without the
10 However, Paines work angered the ruling class, violence and upheaval of a revolution.
and Paine was forced to flee England to France in Revisionists argue that some among the French
order to avoid arrest. ruling class were already pushing for the same
Until recently, the French Revolution was reforms that the Revolution ultimately achieved.
considered a bourgeois revolution, driven by class 65 Critics have responded to this claim by pointing
15 conflict, that destroyed the vestigial remnants of out that the most famous of these reforms was
feudalism and paved the way for capitalism in introduced in 1776 when French aristocrat and
France. This narrative is rooted in traditional economist, A.R. J. Turgot proposed to increase
Marxist interpretations of the social, economic, and trade, abolish guilds, and reform the tax system.
political forces at play in France and the rest of 70 The privileged elite were not supportive of these
20 Europe in 18th and 19th centuries. Revisionist measures and blocked Turgots efforts. Critics of
historians are now challenging this orthodox view revisionist theory use Turgots failure as
of events. evidence that the revolution was actually
Revisionists argue that the French Revolution necessary to effectuate change in France.
cannot be seen as a bourgeois revolution that
25 completely terminated feudal political structures.
The revisionist theory maintains that class 1. The author of the passage refers to vestigial
struggle played only a small role in the French remnants (line 15) in order to
Revolution and that the revolt had little or nothing
to do with capitalism. The revisionist scholars also A) illustrate the revisionist historians
30 argue that the bourgeoisie made up the elite class assertions about the consequences of the
in society. The bourgeoisie consisted of landowning French Revolution
gentry who tended to agree with each other on B) introduce a flaw in an argument that will be
political and social issues. Revisionists argue that discussed in the subsequent paragraph
the elite were generally in favor of the reforms that C) demonstrate a belief formerly held by some
35 the common people were demanding, and therefore monarchists
political reform would have been possible without a D) provide an example of a traditional Marxist
violent revolution. belief
While revisionists argue that the French
Revolution had little or no correlation to the
40 development of capitalism, critics of the revisionist 2. Which choice provides the best evidence for the
theory counter that revisionists ignore the answer to the previous question?
tremendous social and economic changes that had A) Lines 13-17 (UntilFrance)
been occurring prior to the Revolution. These B) Lines 17-20 (Thiscenturies)
critics also point out that the significant economic C) Lines 23-25 (Revisionistsstructures)
45 changes did not occur in isolation. The new D) Lines 45-48 (Theworld)
economic order required new relations

31 31
PSAT Command of Evidence Review

3. The author suggests that the elite members of 7. It can be inferred from the passage that the
social class were revisionists
A) compelled to argue against revisionist
A) never accept Marxist interpretations of
thinking.
historical events.
B) unaccustomed to political discord among
B) do not support the traditional interpretation of
themselves.
the French Revolution.
C) unaware of the common people.
C) favor a decentralized system of government
D) uncomfortable in each others presence.
over a strongly centralized system.
D) think that the French monarchy was actually a
strong supporter of the Revolution.
4. Which choice provides the best evidence for the
answer to the previous question?
A) Lines 23-25 (Revisionistsstructures) 8. Which choice provides the best evidence for the
B) Lines 31-33 (Theissues) answer to the previous question?
C) Lines 45-48 (Theworld)
A) Lines 13-17 (UntilFrance)
D) Lines 62-64 (Revisionistsachieved)
B) Lines 17-20 (Thiscenturies)
C) Lines 45-48 (Theworld)
D) Lines 55-58 (RevisionistsRevolution)
5. In line 39, the phrase little or no correlation
indicates that revisionists
9. According to the passage, which one of the
A) were unable to understand the link between
following statements is most consistent with the
the French Revolution and capitalism.
view held by revisionists?
B) were uncertain in their belief that capitalism
caused political reform.
A) Marxism is a failed ideology.
C) ignored major social and economic trends
B) Non-violence is the most effective means of
brought about by the French Revolution.
achieving social reform.
D) did not believe that the French Revolution
C) There were causes of the French Revolution
impacted the development of capitalism.
that cannot be adequately explained by a
classical Marxist construction.
D) There was no genuine desire among the
6. Which choice provides the best evidence for the
working class to promote capitalism during the
answer to the previous question?
French Revolution.
A) Lines 26-29 (Thecapitalism)
B) Lines 33-37 (Revisionistsrevolution)
C) Lines 45-48 (Theworld) 10. Which choice provides the best evidence for the
D) Lines 52-54 (TheyRevolution) answer to the previous question?
A) Lines 5-9 (Thomasmovements)
B) Lines 33-37 (Revisionistrevolution)
C) Lines 38-43 (WhileRevolution)
D) Lines 52-54 (Theyrevolution)

32 32
PSAT Command of Evidence Answers

Workbook Answers Review Answers

Command of Evidence 1) D

1) A 2) B

2) C 3) B

3) B 4) B

4) C 5) D

6) A

7) B

8) D

9) C

10) B

33 33
PSAT Analyzing Multiple Texts

PSAT Reading:
Analyzing Multiple Texts

34 34
PSAT Analyzing Multiple Texts

Approaching Multiple Texts

Identifying Analyzing Multiple Texts Questions on the PSAT: One PSAT reading
portion is comprised of two passages which are followed by questions that relate to both
passages.

Approaching Multiple Texts on the PSAT: Multiple texts selections always share a
common theme/topic, with each passage having a different perspective on that
theme/topic. A Venn Diagram can help to organize this information for making
anticipations.

Solving Multiple Texts Questions on the PSAT: There are four steps to solving
Multiple Texts questions on the PSAT.

1) Cover the answer choices. Do not look at the answer choices!

2) Anticipate an answer to the question. How would YOU answer the question. Use the
notes you took while reading!

3) Eliminate answers that are clearly incorrect. Answers are clearly incorrect because
they do not align with your anticipation!

4) Make a decision among whatever choices remain. Avoid answers with absolute
language and/or non-relevant information.

35 35
PSAT Analyzing Multiple Texts

Analyzing Multiple Texts

Identifying Analyzing Multiple Texts Questions on the PSAT: One PSAT reading
portion is comprised of two passages which are followed by questions that relate to the
passages.

Analyzing Multiple Texts Strategy: After reading each individual passage, stop and
answer only the questions that relate to that passage. After answering all questions
that relates to one passage, then answer the questions that refer to multiple
passages.

Passage 1
1. The author of Passage 1 compares Lewis and
Before Lewis and Clark, the territory west of
Clark to Christopher Columbus (lines 3-6) primarily
the Mississippi River was about as well
in order to
understood as outer space almost as
Line mysterious as the Americas were to Christopher
A) help the reader understand their methods.
5 Columbus the day he dropped anchor off El
B) characterize them as oppressors.
Salvador. It was home to the red nations, yet an
C) highlight their fearlessness.
enigma to recently arrived whites. French and
D) emphasize how little is known about their
Spanish traders made inroads along the major
expedition.
waterways, yet little was known of this vast slice
10 of continent other than what could be gleaned
from rumors that blended a dash of speculation
with boundless exaggeration. It was our very
own African heart of darkness, with a handful of
2. Both passages are concerned with
men proceeding cautiously upriver to obtain furs
15 from the various native tribes and courting
A) understanding the truth surrounding Lewis and
death in the process.
Clarks discoveries
B) links between the past and the future
Passage 2
C) specific discoveries made during Lewis and
Clarks expedition
In many histories of the American West, the
D) perceptions of the regions explored by Lewis and
Lewis and Clark expedition marks the beginning
Clark
of recorded history; earlier history of the region
20 is relegated to prelude. The Lewis and Clark
expedition was a momentous event and a heroic
achievement, but it was only a subplot in a
historical drama of time, place, and people that
had been playing for thousands of years. The
25 new lands Lewis and Clark explored were in
fact very old. The West they saw had been
shaped by many other histories and ways of
understanding history involving other peoples
and recorded not only in writing but in song and
30 story and earth and memory. Lewis and Clark
did not bring the West into U.S. history, they
brought the United States into western history.

36 36
PSAT Words in Context

Analyzing Multiple Texts

Identifying Analyzing Multiple Texts Questions on the PSAT: One PSAT reading
portion is comprised of two passages which are followed by questions that relate to the
passages.

Analyzing Multiple Texts Strategy: After reading each individual passage, stop and
answer only the questions that relate to that passage. After answering all questions
that relates to one passage, then answer the questions that refer to multiple
passages.

3. The author of Passage 2 would most likely


characterize the other histories (line 27) as

A) essential to a complete understanding of the


American West.
B) intentional falsifications of Native American
history.
C) accurate records based on the journals of
Lewis and Clark.
D) one-sided.

4. The two passages differ in their views of the


Lewis and Clark expedition in that Passage 1
regards it
A) through the eyes of a foreigner, while
Passage 2 assumes a distinctly American
outlook.
B) as a major discovery, while Passage 2
diminishes its importance.
C) from the perspective of Lewis and Clarks
era, while Passage 2 views it in a broader
historical context.
D) with complete ignorance of the facts, while
Passage 2 approaches it with an informed
understanding.

37 37
PSAT Analyzing Multiple Texts Notes

38 38
PSAT Analyzing Multiple Texts Review

The first passage below is an excerpt from a African-American artists dried up. The cultural life
historical survey of early twentieth-century of Harlem seemed to fade slowly into increasing
American society; the second is an excerpt from a Line poverty and bleakness. We were no longer in
1997 article about the Harlem Renaissance. 45 vogue, wrote Jervis Anderson in This Was
Harlem.
Passage 1 Nevertheless, the great legacy of the Harlem
Renaissance has continued to enrich and influence
Hot nights and cool jazz . . . steamy sidewalks American life. The artists who made Harlem their
and fancy dressers . . . songs of the soul and songs 50 home in the 1920s continue to inspire Americans of
of the body . . . the lilt of laughter and the wail of all colors today.
Line the blues . . . .
5 That was Harlem in the 1920s a place that Passage 2
vibrated with excitement, promise, glitter, and joy.
If you had visited Harlem in those days, you might For some, the Harlem Renaissance evokes the
have heard bandleader Duke Ellington playing flourishing of intellectual and artistic output
Take the A Train at the Cotton Club, or Louis exemplified by writers such as Langston Hughes
10 Satchmo Armstrong shaking up the jazz world 55 and Zora Neale Hurston. For others, it conjures
with his trumpet at Connies Inn. Harlem was also images of bacchanalia: the long-legged beauties of
the home of African-American poets, novelists, the Cotton Club and those dark, seedy speakeasies
actors, and philosophers. So great was the cultural such as the Clam Bake and the Hot Feet.
explosion of Harlem during the 1920s that the In that postwar environment of social and
15 period has since been called the Harlem 60 sexual adventure, Harlem symbolized liberation
Renaissance. Harlem has always been measured by for black and white alike. In the great migration of
a spirit and way of life that have touched all of African-Americans from the rural South to the
America and have created a legacy that continues cities, the district became a Mecca for streams of
to inspire today. black writers, musicians, performers, and
20 Tens of thousands of black people flocked to 65 filmmakers, a refuge from the all-pervasive racism
Harlem looking for a better life in the 1920s, of American society.
among them a star-studded group of artists. All But those few city blocks were also a
were eager to bask in the freedom of city life and playground for affluent whites. Emboldened by
the growing excitement of Harlem. Unlike the bootleg liquor, they turned this little hamlet of
25 Souths cities and towns, New York City made 70 New York into their own exotic laboratory, where
African-Americans feel free to express themselves, they could experiment with what was forbidden in
to create, to fully tell the story of the African- their own world, enjoy what they saw as primal
American experience in words, pictures, paintings, and erotic, then leave it behind a few hours later.
and, most popularly, music. The Harlem Renaissance has become so
30 In the 1920s, African-American music was all 75 emblematic that it can be hard to separate the
the rage. Every night, white people took taxis and myth from the reality. Was this Harlem a place or
subways uptown to Harlem to listen and dance to just a state of mind? Did it really change black
music by black musicians and singers. White cultural life forever, as some historians claim? Or
publishers went to Harlem to find black writers was it simply a handful of privileged black artists
35 and poets. Black people, in Harlem at least, began 80 patronized by rich, white Afrophiles?
to feel that they were an important part of the Certainly todays black artists are keen to claim
nations cultural life. a cultural legacy bridging their work to the heroes
But whatever Harlem did for black people of Harlem. But it has also been argued that the
during the 1920s, the Stock Market crash (1929) Renaissance represented nothing more than a
40 and the Great Depression (1929-1935) took it away. 85 bourgeois playpen, retrospectively endowed with
Suddenly, the money that had supported cultural legitimacy by academia and the black

39 39
PSAT Analyzing Multiple Texts Review

middle class. Perhaps we celebrate Harlem at the 3. The statement by Jervis Anderson in lines 44-45
expense of more radical periods, for example, the implies that
Line labor movement of the 1930s that spawned
90 Richard Wright, arguably the greatest African- A) whites stopped frequenting Harlem altogether.
American novelist. Langston Hughes, one of the B) the later figures of the Harlem Renaissance
luminaries of the Harlem Renaissance, admitted at were less talented than the earlier ones.
the time: The ordinary Negro hasnt heard of the C) another, more exciting cultural movement
Negro Renaissance. And if they had it didnt raise supplanted the Harlem Renaissance.
95 their wages any. D) the Harlem Renaissance was a fad.
Indeed, skeptics argue that the plunging rates
of literacy and increasing sense of crisis in
Americas black underclass prove the Harlem 4. The sentence beginning Emboldened by bootleg
Renaissances influence has been only one class liquor (lines 68-69) suggests that
100 deep. Great novels may have come from Harlem,
but how many there now can read them? A) Harlem provided whites a temporary refuge
Part of the energy that fuelled the Harlem from traditional social restrictions.
Renaissance was the belief that black cultural B) social norms were too limiting for most whites.
achievement in the high arts would socially and C) whites preferred African-American culture to
105 spiritually uplift the race. Manifestly this has not their own.
happened. Where the legacy of the Harlem D) whites did not enjoy themselves in Harlem.
Renaissance remains a profoundly romantic one
for the black bourgeoisie, on the streets, where the
great majority of black culture is made, its echoes 5. According to Passage 1, whites
110 are only faintly heard.
A) took unfair advantage of the talents of Harlem
artists.
1. The opening paragraph of Passage 1 relies on B) offered Harlem artists additional channels for
their art.
A) historical facts. C) were solely responsible for the mainstream
B) personal anecdotes. success of Harlem artists.
C) sensory details. D) were opposed to the goals of the Harlem
D) an extended metaphor. Renaissance.

2. In line 41, dried up most nearly means

A) ran out.
B) hardened.
C) withered.
D) grew brittle.

40 40
PSAT Analyzing Multiple Texts Review

6. In line 81, keen most nearly means 9. The author of Passage 2 would most likely
respond to the authors view in Passage 1 of
A) intelligent. Harlems spirit and way of life (line 17) by
B) sharp.
C) intense. A) praising the authors historical insight.
D) eager. B) criticizing the authors oversimplification.
C) lamenting the authors misinterpretation of
Harlems spirit and way of life.
7. Passage 2 differs from Passage 1 in that D) mocking the authors attempts at flattery.

A) Passage 2 ignores the racial aspect of the


Harlem Renaissance; Passage 1 focuses on it. 10. Which choice provides the best evidence for the
B) Passage 2 considers the relevance of the answer to the previous question?
Harlem Renaissance today; Passage 1 centers
A) Lines 61-65 (Insociety)
on the actual experience of the Harlem
B) Lines 76-80 (WasAfrophiles?)
Renaissance.
C) Lines 102-105 (Partrace)
C) Passage 2 underscores the social and historical
D) Lines 106-110 (Whereheard)
context of the Harlem Renaissance; Passage 1
assumes the reader has full knowledge of this
context.
D) Passage 2 praises the legacy of the Harlem
Renaissance; Passage 1 examines it critically.

8. Both passages suggest which of the following


about the major figures of the Harlem
Renaissance?

A) Many of them were not from Harlem.


B) They resented whites encroaching on their
territory.
C) They wanted to help elevate the black
underclass.
D) Their art had an immediate impact on all
people.

41 41
PSAT Analyzing Multiple Texts Answers

Workbook Answers Review Answers

Analyzing Multiple Texts 1) C

2) A
1) C
3) D
2) D
4) A
3) A
5) B
4) C
6) D

7) B

8) A

9) B

10) D

42 42
PSAT Evaluating Quantitative Information

PSAT Reading:
Evaluating Quantitative
Information

43 43
PSAT Evaluating Quantitative Information

Evaluating Quantitative Information

Identifying Evaluating Quantitative Information Questions on the PSAT: On the


PSAT, quantitative information questions assess a students ability to understand a chart
or graph and connect it to a passage.

To solve quantitative information questions, there are two things to keep in mind:

1) What is the data about?: This can be figured out by answering the question
What does the header of the graphic say?

2) What does the data in the graphic show about this topic?: It is
important to look for trends in the data (When one thing increases, another
thing increases; some things are above average and some are below average,
etc.).

To be correct, an answer must be related to the answers to these questions.

Police
Percent Increase in Employment, Projected 2015-2025

Total, All Occupations

Police Officers

Fire Fighters

0% 2% 4% 6% 8% 10% 12% 14% 16% 18% 20%

The United States Bureau of Labor 1. Which choice completes the sentence with accurate
Statistics forecasts job growth in city data based on the graph?
governments, projecting that 20 percent of new
jobs in all occupations will be related to police. A) NO CHANGE
B) warning, however, that job growth for police will
slow to 10 percent by 2025.
C) predicting that police employment will increase 20
percent between 2015 and 2025.
D) indicating that 10 to 14 percent of police jobs will
remained unfilled.

44 44
PSAT Evaluating Quantitative Information

Evaluating Quantitative Information

Identifying Evaluating Quantitative Information Questions on the PSAT: On the


PSAT, quantitative information questions assess a students ability to understand a chart
or graph and connect it to a passage.

Questions 2 and 3 refer to the below information

Heart rate of five 20 year olds


240
220
Heart rate beats/minute

maximum
200
desirable level
180
160
140
120
100
80
60
40
Walking Running

A recent analysis of heart rates was 2. What information discussed in the paragraph is
conducted on a group of healthy 20 year old represented by the graph?
humans. This analysis revealed that for these
Line people, their heart rates in beats per minute A) The information in lines 3-7 (This . . .olds)
5 increased substantially when running, yet the B) The information in lines 7-10 (The . . .exercise)
heart rates remained below the maximum C) The information in lines 10-12 (Exercise . . .
desirable level for 20 year olds. The researchers concluded)
reported that for these healthy humans, the D) The information in lines 12-14 (Its . . . ailments)
increase in heart rate was related to the bodily
10 strain caused by the aerobic exercise. Exercise
is highly beneficial for humans, the study
concluded. Its an activity which decreases the 3. Which statement about the effect of running on 20
risk for high blood pressure and other cardiac year olds is best supported by the graph?
ailments.
A) Only one of the people did not experience an
appreciable change in heart rate when running.
B) All of the people remained above the minimum
desirable heart rate for humans.
C) All of the people remained below the maximum
desirable heart rate for humans.
D) The person with the lowest walking heart rate had
the highest running heart rate.

45 45
PSAT Evaluating Quantitative Information

Evaluating Quantitative Information

Identifying Evaluating Quantitative Information Questions on the PSAT: On the


PSAT, quantitative information questions assess a students ability to understand a chart
or graph and connect it to a passage.

Question 4 refers to the below information

Table 1: Tiger Woods Major Championships Won By Time Period

1997-2000 5
2001-2004 3
2005-2008 6
2009-2012 0
0 2 4 6 8

Tiger Woods won the 2008 U.S. Open in 4. Which choice best completes the sentence based on
sudden death, even while enduring a double the information provided in Table 1?
stress fracture in his left tibia. He is no stranger
Line
to overcoming hurdles as he lives out his basic A) NO CHANGE
5 principle: If youre not going forward, youre B) and it may be a challenge for Tiger Woods to regain
going backward. Tiger will take some time to his pre-injury form in major championships.
heal, 4 but will soon be back on the golf course C) and Tiger Woods will never win another major
winning major tournaments and doing what championship after he returns to golf.
comes so naturally. D) but Tiger Woods will be just as competitive as
before his injury.

46 46
PSAT Evaluating Quantitative Information Notes

47 47
PSAT Evaluating Quantitative Information Review

Questions 1 and 2 refer to the below information

Number of Users (in millions) Facebook Users By Month


1500

1200

900

600

300

0
2007 2008 2009 2010 2011 2012 2013

Year

1. Which statement best summarizes the 2. According to the graph, which statement is true
information presented in the graph? about the number of Facebook users in 2013?

A) Far more people around the world used the A) Growth in the number of Facebook users in 2013
internet in 2013 than in 2007. was widely out of proportion to growth during
B) The number of people using Facebook each the three years prior.
month more than tripled from 2010 to 2013. B) The growth in the number of Facebook users
C) The number of people using Facebook each began a new upward trend.
month increased tremendously from 2007 to C) The growth in the number of Facebook users
2013. reached its peak.
D) The number of Facebook users each month is D) The number of Facebook users surpassed 1.2
likely to be almost 3 billion in 2015. billion for the first time.

48 48
PSAT Evaluating Quantitative Information Review

Questions 3 and 4 refer to the below information

Mosquito Morality Rate in the United States


40%

35%
Percent Morality Rate

Range of Desired Eradication


30%

25%

20%

15%

10%

5%

0%
2009-2010 2010-2011 2011-2012 2012-2013 2013-2014 2014-2015

Winter Season

Since 2009, several states have experimented 3. Which choice offers the most accurate
with spraying pesticides during winter months interpretation of the data in the chart?
in the hope of reducing the number of mosquitos
Line that emerge during the summer. As a result of A) NO CHANGE
5 this spraying, drops in mosquito populations B) been below the acceptable range.
have 3 exceeded 10 percent of the population C) not changed noticeably from year to year.
each winter. In an attempt to increase the D) greatly increased every year.
effectiveness of this program, during the winter
of 20112012, the states changed this chemical
10 used during the process, 4 and the number of
insects killed has increased every year since.

4. Which choice offers the most accurate


interpretation of the data in the chart?

A) NO CHANGE
B) the percentage of insects killed has been within
the range of desired eradication each winter
since 2011-2012.
C) the percentage of mosquitos killed has exceeded
the range of desired eradication each winter
since 2011-2012.
D) the percentage of mosquitos killed has increased
each winter since 2011-2012.

49 49
PSAT Evaluating Quantitative Information Answers

Workbook Answers Review Answers

Evaluating Quantitative Information


1) C

1) C 2) D

2) A 3) A

3) C 4) D

4) B

50 50
PSAT Words in Context

PSAT Reading:
Words in Context

51 51
PSAT Words in Context (Reading)

Words in Context (Reading)

Identifying Words in Context Questions on the PSAT: On Words in Context


questions, students are asked to define a word within the context of the surrounding text.
No prior knowledge of the word is necessary.

To solve Words in Context questions, first anticipate a word or tone that fits within the
surrounding text. Then, use your anticipation to eliminate answers that do not align
with your anticipation.

Question 1 Refers to the Paragraph Below

I am not saying that a good literary 1. As it is used in the paragraph, shortchanged


education is sufficient to create morally sensitive most nearly means
human beings, but keeping children ignorant of
their moral heritage is one way to get ourselves A) penniless
a generation of morally shortchanged human B) robbed
beings. Children who are basically honest but C) deficient
who have not been taught and so dont know D) underpaid
that lying and cheating are wrong or cannot say
why they are wrong; children who are
compassionate but who arent sure that cruelty
in exotic societies is just as wrong as it is in our
own. Such students are being cheated of their
moral heritage that is the glory of our nation.
For all our children are rightful heirs to the
Judeo-Christian tradition and the European
Enlightenment that taught our Founding
Fathers their morals and politics.

Question 2 Refers to the Paragraph Below

It can be difficult to ask tough questions 2. As it is used in the paragraph, fabric most
about immigration when we see nostalgic images nearly means
of Ellis Island, recall our own families coming to
America, or encounter a new immigrant who is A) structure
striving admirably to achieve the American B) cloth
dream. But tough questions about immigration C) weaving
can no longer be avoided as we enter a fourth D) surface
decade of unprecedentedly high immigration and
struggle with its impact on job markets, on the
quality of life and social fabric of our
communities, and on the state of the
environment.

52 52
PSAT Words in Context (Writing)

Words in Context (Writing)

Identifying Words in Context Questions on the PSAT: On Words in Context


questions, students are asked to define a word within the context of the surrounding text.
No prior knowledge of the word is necessary.

To solve Words in Context questions, first anticipate a word or tone that fits within the
surrounding text. Then, use your anticipation to eliminate answers that do not align
with your anticipation.

Question 3 Refers to the Paragraph Below

I also think she wanted me to go there 3. A) NO CHANGE


because she was worried Id be lost if she sent B) moral
me south to a historically black college or some C) innocent
big university. I was young and sheltered, and D) inexperienced
when it came to racial politics, I was largely
harmless. I was completely unprepared to play
the role people would want me to play as the
daughter of an African-American hero. I had not
a clue who I was, either as the daughter of
Malcolm X or even as simply myself, Ilyasah
Shabazz. Mommy probably worried that if I
enrolled at a historically black college, the
expectations she had so carefully and thoroughly
shielded me from my entire life would rise up
like a tidal wave and swallow me whole.

Question 4 Refers to the Paragraph Below

Everyone wanted to see the men on the 4. A) NO CHANGE


mountain. Gutzon Borglum, who regarded his B) blow
masterpiece as far more than a tourist C) utter
attraction, was no doubt reassured when the D) perspire
phrase Shrine of Democracy was coined at the
1930 dedication of the Washington head. A
monuments dimensions, said Borglum, should
be determined by the importance to civilization
of the events commemorated . . . Let us place
there, carved high, as close to heaven as we can,
the words of our leaders, their faces, to show
posterity what manner of men they were. Then
send a prayer that these records will endure
until the wind and the rain alone shall wear
them away.

53 53
PSAT Words in Context Notes

54 54
PSAT Words in Context Review

Question 1 Refers to the Paragraph Below

So, superstitious or not, the Chinese, 1. As it is used in the paragraph, confirm most
Muslims, and Babylonians did their scientific nearly means
duty. Without their records we would not be able
to confirm the dates of many key historical A) authorize
events including the birth of Christ. Where B) state
would the millennium celebrations be then? C) demonstrate
D) verify

Question 2 Refers to the Paragraph Below

It was the week before the school session 2. A) NO CHANGE


began. After a long summer, the returning B) pushed
students were enjoying their free time, realizing C) abused
that there was not a moment to lose. Facing this D) mesmerized
scene, I was shaken and awed. The reason was
simple. Three months before, I had stood in
Tiananmen Square. My mind recalled the
indignation of the demonstrators, the students
on hunger strike, the banners, the slogans, and
the loudspeakers.

Question 3 Refers to the Paragraph Below

Guiding Richies hands with her own, she 3. A) NO CHANGE


helps him dip the cup into the flour. The cup B) frees
goes in easily, and through its thin wall he can C) erases
feel the silkiness and slight grit of the sifted D) dismisses
flour. A tiny cloud rises in the cups wake.
Mother and son bring it up again, heaped with
flour. Flour cascades down the silver sides.
Laura tells the boy to hold the cup steady, which
he nervously manages to do, and with one quick
gesture she rejects the grainy little heap on top
and creates a flawless white surface exactly level
with the lip of the cup. He continues holding the
cup with both hands.

55 55
PSAT Words in Context Review

Question 4 Refers to the Paragraph Below

How do I explain my concept of married 4. As it is used in the paragraph, sterile most


feminist? I am a woman who feels pulled in two nearly means
directions between traditional values and
conventions on one hand and a commitment to A) purified
feminist ideology on the other. A woman who B) childless
finds custom appealing and comforting, yet C) uninspired
despises the patriarchal patterns that make D) clean
women second-class citizens. I am a woman who
wants a loving, long-term relationship with a
man but bitterly resents being considered only
someones other half. A woman who values
family life, but deplores the sterile, functional
view of man as head of the family and woman as
its heart.

Question 5 Refers to the Paragraph Below

Helen picked out a flat fish, pom-pom fish, 5. As it is used in the paragraph, slipped away
she called it, only a dollar sixty-nine a pound, most nearly means
bargain bin.
And I said, This kind of bargain you dont A) escaped
want. Look at his eye, shrunken in and cloudy- B) fled
looking. That fish is already three days old. C) departed
But Helen stared at that fish eye and said D) snuck
she saw nothing wrong. So I picked up that fish
and felt its body slide between my fingers, a fish
that had slipped away from life a long time ago.
Helen said it was a good sign a juicy, tender
fish!

56 56
PSAT Words in Context Answers

Workbook Answers Review Answers

Words in Context
1) D
1) C
2) A
2) A
3) C
3) D
4) C
4) C
5) C

57 57
PSAT Writing

PSAT Writing:
Table of Contents

58 58
PSAT Writing: Table of Contents

Writing: Table of Contents

PSAT Writing: Introduction (Page 61)

Adjectives and Adverbs (Page 63)

Parallelism (Page 68)


Parallelism Basics
Correlative Conjunctions

Passage Analysis (Page 74)


Adding Precision
Introductions and Conclusions
Main Idea Analysis
Sequence

Prepositions (Page 83)

Pronouns (Page 88)


Singular vs. Plural
Personal Pronouns
Possessive Pronouns
Pronoun Case
Unclear Pronoun Subject

Punctuation (Page 97)


Comma Usage
Colon Usage
Contractions
Dash Usage
Possession
Run-on Sentences and Sentence Fragments

Transitions (Conjunctions) (Page 107)

Verbs (Page 112)


Subject-Verb Agreement
Verb Tense

Word Choice (Page 118)


Frequently Confused Words
Inconsistent Tone and/or Style

Wordiness and Redundancy (Page 124)


Redundancy
Wordiness

Advanced Grammar (Page 130)


Affect vs. Effect
Logical Comparisons
Misplaced Modifiers
Who, Whom, That, Which, and Whose

59 59
PSAT Writing

PSAT Writing:
Introduction

60 60
PSAT Writing: Introduction

PSAT Writing Introduction

Test 2: Writing

35 Minutes for 44 Questions


Four passages

Concepts covered on PSAT Writing:

Basic Grammar and Syntax


o Prepositions, punctuation, verbs, etc.
Main Idea Analysis
Quantitative Analysis
Words in Context

Quick Writing Tips


Trust your grammar ear!
Stay confident you know all the grammar you need to know.
Identify the grammatical issue that is being tested.
Dont make a problem harder than it is.
Dont make stupid mistakes stay focused.

61 61
PSAT Adjectives and Adverbs

PSAT Writing:
Adjectives and Adverbs

62 62
PSAT Adjectives and Adverbs

Adjectives and Adverbs

Adjectives are words or phrases that modify nouns and pronouns.

Adverbs modify everything but nouns and pronouns (adjectives, verbs, other
adverbs).

Identifying Adverb Issues on the PSAT: Look for adjectives describing words or
phrases that are not nouns or pronouns, or for adverbs describing words or phrases that are
not verbs, adjectives, or other adverbs.

1. As developed by Read, the Shaw Alphabet contains 48 letters each with only one
pronunciation as well as eight different vowel markers. As Shaw had specified, it is
essentially a purely phonetic system.

A) NO CHANGE
B) pure phonetic
C) purely phonetically
D) pure phonetically

2. Today, people have moved so easy from paper to paperless transactions that swiping a card
is the norm.

A) NO CHANGE
B) so easiest
C) so easily
D) so easier

3. James approached the test confident and diligent, trusting all of the studying he had done
the week before the test.

A) NO CHANGE
B) approached the test with confidence and diligence
C) approached the test confidently and with diligence
D) approached the test confidently and diligently

63 63
PSAT Adjectives and Adverbs Notes

64 64
PSAT Adjectives and Adverbs Review

1. Abagail wanted to keep the party a secret, so she had to plan stealthy and talk discreet in order to not let
the cat out of the bag.

A) NO CHANGE
B) plan stealthily and to talk with discreetness
C) plan stealthily and to talk discreetly
D) plan stealthy and talk discreetly

2. Shamu, a world famous orca whale, was an important factor in the growth of the SeaWorld park in San
Diego, California.

A) NO CHANGE
B) importantly a factor
C) an important factor who took part in
D) the most importantly factor

3. This year, Americans acted very swift to file their taxes on time and thus avoid fines from the
government.

A) NO CHANGE
B) very swifter
C) very swiftly
D) very swiftest

4. One worry of walking through tall grass is Lyme Disease, an infectious disorder caused by bacteria. Lyme
disease can cause joint pain by the severe swelling joints.

A) NO CHANGE
B) severe swelling
C) the severely swelling of
D) severely swelling

5. My tutor said that anxiety is a complete understandable reaction to taking the SAT. However, with
proper preparation those worries can be transformed into confidence.

A) NO CHANGE
B) complete, understandably
C) completely understandable
D) completely, understandable

65 65
PSAT Adjectives and Adverbs Answers

Workbook Answers Review Answers

Adjectives and Adverbs


1) C

1) A 2) A

2) C 3) C

3) D 4) D

5) C

66 66
PSAT Parallelism

PSAT Writing:
Parallelism

67 67
PSAT Parallelism

Parallelism

Proper grammar requires that all items in a list or all related clauses be in the same
grammatical form (nouns, verbs, correlative conjunctions, etc.).

Identifying Parallelism Issues on the PSAT: Look for two or more items or clauses in a
list.

1. The local representative considers the new law to be a great success: it improves the quality
of roadways, creates many jobs, and it saves the state money.

A) NO CHANGE
B) saving
C) saves
D) will save

2. A green space is set aside in a downtown area. A baseball field is built to encourage youth
fitness. To mitigate difficulties in traveling to the field, a new bus stop is established.

Which choice best maintains the sentence pattern already established in the paragraph?

A) NO CHANGE
B) A new bus stop can help mitigate difficulties in traveling to the field.
C) A new bus stop is established to mitigate difficulties in traveling to the field.
D) By a new green space, a new bus stop is established to mitigate travel difficulties.

3. Although we were sitting in last row, the concert was as exciting to us as the people sitting
adjacent to the stage.

A) NO CHANGE
B) people
C) as it was
D) as to the people

68 68
PSAT Parallelism: Correlative Conjunctions

Parallelism: Correlative Conjunctions

Correlative conjunctions are conjunctions that come in pairs, and you have to use both of
them to maintain parallelism in a sentence.

Identifying Parallelism Issues: Correlative Conjunctions on the PSAT: Look for


conjunctions in the below pairs:

both and
either or
neither nor
not but
not only but also

1. The shopping malls construction is not going according to plan this morning. Neither the
foreman or the building materials has arrived on time.

A) NO CHANGE
B) Either the foreman or the building materials
C) Either the foreman nor the building materials
D) Neither the foreman nor the building materials

2. During the early part of the 21st century, companies have increasingly promoted fitness for
employees. In light of the resulting benefits not only to employee productivity and again to the
bottom line of corporations in the form of reduced health care costs, an expansion of these
programs can be expected.

A) NO CHANGE
B) but it benefits
C) as also to
D) but also to

69 69
PSAT Parallelism Notes

70 70
PSAT Parallelism Review

1. The tsunami tore through the small ocean-side town, hurling boats and cars, engulfing farmlands, and
houses were swept away.

A) NO CHANGE
B) houses swept away
C) sweeping houses away
D) swept away houses

2. Renowned for fighting hard, marching hard, and obedience, the Vermont Brigade earned a reputation as
one of the finest units in the Army of the Potomac.
A) NO CHANGE
B) for its obedience
C) being obedient
D) obedient

3. Lighthouse devotees tend to be passionate: they enjoy decorating their homes with lighthouse
memorabilia, putting bumper stickers on their cars, and to wear clothes emblazoned with their favorite
lighthouses.
A) NO CHANGE
B) wearing
C) having to wear
D) to be wearing

4. Gall and Medek are known for working with builders during the design stages of a project to maximize
the economy of materials, to achieve ecological sustainability in construction processes, and building
successful working relationships with all the project stakeholders.

A) NO CHANGE
B) in building
C) she built
D) to build

5. Edward Abbeys essays earned him praise and secured his position as a leading American
environmentalist during the late 1960s, a period during which he wrote prolifically.
A) NO CHANGE
B) earn him praise or secure his position to be
C) earned him praise and secure his position as
D) would earn him praise and secured his position as that of

71 71
PSAT Parallelism Answers

Workbook Answers Review Answers

Parallelism
1) C

1) C 2) C

2) C 3) B

3) D 4) D

Parallelism: Correlative Conjunctions 5) A

1) D

2) D

72 72
PSAT Passage Analysis

PSAT Writing:
Passage Analysis

73 73
PSAT Passage Analysis: Adding Precision

Passage Analysis: Adding Precision

Identifying Passage Analysis: Adding Precision Questions on the PSAT: The


question will ask which answer provides the most relevant details. Look for the answer
that aligns with the main idea and provides the most vivid imagery through the use of
nouns, adverbs, and adjectives.

Question 1 Refers to the Paragraph Below

There are four categories of specific phobias 1. Which choice best completes the description of
covering over 350 distinct fears, including phobias if added to the end of the underlined text?
arachnophobia (fear of spiders), acrophobia (fear
of heights), trypanophobia (fear of needles), and A) NO CHANGE
my favorite claustrophobia (fear of enclosed B) like a rapid heartbeat and shortness of breath.
spaces). In many cases, persons facing a phobia C) and act frightened.
will show visible signs of panic. Often, people D) whenever they confront enclosed spaces.
begin to fear the very possibility of a phobic
attack. It has happened to me at work. Once, a
co-worker found me trembling outside the
supply closet, trying to get up the courage to pick
up a new ink cartridge; I was immobilized by
fear.

Question 2 Refers to the Paragraph Below

Wildlife and livestock are killed, plants are 2. Which choice provides the most specific
consumed, and hundreds of homes are burned to information about the devastation brought by
the ground. Fall means calling friends to see if wildfires?
they need a place to stay or packing up my own
things to seek shelter somewhere else. The A) NO CHANGE
danger isnt over once the fires are put out, B) The wildfires kill many plants and animals and
either. Winter in California means our annual also destroy homes.
allotment of rain and the potential for C) Wildfires can be very devastating.
landslides. These landslides can pollute water D) Raging fires burn out of control.
sources and result in the loss of the topsoil
necessary to support new growth. Instead,
stripped hillsides are extremely unstable. This
can also pose a threat to homes in the area.

74 74
PSAT Passage Analysis: Introductions and Conclusions

Passage Analysis: Introductions and Conclusions

Identifying Passage Analysis: Introductions and Conclusions Questions on the


PSAT: The question will ask what answer best introduces or concludes a given
paragraph or passage. Any introduction or conclusion must connect to the main idea(s)
of the paragraph or passage.

Question 1 Refers to the Paragraphs Below

Garveys radical ideas were far from


universally popular, even among members of the 1. Which choice most effectively sets up the
African American community. His most powerful second paragraph?
enemies came due to his controversial stances
against the National Association for the A) Garvey made many people angry.
Advancement of Colored People (NAACP) and B) History shows that Garvey performed some
the peaceful Civil Rights Movement. Garvey imprudent actions during his career.
argued that the NAACP only supported and C) The NAACP still exists today.
helped certain members of the African American D) As a testament to Garveys success, his ideas
population. His harsh comments against the were very bold.
popular NAACP and the Civil Rights Movement
turned many important people against him.
However, it must be remembered that he
devoted his life to a cause in which he believed.
Garveys views never wavered even when he was
harshly criticized. Today, there is a United
Negro Improvement Association that promotes
many of the same values that Garvey preached
in the 1920s. Marcus Garvey is dead, but his
pride and goals live on in todays dreamers, and
his writings continue to inspire leaders around
the world

Question 2 Refers to the Paragraph Below

Today, thanks in part to the Hart-Cellar Act, 2. Which of the following offers the best
the size of these groups has tripled or introduction to the paragraph?
quadrupled. This increase has allowed the
diversity of the immigrant population to be felt A) Many people emigrate to the United States
in areas that, previously, had not had the in order to achieve personal freedom that
experience. In the 1800s, immigrants from may not have been available to them in their
Europe tended to settle on the East Coast or in country of origin.
the Midwest. In the 21st century, while B) Only 40 years ago, Asians and Hispanics
immigrants continue to relocate to the made up an infinitesimal percentage of the
traditional areas, they have increasingly decided American population.
to settle on the West Coast, in the Southwest, C) Birth rates have been increasing in the
and in the Southeast. United States for many reasons.
D) The total number of Americans immigrating
to other countries has also been recently
studied.

75 75
PSAT Passage Analysis: Main Idea Analysis

Passage Analysis: Main Idea Analysis

Identifying Passage Analysis: Main Idea Analysis Questions on the PSAT: The
question will ask what meaning will be added or lost if a word or phrase is added,
deleted, or modified. Any meaning that is lost must connect to the main idea(s) of the
surrounding text.

Question 1 Refers to the Paragraph Below

The giant panda is one of the most adored 1. If the writer were to delete the opening
animals in the world. Its furry white and black sentence of this paragraph (beginning the
body appears cuddly and soft, while the bears paragraph with A giant pandas furry body . .
round, white head and black eye patches are .), the paragraph would primarily lose:
instantly recognizable. Nevertheless, the giant
panda is a very rare creature and is protected by A) information that sets up a contrast that
law in its native China, where it lives in the immediately follows
bamboo forests and mountain slopes in the B) an irrelevant but humorous digression
central part of the country. At one time, giant C) information that complements the
pandas lived at lower altitudes, but farming and subsequent sentence
land development have pushed the animals high D) an important description of the setting
into the mountains. Despite conservation efforts
and the pandas popularity, scientists really
know very little about how wild pandas live,
since so few people have seen them in their
natural habitat.

Question 2 Refers to the Paragraph Below

All living things are made of cells. In fact, the 2. The writer is considering adding the following
cell is the basic unit of life and is the simplest true statement at the end of the paragraph:
unit capable of independent existence. Some one-
celled organisms can lead independent lives. In this, they are remarkably similar to their
Others live in loosely organized groups, distant, one-celled cousins.
depending on one another in order to function.
In more complex organisms like plants and Should the writer make this addition here?
animals, cells are no longer independent.
Instead, they become specialized, with strict A) Yes, because all living things are made up of
organization and specific jobs to perform. cells.
Certain cell types are grouped into tissues, and B) Yes, because it provides information
these tissues form organs. Other cell types stay necessary to the understanding of cells.
more or less discrete, becoming blood cells or C) No, because it will distract the reader from
antibodies, for example. Regardless of the main idea of the paragraph.
specialization, cells all work together to form the D) No, because it contradicts statements made
body of one living thing. previously in the paragraph.

76 76
PSAT Passage Analysis: Sequence

Passage Analysis: Sequence

Identifying Passage Analysis: Sequence Questions on the PSAT: The question will
ask which sequence of sentences within a paragraph, or which sequence of
paragraphs within a passage, is most logical. To deduce proper sequencing, look for a
topic sentence to use as Sentence 1 or look for connections between ideas to link
different sentences.

Question 1 Refers to the Paragraph Below

[1] While knocking down in native forests is 1. For the sake of logic and coherence, Sentence 4
still under debate, everyone agrees it is should be placed:
important for residences that border open land.
[2] In my neighborhood, homeowners are A) where it is now.
required to keep their property cleared of B) before Sentence 1.
anything that might serve as kindling during C) before Sentence 2.
fire season. [3] Even more, we are told to make D) before Sentence 3.
our houses as fireproof as possible. [4] This can
be accomplished by using special building
materials. [5] As some people have learned the
hard way, it takes just one spark to burn a house
to the ground.

Question 2 Refers to the Paragraph Below

[1] Although they can be found at such a 2. Which of the following sentence sequences will
variety of levels, synchronized swimming still make the paragraph more logical?
lags behind many other sports in the race for
acknowledgment and respect. [2] Perhaps if A) NO CHANGE
more people were aware, of the dedication and B) 1, 3, 2
talent required for success in the sport, it would C) 2, 3, 1
be taken more seriously. [3] Synchronized D) 3, 1, 2
swimming teams can be found at the
elementary, secondary, collegiate, and Olympic
levels

77 77
PSAT Passage Analysis Notes

78 78
PSAT Passage Analysis Review

Question 1 Refers to the Paragraph Below

[1] We looked at several more places until we 1. The writer is considering dividing this paragraph
found one that was not only economical, but also into two separate paragraphs. In terms of the logic
perfect for me. [2] It was on the third floor of a and coherence of the essay, the best course of action
stately brick home with a cozy studio apartment. to take would be to:
[3] The apartment had hardwood floors
throughout and a sizeable walk-in closet lined A) leave the paragraph as it is (NO CHANGE)
entirely in cedar. [4] The bathroom was B) begin a new paragraph with Sentence 3
equipped with a claw foot tub and pedestal sink, C) begin a new paragraph with Sentence 5
both of which were original to the house. [5] It D) begin a new paragraph with Sentence 7
didnt have a fireplace or ornate light fixtures,
but it did have a charming bay window. [6] I told
my parents that even though the apartment
didnt have everything I wanted, nothing in life
was perfect, so I should be happy with what I
had found. [7] Of course, they laughed, but I
could tell that they were also pleased by my
decision. [8] We gave the landlord the deposit
and started making plans to move me in the
very next weekend.

Question 2 Refers to the Paragraph Below

While Dr. Rices demeanor throughout her 2. Which choice best completes the description of Dr.
career has often been peace-promoting and Rices attitude toward her position as National
diplomacy-oriented, she has also taken strong Security Advisor?
stances in regard to the countrys ability and
responsibility to defend itself and its citizens. A) NO CHANGE
She earned a reputation as a decisive leader B) strength
during her tenure as National Security Advisor C) contributions
because of her work. D) tenacity and boldness

Question 3 Refers to the Paragraph Below

[1] Nymans most famous achievement was 3. The writer wants to add the following sentence to
the music for The Piano, winner of the 1993 the paragraph:
Cannes Film Festivals prestigious Palme dOr
award for best picture. [2] In the U.S., the film Despite this oversight, the soundtrack remains
was nominated for six Academy Awards and won among the best selling film music recordings of all
three. [3] Surprisingly, a nomination was not time.
granted to Nymans soundtrack for the Best
Score award. [4] Its grace is achieved through The best placement for the sentence is immediately:
skillful use of the piano to replace the female
leads voice, which is absent throughout the film. A) before sentence 1
B) after sentence 2.
C) after sentence 3.
D) after sentence 4.

79 79
PSAT Passage Analysis Review

Question 4 Refers to the Paragraphs Below

Not everyone who comes into contact with 4. Which of the following choice would provide the
toxic mold is so lucky. In the mid 1990s, cases of best transition between the paragraphs, guiding the
pulmonary hemorrhage (bleeding in the lungs) reader from the topic of the first paragraph to the
occurred in Ohio in infants. Scientists later new topic of the second paragraph?
found that all the homes of these babies had
high levels of stachybotrys chartarum. Not every A) My boss frequently took my advice about making
scientist believes that toxic mold was directly improvements in the office.
responsible for making the Ohio children sick. B) Armed with this new information, I approached
Still, many researchers point to this case and my boss about the subject of toxic mold.
others to link mold exposure to bleeding in the C) Although our office was not in Ohio, both my
lungs. boss and I were concerned about those children.
I wanted him to hire a professional crew to D) Toxic mold appeared to be very dangerous
clean up our basement; I was still curious, indeed.
however. How did this dangerous fungus get into
the building in the first place? Had it been there
all along? More research was needed. I
discovered that stachybotrys chartarum comes
from the soil and generally enters clean
buildings through floodwaters, dust, and dirt.
Moreover, the fungus is there from the
beginning, brought in on the materials when the
structure is built. Its usually only noticed after
water damage occurs, like in our basement. Now
I knew how the fungus got there and I was
keeping my distance!

Question 5 Refers to the Paragraph Below

While most of his fame is due to criticism he


received in the 18th century, he was actually
highly regarded in his lifetime. Still, despite his 5. Which introduction most effectively sets up the
reputation, very few detailed records exist about information that follows?
his life, and most of those refer to his career in
London. The little information we have suggests A) There is modern debate about the true
that Shakespeare grew up in somewhat scholarship of Shakespeares works.
prosperous conditions. His father was a glove B) William Shakespeare was born in 1564 in
maker and town official, while his mother came Stratford-upon-Avon, United Kingdom.
from an affluent family of farmers. There is C) William Shakespeare produced plays of varying
some indication that his family was Catholic, a quality during his lifetime.
crime in England at that time, though there is D) William Shakespeare has long been considered
no proof to support it. the greatest English playwright and poet of all
time.

80 80
PSAT Passage Analysis Answers

Workbook Answers Review Answers

Passage Analysis: Adding Precision


1) A

1) B 2) D

2) A 3) C

Passage Analysis: Introductions and 4) B


Conclusions
5) D

1) B

2) B

Passage Analysis: Main Idea Analysis

1) C

2) C

Passage Analysis: Sequence

1) A

2) D

81 81
PSAT Prepositions

PSAT Writing:
Prepositions

82 82
PSAT Prepositions

Prepositions

Prepositions are words that combine with a noun, noun phrase, or pronoun to show
its relationship to another word in the sentence.

Ten most common prepositions: of, in, to, for, with, on, at, from, by, about

Identifying Preposition Issues on the PSAT: Look for a noun paired with an
improper preposition. Use your grammar ear to help determine if the preposition is
correct.

1. Death Valley occupies approximately 3,000 square miles, is about 130 miles long,
and varies from about six to 14 miles wide. Most of the valley is within an extremely low
altitude.

A) NO CHANGE
B) by
C) at
D) for

2. Steve just returned from his trip to The Netherlands, a European country dotted with
windmills.

A) NO CHANGE
B) inside
C) for
D) on

3. During some thunderstorms, the light from lightning bolts can be seen as far as 100 miles
away, depending on the height of the bolt, the clarity of the air, and the elevation to the
viewer.

A) NO CHANGE
B) by
C) for
D) of

83 83
PSAT Prepositions Notes

84 84
PSAT Prepositions Review

1. Streaks of many shades of blue, green, and red run thinly and discretely from the base to the top of the
bottles. There are parallels among the action of glassblowing and, say, pulling taffy. And Murano glass does
look good enough to eat.

A) NO CHANGE
B) between
C) within
D) DELETE the underlined part

2. Some medical research shows that meat actually starts to rot while it is being digested. While this
process rarely does harm, vegetarians argue that it proves our long intestines are designed to ensure that
we can break down and absorb nutrients within fibrous plant material, not animal flesh.

A) NO CHANGE
B) that
C) by
D) from

3. The act of giving out bonuses reflects the value the company places on the work of its employees. This
idea was demonstrated by company executives, who took pride in personally congratulating each person for
meeting his or her annual goals.

A) NO CHANGE
B) by
C) while
D) as if

4. Most online auction companies are very concerned by the security of their systems. Ensuring that
neither the seller nor the buyer is going to be jeopardized in any way is an important feature of a good
online auctioneer.

A) NO CHANGE
B) about
C) for
D) into

5. I learned that tortoises are among the most endangered families in reptiles. That means having a tortoise
is a privilege, and Im proud that my family has entrusted me with Rosies care. By caring for Rosie Ill be
able to share something with the grandma I never knew.

A) NO CHANGE
B) families of
C) family in
D) family of

85 85
PSAT Prepositions Answers

Workbook Answers Review Answers

Prepositions
1) B

1) C 2) D

2) A 3) A

3) D 4) B

5) B

86 86
PSAT Pronouns

PSAT Writing:
Pronouns

87 87
PSAT Pronouns: Singular vs. Plural

Pronouns: Singular vs. Plural

Pronouns are words that take the place of nouns. Pronouns are used to reduce the amount
of repetition in a sentence.

A pronoun error on the SAT is to match a singular subject with a plural pronoun or a
plural subject with a singular pronoun.

Identifying Singular vs. Plural Pronoun Issues on the PSAT: Look for an underlined
pronoun in the sentence. To check for singular vs. plural agreement, find the noun
which the pronoun in replacing. Is the word singular or plural? Does the
pronoun match?

1. My mother said rationing was important because they helped guarantee supplies for the
thousands of military troops overseas.

A) NO CHANGE
B) it
C) she
D) one

2. Many basketball players have started wearing a headband during games. However, other
players believe they negatively impact their ability to run up and down the court.

A) NO CHANGE
B) it negatively impacts
C) theyre negatively impact
D) its negatively impacts

3. Powerful music, be it in the form of sixteenth-century operas or modern movie soundtracks,


affects listeners because they create distinctive and fresh impressions upon each new listening
experience.

A) NO CHANGE
B) by it having created
C) since they create
D) because it creates

88 88
PSAT Pronouns: Personal Pronouns

Pronouns: Personal Pronouns

Pronouns are words that take the place of nouns. Pronouns are used to reduce the amount
of repetition in a sentence.

A pronoun error on the SAT is to switch between the types of personal pronouns (1st,
2nd, and 3rd person pronouns).

Identifying Personal Pronoun Issues on the PSAT: Look for one or you in the
sentence. A sentence can use one (3rd personal pronoun) or you (2nd person pronoun), but
cannot switch between them.

1. If you want to store asparagus at home, one should wrap the stalk bottoms in a damp paper
towel, cover the towel in foil, and store the foil package in the refrigerator.

A) NO CHANGE
B) one should have wrapped
C) you should wrap
D) they should wrap

2. In the 21st century, you can go to the mall to pick up the latest compact disc, or you can
purchase the music online.

A) NO CHANGE
B) or one can
C) or it can
D) or they can

3. The philosopher Epictetus popularized the Stoic doctrine of limiting your desires, believing
that one should act in life as at a banquet by taking a polite portion of all that is offered.

A) NO CHANGE
B) having your desires limited
C) having ones desires limited
D) limiting ones desires

89 89
PSAT Pronouns: Possessive Pronouns

Pronouns: Possessive Pronouns

Pronouns are words that take the place of nouns. Pronouns are used to reduce the amount
of repetition in a sentence.

A pronoun error on the SAT is to replace a possessive noun with a non-


corresponding personal pronoun.

Identifying Possessive Pronoun Issues on the PSAT: Look for pronouns that indicate
possession in the sentence (my, her, your, their). Are they aligning with the subject
correctly?

We, our, and ourselves are only valid when the writer is part of the group being
discussed.

1. During the weekend of the wedding, my parents hosted many of our family members. They
definitely made themselves at home: eating all of our snacks and moving our clothes into the
available closet space.

A) NO CHANGE
B) his or her
C) their
D) ones

2. Most current hockey players do not wear face masks with their uniforms, so the athletes
may find the newly mandated attire to be a burden.

A) NO CHANGE
B) those
C) there
D) its

3. Modern medicine does not include traditional Chinese techniques in their procedures, so
those practices may seem unusual.

A) NO CHANGE
B) its
C) those
D) its

90 90
PSAT Pronouns: Pronoun Case

Pronouns: Pronoun Case

Pronouns are words that take the place of nouns. Pronouns are used to reduce the amount
of repetition in a sentence.

A pronoun error on the SAT is to have two pronouns, or a pronoun next to a noun, in
a sentence while misusing one, or both, pronouns.

Identifying Pronoun Case Issues on the PSAT: Look for two underlined
pronouns. To check usage, read the sentence twice and plug in one subject each time. Use
your grammar ear to identify errors.

1. Both John and Laura wanted to go to the game, so he and her were willing to pay the
exorbitant ticket price.

A) NO CHANGE
B) him and her were
C) he and she were
D) him and she were

2. Other African American musicians and athletes suffered under the same constraints. As a
result, Anderson traveled to Europe, where her and other American minorities were able to
perform and refine their skills more freely.

A) NO CHANGE
B) she
C) herself
D) they

3. He claims to have paid the contractor on time, but, between you and I, this is hardly the
first time he has gotten embroiled in a dispute.

A) NO CHANGE
B) you and we
C) you and me
D) yourselves

91 91
PSAT Pronouns: Unclear Pronoun Subject

Pronouns: Unclear Pronoun Subject

Pronouns are words that take the place of nouns. Pronouns are used to reduce the amount
of repetition in a sentence.

A pronoun error on the SAT is to a have a pronoun without a clear subject.

Identifying Unclear Pronouns Subject Issues on the PSAT: Look for an underlined
pronoun. Is the pronoun replacing a noun? Is there vagueness about which noun the
pronoun is replacing?

1. Panda cubs weigh just a few ounces when they are born, but they will eventually weigh up
to 300 pounds and stand five to six feet tall. The cub will stay with its mother for about three
years, after which they will be ready to mate again.

A) NO CHANGE
B) it
C) the mother
D) one

2. Sometimes in the afternoon, my grandmother and I would go shopping with my mother at


her favorite stores.

A) NO CHANGE
B) its
C) my grandmothers
D) mine

3. Among the most influential Murano glassmakers was Carlo Scarpa. Scarpa worked with
Murano glass from 1933 until 1947, when he turned his full attention to architecture.

A) NO CHANGE
B) some
C) it
D) this medium

92 92
PSAT Pronouns Notes

93 93
PSAT Pronouns Review

1. While some vegetarians have cultural or religious reasons for not eating meat, others cite animal cruelty
and better health as their reasons for the practice.

A) NO CHANGE
B) its
C) theyre
D) such

2. Garveys radical idea was that a society benefits most from people acting in their own self-interest, not in
the interest of it as a whole.

A) NO CHANGE
B) them
C) himself
D) the community

3. Last summer, me and my mother spent a month at my grandmothers house.

A) NO CHANGE
B) my mother and me
C) my mother and I
D) myself and my mother

4. Margo received a message from the bookstore that the first edition she had ordered had been located, but
she would need to come to the store to examine its condition.

A) NO CHANGE
B) so its condition can be examined
C) for the examination of its condition
D) to examine the books condition

5. Death Valley is located in the east-central part of California, and is part of the larger Mojave Desert. It
was named by miners who were heading west in the 1849 California gold rush. Many miners died trying to
cross it.

A) NO CHANGE
B) the valley
C) over
D) DELETE the underlined portion

94 94
PSAT Pronouns Answers

Workbook Answers Review Answers

Pronouns: Singular vs. Plural


1) A

1) B 2) D

2) B 3) C

3) D 4) D

Pronouns: Personal Pronouns 5) B

1) C

2) A

3) D

Pronouns: Possessive Pronouns

1) C

2) A

3) B

Pronouns: Pronoun Case

1) C

2) B

3) C

Unclear Pronoun Subject

1) C

2) C

3) A

95 95
PSAT Punctuation

PSAT Writing:
Punctuation

96 96
PSAT Punctuation: Comma Usage

Punctuation: Comma Usage

A PSAT error is to use commas in a way that does not align with their grammatical
function.

Identifying Comma Usage Issues on the PSAT: If a comma is located in the sentence or
the answer choices, check to see if it is performing one of the four functions of commas:
separating items in a list; pairing with a conjunction to join two complete
sentences; separating an introductory thought; or separating a descriptive
phrase.

1. Principals, who have a large role in creating a positive school culture also need to be adept
in many other areas: driving high test scores, interacting with parents, and summoning
potential donors.

A) NO CHANGE
B) Principals who have a large role in creating a positive school culture
C) Principals who have a large role in creating a positive school culture,
D) Principals, who have a large role in creating a positive school culture,

2. In 1863, Abraham Lincoln traveled, to Gettysburg, Pennsylvania and delivered a speech


honoring those who had died in battle. These 272 words are now some of the most famous in
American history.

A) NO CHANGE
B) traveled;
C) traveled
D) traveled

3. A decrease in newspaper subscriptions has led to a decrease in revenues for publishers,


which in turn has caused publishers to turn to more desperate actions in the attempt to boost
readership. Some newspapers have resorted to printing more salacious content; when there is
a risk of alienating long-time readers this is a risky proposition.

A) NO CHANGE
B) content, this is a risky proposition when there is the risk of alienating long-time readers.
C) content, a risky proposition when there is a risk of alienating long-time readers.
D) content; a risky proposition when there is a risk of alienating long-time readers.

97 97
PSAT Punctuation: Colon Usage

Punctuation: Colon Usage

A PSAT error is to use colons in a way that does not align with their grammatical function.

Identifying Colon Usage Issues on the PSAT: If a colon is located in the sentence or
the answer choices, check to see if it is performing one of the two functions of commas:
introducing a list or joining a phrase with additional details about that phrase.

1. During a speech at West Point in 1962, General Douglas MacArthur discussed the three
things he believed all graduates should hold most sacred, duty, honor, and country.

A) NO CHANGE
B) sacred; duty, honor, and country.
C) sacred: duty, honor and country.
D) sacred: duty, honor, and country.

2. Chris had an idea for an invention that would change the world: a watch that could also be
used to answer phone calls.

A) NO CHANGE
B) world. A watch
C) world; a watch
D) world, a watch

3. Once the ball is tipped, the jovial attitude of the players dissipates, cheap shots and other
underhanded tactics are common occurrences.

A) NO CHANGE
B) dissipates: because
C) dissipates:
D) dissipates, thats when

98 98
PSAT Punctuation: Contractions

Punctuation: Contractions

A PSAT error is to use apostrophes to form contractions whose constituent words do not fit
within the sentence.

Identifying Contraction Issues on the PSAT: If a contraction is located in the sentence


or the answer choices, substitute both of the words in the contraction into the
sentence and then check the sentence using your grammar ear.

1. The company decided to stake its future on the belief that people would pay for more for
songs with higher quality audio.

A) NO CHANGE
B) their
C) its
D) its

2. Although there is debate about the best way to stretch before exercise, most people agree
about its importance for avoiding injury.

A) NO CHANGE
B) its
C) theyre
D) its

3. They were able to build theyre own theater, The Globe, which gave them some
independence.

A) NO CHANGE
B) there
C) their
D) his

99 99
PSAT Punctuation: Dash Usage

Punctuation: Dash Usage

A PSAT error is to use dashes in a way that does not align with their grammatical function.

Identifying Dash Usage Issues on the PSAT: If a dash is located in the sentence or the
answer choices, check to see if it is performing one of the three functions of dashes:
separating a descriptive phrase, introducing a change in tone, or presenting an
explanation.

1. Philanthropists who support the arts like to point out that exposure to art, even to works
that are not highly regarded improves academic outcomes for students of all ages.

A) NO CHANGE
B) art even
C) art. Even
D) art: even

2. Bacteria play an important role in sewage reduction by consuming human waste product.
Data has shown that without bacteria's involvement, including consuming various other
materials during the water treatment processrivers and oceans would be much more
polluted.

A) NO CHANGE
B) process: this is
C) process,
D) process;

3. In 2004, the University of Notre Dame opened the DeBartolo Performing Arts Center. This
building includes an organ hall that hosts regular performances of sacred music
compositions that are played during religious ceremonies.

A) NO CHANGE
B) music. Compositions
C) music, compositions
D) music; compositions

100 100
PSAT Punctuation: Possession

Punctuation: Possession

A PSAT error is to use apostrophes in a way that do not correctly indicate possession.

Identifying Possession Issues on the PSAT: Look for apostrophes in the sentence or in
the answers choices. Possession errors on the PSAT include the text missing an
apostrophe when there is possession, having an apostrophe when there is no
possession, or placing an apostrophe in the incorrect location when indicating
possession.

1. Stephanies hope of getting home before lunch was quashed as her childrens recital
stretched into its fourth hour .

A) NO CHANGE
B) childrenss
C) childrens
D) childrens

2. During the 1970s and 1980s, Fleetwood Mac was one of the biggest musical acts in the
world. However, the groups internal conflicts led to a breakup of the band in 1995.

A) NO CHANGE
B) groupss
C) groups
D) groups

3. Say, for example, a significant hurricane is approaching shore. Citizens need to be


evacuated from the city in a prompt and orderly manner. The disaster managers job might
involve organizing contraflow on the roadways or authorizing broadcasts of the emergency
broadcast system.

A) NO CHANGE
B) managers job
C) managers job,
D) managers job,

101 101
PSAT Punctuation: Run-on Sentences and Sentence Fragments

Punctuation: Run-on Sentences and Sentence Fragments

An SAT error is to either use incorrect punctuation when joining two complete sentences or
to use incorrect punctuation when joining two sentence fragments.

Identifying Run-on Sentence Issues on the PSAT: Look for two complete thoughts
joined by some punctuation. Two complete sentences can only be joined using a period, a
semi-colon, or a comma joined with a conjunction.

1. James wanted to take advantage of the new technology to bring education to the masses.
His first project creating digital workbooks sold barely well enough to cover costs. James
then tried again with a technology application for smartphones; this idea is what helped him
to strike it rich.

A) NO CHANGE
B) smartphone, this
C) smartphones, this
D) DELETE the underlined portion.

2. The spirit of American adventurism was at its peak in the early 19th century, many people
packed up their belongings and headed West in search of opportunity.

A) NO CHANGE
B) century; many
C) century, but many
D) century, while

3. Vinyl record aficionados try to keep the medium alive through a scattered; but vibrant
community of listeners.

A) NO CHANGE
B) scattered but vibrant
C) scattered, but vibrant
D) scatted but vibrant,

102 102
PSAT Punctuation Notes

103 103
PSAT Punctuation Review

1. Conditions like sleep apnea are more common in men, people who snore, overweight people, and the
elderly. A person might have a sleep-related breathing disorder if wheezing; grunts; snorts; or other
abnormal sounds interrupt his or her breathing.

A) NO CHANGE
B) wheezing; grunts snorts; or
C) wheezing, grunts, snorts, or
D) wheezing grunts, snorts or

2. Time Dollars began in 1980 as a means of encouraging and rewarding community involvement. Time
Dollars exchange programs recognize that each person is valuable and has something, meaningful, to
contribute.

A) NO CHANGE
B) something meaningful
C) something, meaningful
D) something meaningful

3. A portmanteau is a linguistic blend that merges parts of two words into a single phrase. An example of a
portmanteau is smog, which combines two parts, smoke and fog.

A) NO CHANGE
B) parts: smoke and fog.
C) parts smoke and fog.
D) parts; smoke and fog.

4. To this day, the Saturday Evening Gazette includes horoscopes based on astrological calendars. The
newspapers editor, Karen Finsilver, says, The information is essential to readers who follow their
astrological signs.

A) NO CHANGE
B) newspaperss
C) newspapers
D) newspapers

5. Adults who pursue vocations as teachers have a wide array of reasons. These motivations can include a
love of working with children or a passion for a particular academic subject.

A) NO CHANGE
B) Adults, who pursue vocations as teachers,
C) Adults who pursue vocations, as teachers,
D) Adults who pursue vocations as teachers,

6. Mr. Holmes claims to have a prairie eye, he particularly enjoys the magnitude and gracefulness of open
grasslands.

A) NO CHANGE
B) eye, thus,
C) eye he
D) eye. He

104 104
PSAT Punctuation Answers

Workbook Answers Workbook Answers

Punctuation: Comma Usage Punctuation: Run-on Sentences

1) D 1) A

2) D 2) B

3) C 3) B

Punctuation: Colon Usage


Review Answers
1) D
1) C
2) A
2) D
3) C
3) B
Punctuation: Contractions
4) C

1) C 5) A

2) A 6) D

3) C

Punctuation: Dash Usage

1) B

2) C

3) A

Punctuation: Possession

1) A

2) C

3) B

105 105
PSAT Transitions (Conjunctions)

PSAT Writing:
Transitions
(Conjunctions)

106 106
PSAT Transitions (Conjunctions)

Transitions (Conjunctions)

A transition is a word or phrase that connects or joins together words, phrases,


clauses, or sentences.

Conjunctions are the most common type of transition. Conjunctions can indicate agreement
(cause and effect), disagreement (contrast), or the order in which events took place.

Identifying Transition Issues on the PSAT: Look for an agreeing conjunction joining
two disagreeing words, phrases, clauses, or sentences, or a disagreeing conjunction joining
two agreeing words, phrases, clauses, or sentences.

1. Due to the lucrative nature of the agricultural industry, notwithstanding the significance of
his invention, he struggled to make a profit from it.

A) NO CHANGE
B) Despite
C) As a result of
D) Although

2. Books about recent catastrophes may seem like morally bankrupt attempts to profit from
the misfortune of others, so they may also inform and educate, thus legitimizing them in some
readers eyes.

A) NO CHANGE
B) and they
C) moreover they
D) but they

3. Some ancient cultures held the belief that the gods controlled the success of the yearly
harvest. These societies thus placed a high priority on appeasing the wishes of their deities.
Given that, it is not surprising that within these cultures animal sacrifices and other religious
ceremonies were common during the planting season.

A) NO CHANGE
B) However,
C) By contrast,
D) Thereafter,

107 107
PSAT Transitions (Conjunctions) Notes

108 108
PSAT Transitions (Conjunctions) Review

1. Because he often blared music while doing his homework, the tumultuous atmosphere in the caf posed
no impediment to Tylers studying.

A) NO CHANGE
B) Although
C) Yet
D) Despite the fact that

2. They continued to berate the clumsy worker who had broken the machine, even though their anger had
abated somewhat.

A) NO CHANGE
B) because
C) since
D) in fact

3. The instructor of the course scheduled weekly quizzes, ostensibly to encourage the students to pay
attention, so the class believed the real rationale was to torment them.

A) NO CHANGE
B) attention, yet
C) attention, and
D) attention, because

4. Although history is supposed to instruct, some archaeologists believe that their research is rife with
evidence that mankind has sometimes repeated the same mistakes by those living in earlier eras.

A) NO CHANGE
B) Because history
C) Since history
D) Is it true that

5. During their family reunion in Vermont, the Johnsons met a distant cousin who was the most
unemotional individual they had ever known, yet when he wept as they boarded the airplane to depart, they
were surprised.

A) NO CHANGE
B) so
C) though
D) hence

109 109
PSAT Transitions (Conjunctions) Answers

Workbook Answers Review Answers

Transitions (Conjunctions)
1) A

1) B 2) A

2) D 3) B

3) A 4) A

5) B

110 110
PSAT Verbs

PSAT Writing:
Verbs

111 111
PSAT Verbs: Subject-Verb Agreement

Verbs: Subject-Verb Agreement

Verbs are words that convey actions, occurrences, or states of being.

In proper grammar, singular subjects are paired with singular verbs and plural subjects are
paired with plural verbs.

Identifying Subject-Verb Agreement Issues on the PSAT: Look for an underlined


verb in the sentence. To check for subject and verb agreement, place the verb and
its subject directly next to each other, read together, and check using your
grammar ear.

1. Since the economic slow-down, retail sales has been plummeting across the country.

A) NO CHANGE
B) plummets
C) have been plummeting
D) has plummeted

2. Legal regulations aimed at protecting the innocent requires districts to provide lawyers for
every accused criminal.

A) NO CHANGE
B) require districts to provide
C) require districts providing
D) that requires districts to provide

3. The grim realities of street life belies the traditional romantic notions of adventure found in
the novels of Mark Twain or Charles Dickens.

A) NO CHANGE
B) are belied by
C) belie
D) have been belied by

112 112
PSAT Verbs: Tense

Verbs: Tense

Verbs are words that convey actions, occurrences, or states of being.

In proper grammar, verb tense must align with the tense of the surrounding text.

Identifying Verb Tense Issues on the PSAT: Look for an underlined verb in the
sentence. When checking tense alignment, look for descriptive text (ending in
1994) and the tense of surrounding verbs to indicate the tense of the sentence.

1. Isaac Newton may have discovered gravity when he saw an apple fall from a tree, but he
did not really know what he had discovered; gravity remains a mystery to him until his death.

A) NO CHANGE
B) has remained
C) remained
D) remaining

2. The documentary of the 2010 championship shows the final, tense moments of the game.
The rabid fans cheer as the captain walked to the free throw line to attempt the winning
shots.

A) NO CHANGE
B) had walked
C) would walk
D) walks

3. Edward Abbeys essays earned him praise as a leading American environmentalist during
the late 1960s, a period during which he wrote prolifically.

A) NO CHANGE
B) earn him praise
C) have earned him praise
D) would earn him praise

113 113
PSAT Verbs Notes

114 114
PSAT Verbs Review

1. The minutes the average American child spend playing outside have decreased dramatically over the last
decade.

A) NO CHANGE
B) has spent
C) spends
D) are spent

2. The enigmatic smile that graces the face of the woman portrayed in Leonardo da Vincis Mona Lisa has
been attributed to her knowing a mysterious secret.

A) NO CHANGE
B) is being attributed
C) have been attributed
D) has had its attribution

3. When the committee revised the judging standards in 2004, it believed it had created a set of guidelines
that will ensure fairness at the Olympic Games.

A) NO CHANGE
B) would have ensured
C) had ensured
D) would ensure

4. The particles in Dr. Jones experimental sample appear to have changed in an unusual way.

A) NO CHANGE
B) appears
C) have appeared
D) DELETE the underlined portion

5. The success of George Washingtons program for reforming weights and


measures, Andro Linklater and Tom Smith argue, were essential to the eventual emergence of a consumer
economy.

A) NO CHANGE
B) was
C) have been
D) being

115 115
PSAT Verbs Answers

Workbook Answers Review Answers

Verbs: Subject-Verb Agreement


1) C

1) C 2) A

2) B 3) D

3) C 4) A

Verbs: Tense 5) B

1) C

2) D

3) A

116 116
PSAT Word Choice

PSAT Writing:
Word Choice

117 117
PSAT Word Choice: Frequently Confused Words

Word Choice: Frequently Confused Words

An error on the SAT is to use a word that sounds similar to a word that works in the
context of the sentence, but has a different meaning.

Identifying Frequently Confused Words Issues on the PSAT: Look for words that
have homophones which may confuse your grammar ear.

Examples:

Affect vs. Effect


Accept vs. Except
Farther vs. Further

1. From there, we would take the train to my mothers house, spend the night, than return
home the following day.

A) NO CHANGE
B) then return
C) than returning
D) then be returned

2. The government suggested to tribal leadership that it would be in its best interest to own
real property and pay taxes on it like all property owners. The Native American leadership did
not accept the offers of land ownership. They believed that it would cause reservation territory
to recede.

A) NO CHANGE
B) except
C) with the exception of
D) have an acceptance of

3. One significant affect of the laws passage was that traffic accidents decreased by 20% when
compared to the year prior.

A) NO CHANGE
B) A significant affect
C) One effect significantly
D) One significant effect

118 118
PSAT Word Choice: Inconsistent Tone and/or Style

Word Choice: Inconsistent Tone and/or Style

An error on the SAT is to use diction/tone/style that is inconsistent with the


surrounding text.

Identifying Inconsistent Tone and/or Style Issues on the PSAT: Inconsistencies can
include the use of vague language, overly informal language, colloquialisms, or jargon
(vocabulary of a particular group).

1. As a general rule, temperature increases when altitude decreases. Thus, the low altitude of
deserts is one of the reasons for their ridiculously scorching temperatures.

A) NO CHANGE
B) steel melting temperatures.
C) insanely asinine temperatures.
D) extremely hot temperatures.

2. A big reason for the passage of the bill was the fear that the invasive species would soon
overrun the wetland.

A) NO CHANGE
B) main thing causing
C) huge thing about
D) primary cause of

3. Following the American Revolution, George Washington resigned his commission and
returned home to Virginia. During this time he assiduously superintended the plantation in
the hope of developing goods for sale.

A) NO CHANGE
B) oversaw
C) fastidiously directly
D) diligently administrated

119 119
PSAT Word Choice Notes

120 120
PSAT Word Choice Review

1. Dunbars breakthrough came at the age of 20, when he gave his first public reading before the Western
Association of Writers. The praise he gathered attracted the attention of other writers, whom began to take an
interest in his career.

A) NO CHANGE
B) writers, that
C) writers, who
D) writers

2. Most people had little interest in the merger of the two companies, but for the employees of those
organizations it was a pretty big deal.

A) NO CHANGE
B) could not be put on the backburner.
C) was nothing to shake their tails at.
D) was an important occurrence.

3. When politicians are in need of extra exposure for example, near election season one guaranteed way
to gain attention is to get on their soapboxes and talk directly to the public.

A) NO CHANGE
B) invest some sweat equity
C) give a speech
D) DELETE the underlined portion

4. Even though movies usually do not effect Jeremy, for some reason he could not stop crying when he left
the theater.

A) NO CHANGE
B) have an effect on
C) affect
D) create an affect for

5. Jim wanted Tom to go farther into the distance: He had just purchased some new Frisbees and he wanted
to give them a good throw.

A) NO CHANGE
B) farther by
C) further into
D) further along

121 121
PSAT Word Choice Answers

Workbook Answers Review Answers

Word Choice: Frequently Confused


Words 1) C

2) D
1) B
3) C
2) A
4) C
3) D
5) A
Word Choice: Inconsistent Tone and/or
Style

1) D

2) D

3) B

122 122
PSAT Wordiness and Redundancy

PSAT Writing:
Wordiness and
Redundancy

123 123
PSAT Wordiness and Redundancy: Wordiness

Wordiness and Redundancy: Wordiness

Wordiness occurs when an underlined phrase uses more words than it needs.

Identifying Wordiness on the PSAT: Look for the use of passive voice, the presence of
several two or three letter words, or the inclusion of an unnecessary ing verb.

On the PSAT, if you are stuck between two or more choices, pick the answer that has
the fewest number of words.

1. The Statute of Frauds was enacted in 1679 and required that all contracts for the purchase
and sale of any land be in writing and signed by the party to be charged in order to be
enforceable. With some modifications by someone making changes, the same rule is in place
today.

A) NO CHANGE
B) changes made by modifying,
C) modifications,
D) DELETE the underlined portion.

2. Garvey was born in Jamaica in 1887 and went to be in college in London just before World
War I. During the war, he moved to the United States to begin working on his dream of
political unity for all people with African ancestry.

A) NO CHANGE
B) would have been in
C) attended
D) went to attend

3. Yoga has ardent practitioners. These people find that the strength and mental clarity
provided by yoga cannot be replicated by any other exercises.

A) NO CHANGE
B) practitioners, who will go to the studio at least once a day.
C) practitioners, who fully believe in its benefits.
D) practitionerslovers of the exercise.

124 124
PSAT Wordiness and Redundancy: Redundancy

Wordiness and Redundancy: Redundancy

Redundancy occurs when an underlined word or phrase repeats an idea that is


already in the sentence.

Identifying Redundancy on the PSAT: Look for common redundancies.

Examples:

6:30 p.m. at night


And also
Close proximity
Manually by hand
Repeat again

1. The next day, we walked out to one of the spectacular view sites overlooking the Grand
Canyon. While my husband took more photos of the sprawling vista, I again looked down at
my feet. This time, there was no confusion or doubt.

A) NO CHANGE
B) doubt by which to be confused
C) confusing doubt
D) DELETE the underlined portion.

2. Last year, Samantha Johnson, alumna of Dedham High School, donated twelve million
dollars to the school for the construction of a new gymnasium and athletic fields. Because of
the generosity of Johnsons contribution to the school, Dedham students will have access to
world class facilities for years to come.

A) NO CHANGE
B) the generous donation of so much money to the school,
C) the substantial donation of resources,
D) Johnsons generosity,

3. The government suggested to tribal leadership that it would be in the tribes best interest to
own real property and pay taxes on it like all property owners. Legally, the government owned
tribal lands but offered to donate it to the tribe without charge.

A) NO CHANGE
B) free of charge
C) without charging them
D) DELETE the underlined portion

125 125
PSAT Wordiness and Redundancy Notes

126 126
PSAT Wordiness and Redundancy Review

1. While reflux can often usually be controlled during the daytime by swallowing and sitting in an upright
position, during the night, people suffering from acid reflux do not swallow as frequently, and their reclined
position causes more reflux to occur.

A) NO CHANGE
B) typically
C) quite possibly
D) DELETE the underlined portion.

2. Ever since switching their business from selling lumber to selling mechanical equipment, Jim and Steve
have lost a yearly sum of $1.2 million annually due to the fact they have entered a more competitive market.

A) NO CHANGE
B) experienced an annual loss of $1.2 million each year
C) lost $1.2 million annually
D) have a yearly loss of $1.2 billion annually

3. The Statue of Liberty was brought to America in 1886. France gave the statue to the United States as a
sign of friendship, and it was dedicated on October 28. French artist Frederic Bartholdi sculpted the statue;
engineer Maurice Koechlin, the man who designed the Eiffel Tower, created the statues internal structure.

A) NO CHANGE
B) designer of the Eiffel Tower,
C) who was also a designer of the Eiffel Tower
D) who was the man that designed the Eiffel Tower

4. Dr. Condoleezza Rice has lived a life of great accomplishment and achievement. She was an avid scholar,
graduating from high school at the age of 15. At only 19 a very young age she graduated from the
University of Denver with a degree in political science.

A) NO CHANGE
B) which is quite young
C) not yet in her twenties
D) DELETE the underlined portion.

5. John was not accustomed to waking up at 5:00 a.m. in the morning, but taking his first trip to Europe
was worth the early rise.

A) NO CHANGE
B) 5:00 a.m.
C) 5:00 a.m. during the mornings
D) DELETE the underlined portion.

127 127
PSAT Wordiness and Redundancy Answers

Workbook Answers Review Answers

Wordiness and Redundancy:


Wordiness 1) D

2) C
1) C
3) B
2) C
4) D
3) A
5) B
Wordiness and Redundancy:
Redundancy

1) D

2) D

3) D

128 128
PSAT Advanced Grammar

PSAT Writing:
Advanced Grammar

129 129
PSAT Advanced Grammar: Affect vs. Effect

Advanced Grammar: Affect vs. Effect

An error on the PSAT is to misuse the words affect and effect

Identifying Affect vs. Effect Issues on the PSAT: Looks for the word affect or effect
underlined in a sentence.

Affect means to produce a change or to act in a way you do not feel (used as a verb).
The weather affected Jeremys plans.

Effect means a result (used as a noun).


The weather had no effect on Jeremys plans.

1. One of the primary causes for the financial crisis of 2008 was the effect of banks granting
loans to people who could not pay them back.

A) NO CHANGE
B) the affect of
C) an effect of
D) an affect of

2. Tom had no idea of the immediately positive affect that would result from switching the
part in his hair.

A) NO CHANGE
B) immediately positive effect
C) immediate positive effect
D) immediate positive affect

3. The presidential election had a major affect on the countrys policies.

A) NO CHANGE
B) effect on
C) affecting on
D) effect

130 130
PSAT Advanced Grammar: Logical Comparisons

Advanced Grammar: Logical Comparisons

An error on the PSAT is to set up a comparison (words linked by like, than, or as)
where the items being compared are not of the same type.

Identifying Logical Comparison Issues on the PSAT: Look for a comparison being
made in the sentence. Are the items of the same type?

1. As an avid mystery reader, Patrick felt that novels by John Grisham were more enjoyable
than other authors.

A) NO CHANGE
B) enjoyable, more so than other authors.
C) more enjoyable than books by other authors.
D) more enjoyable, particularly when compared to other authors.

2. The average cost of a students yearly education in private school is several times a student
in public school.

A) NO CHANGE
B) what it is for a student being in public school
C) that of a students yearly education in public school
D) like for a student in public school

3. Begun during the Great Depression, Womans Day magazine had an effect on the American
family that was greater than any other magazine.

A) NO CHANGE
B) as greater as any other magazine
C) greater than those of any other magazine
D) greater than that of any other magazine

131 131
PSAT Advanced Grammar: Misplaced Modifiers

Advanced Grammar: Misplaced Modifiers

An error on the PSAT is to have a descriptive word or phrase that is NOT next to
what it is describing.

Identifying Misplaced Modifier Issues on the PSAT: Look for a descriptive phrase
that is next to a comma. What is being described (the subject of the sentence) must come
directly after the comma.

1. First sighted in the 17th century, the moons of Jupiter contradicted much of the traditional
thinking about Earths place in the cosmos.

A) NO CHANGE
B) In the 17th century,
C) Galileo first sighted them in the 17th century,
D) They were first sighted in the 17th century,

2. Born one hundred years ago in St. Petersburg, the peaceful childhood of choreographer
George Balanchine was disrupted by World War I.

A) NO CHANGE
B) choreographer George Balanchine had his peaceful childhood
C) World War I disrupted the peaceful childhood of choreographer George Balanchine
D) choreographer George Balanchines peaceful childhood

3. Thumbing through Sonnets from the Portuguese, it wasnt difficult to find more than one
poem that spoke to Richard.

A) NO CHANGE
B) Richard realized it wasnt difficult to find more than one poem that spoke to him
C) Richard realized that more than one poem spoke to him without difficulty
D) more than one poem spoke to Richard, which wasnt difficult to realize

132 132
PSAT Advanced Grammar: Who, Whom, That, Which, and Whose

Advanced Grammar: Who, Whom, That, Which, and Whose

An error on the PSAT is to misuse the words who, whom, that, which, and whose.

Identifying Who, Whom, That, Which, and Whose Issues on the PSAT: Look for the
word(s) who, whom, that, which, or whose in a sentence. Use the following rules to identify
correct usage.

Who and Whom are used with sentence subjects or objects that are a person.

Who is used with a sentence subject (We all know who was responsible for the
prank).
Whom is used with a sentence object (You gave the card to whom?).

That and which are used with sentence subjects or objects that are not a person.

That is used to introduce a clause that is essential to the sentence.


Which is used to introduce a clause that is not essential to the sentence.

Whose is used to indicate possession (Whose jacket is this?).

1. Moreover, the chemical has been severely criticized by people that believe it is having a
negative impact on the environment.

A) NO CHANGE
B) who
C) whom
D) which

2. Benches, that are found at many bus stops, are often uncomfortable to sit on.

A) NO CHANGE
B) which
C) who
D) of that

3. Emily Dickinson, one of Americas great nineteenth-century poets, was a prolific letter
writer. Although her physical contact with the world was limited by caring for her invalid
mother and by her own poor health, whose correspondence was extensive

A) NO CHANGE
B) their
C) Dickinsons
D) whos

133 133
PSAT Advanced Grammar Notes

134 134
PSAT Advanced Grammar Review

1. Brightly lit stations welcomed the public, many of them were skeptical of traveling underground. It didnt
take long for New Yorkers to adapt, however.

A) NO CHANGE
B) of whom
C) of who
D) DELETE the underlined portion

2. In 1998, dust from the Gobi Desert crossed the Pacific Ocean and sent dust-pollution levels on the West
Coast much higher than the East Coast.

A) NO CHANGE
B) higher than those on
C) more high than what they were on
D) as higher than on

3. With great enthusiasm, they added on a year every New Years Day. By contrast American society has
often been described as one that values the vibrant energy of youth over the wisdom and experience gained
with age.

A) NO CHANGE
B) whose
C) this
D) whom

4. The vowel sounds in English are more numerous than Spanish.

A) NO CHANGE
B) numerous in Spanish
C) numerous than that in Spanish
D) numerous than those in Spanish

5. Studying for a test can affect a students final grades.

A) NO CHANGE
B) effect
C) have the effect on
D) be affecting on

6. Although criticized by a few for his daredevil stunts, most people viewed Evel Knievel as a skillful
motorcyclist.
A) NO CHANGE
B) Evel Knievel was viewed by most people as a skillful motorcyclist
C) most people viewed Evel Knievel to be a skillful motorcyclist
D) Evel Knievel, a skillful motorcyclist in the view of most people

135 135
PSAT Advanced Grammar Answers

Workbook Answers Advanced Grammar: Who, Whom, That,


Which, and Whose
Advanced Grammar: Affect vs. Effect
1) B

1) A 2) B

2) B 3) C

3) B
Review Answers

Advanced Grammar: Logical


Comparisons 1) B

2) B
1) C
3) A
2) C
4) D
3) D
5) A
Advanced Grammar: Misplaced
Modifiers 6) B

1) A

2) B

3) B

Advanced Grammar: Parallelism

1) C

2) C

3) D

136 136
PSAT Math

PSAT Math:
Table of Contents

137 137
PSAT Math: Table of Contents

Math: Table of Contents

PSAT Math: Introduction (Page 142)

Arithmetic and Algebra

Foundations of Arithmetic and Algebra (Page 144)


Absolute Value
Exponents
Percentage
Terms and Operations

Average, Median, and Mode (Page 152)


Average
Median and Mode

Functions (Page 159)


f(x)
f(x a)
Function Graphs
Function Tables

Quadratics and Polynomials (Page 167)


Higher-Order Equations
Quadratics
Polynomial Division
Polynomial Factors
Solving Polynomials: Completing the Square
Solving Polynomials: Quadratic Formula

Ratio, Proportion, and Unit Conversion (Page 176)


Probability
Ratio and Proportional Relationships
Ratio and Proportional Relationships (Colons)
Unit Conversions

Solving Linear Equations (Page 184)


Distance and Midpoint Formulas
Slope
Slope-Intercept Form

Solving Non-Linear Equations (Page 190)


Rewriting Equations
Single Equations Questions (Non-Quadratic)
Systems of Equations

Translating Word Problems (Page 197)


Linear Equations
Non-Linear Equations
Systems of Equations

138 138
PSAT Math: Table of Contents

Math: Table of Contents

Geometry

Circles (Page 205)


Arc and Sector
Circumference and Area
Chords
Inscribed Circles

Geometry in Three Dimensions (Page 213)


Cones
Cylinders
Rectangular Prisms
Spheres
Triangular Prisms
Triangular Pyramids

Graphing (Page 220)


Circles
Parabolas

Lines and Angles (Page 226)


Intersecting Lines
Rule of 180

Quadrilaterals and Polygons (Page 232)


Polygons
Rhombuses
Trapezoids

Triangles (Page 239)


Area
Pythagorean Theorem
Rule of 180
Similar Triangles
Special Right Triangles

139 139
PSAT Math: Table of Contents

Math: Table of Contents

Additional Topics

Advanced Topics (Page 248)


Complex Numbers
Compound Interest
Radians
Trigonometry: Sine, Cosine, and Tangent
Unit Circle

Interpreting Equations (Page 257)


Interpreting Linear Equations
Interpreting Non-Linear Equation

Plug and Chug & Backsolving (Page 263)


Backsolving
Plug and Chug

Statistics (Page 269)


Exponential Growth
Interpreting Charts and Graphs
Line of Best Fit
Margin of Error

140 140
PSAT Math

PSAT Math:
Introduction

141 141
PSAT Math: Introduction

PSAT Math Introduction

70 Minutes for 48 Questions

Test 3: Math (No Calculator)


25 minutes for 17 questions
Multiple choice questions and student-produced responses

Test 4: Math (Calculator)


45 minutes for 31 questions
Multiple choice questions and student-produced responses

Concepts covered on PSAT Math:

Algebra
o Formulas, expressions, and equations
Data analysis
o Ratios, proportions, and percentages
Advanced math
o Quadratics and polynomials
Additional topics
o Geometry and trigonometry

Quick Math Tips


Stay confident you know all the math you need to know.
Identify the concept that is being tested.
Dont make a problem harder than it is.
Dont make stupid mistakes stay focused.
Answer the question that is being asked (variables/units/etc.).

142 142
PSAT Foundations of Arithmetic and Algebra

PSAT Math:
Foundations of
Arithmetic and Algebra

143 143
PSAT Foundations of Arithmetic and Algebra: Terms and Operations

Factor vs. Multiple


Term Definition
Whole Number A number with no factional or decimal part.
Integer Any whole number (400, 0, 2, 1000).
Factor An integer that divides evenly into another integer.
Multiple An integer that can be divided by a smaller integer with no remainder.

Prime vs. Composite


Term Definition
Prime A number with only 1 and itself as factors. 1 is NOT a prime number.
Composite A number with factors other than 1 and itself. 1 is NOT a composite number.

Even vs. Odd


Term Definition
Even An integer which divided by 2 yields a whole number (0 is an even number).
Odd An integer which divided by 2 does not yield a whole number.

1. Which of the following is a multiple of 90 ?

A) 1,120
B) 1,530
C) 1,720
D) 2,230

2. If x is an even integer, then which of the following could be the value of (x + 1)(x + 2) ?

A) 20
B) 30
C) 40
D) 50

3. If a is the greatest prime factor of 34 and b is the greatest prime factor of 96, what is the
value of a + b?

144 144
PSAT Foundations of Arithmetic and Algebra: Exponents

Term Definition Example

Positive powers Multiply the base by itself as many times as the exponent tells you to. 35 =

Put a one over the base and multiple the base by as many times as the
Negative powers 35 =
exponent tells you to.

0 as a power Any number raised to the power of zero equals 1. 30 =

1 as a power Any number raised to the power of 1 equals itself 31 =

1
Powers on fractions Multiply the fraction by itself as many times as the exponent tell you to. ( 3 )3 =

Powers on negative It a negative number is raised to an even power, it becomes positive, if it is


(3)2 =
numbers raised to an odd power, it stays negative.

Multiplying powers When multiplying similar bases with powers, add the powers together 33 x 34 =

35
Dividing powers When dividing similar bases with powers, subtract the powers ( )=
34
Raising a power to
When raising a power to another power, multiply the powers (35)3 =
another power
The numerator of the fractional power represents the power and the
Fractional powers 30.5 =
denominator of the fractional power represents the root.

2
1. (NO CALCULATOR) Which of the following is equal to 9 for all values of t ?

A) 9
B) 2
2
C) t9
9
D) 2

2. (NO CALCULATOR) Which real number satisfies (2n)(8) = 163 ?

2
xa
3. If 2 = x , x > 1, and a b = 4, what is the value of a + b ?
12
xb

145 145
PSAT Foundations of Arithmetic and Algebra: Absolute Value

Foundations of Arithmetic and Algebra: Absolute Value

Absolute value is a numbers distance from zero. As such, absolute values are never
negative.

Identify: Any problem that uses absolute value bars

Set Up: Create two equations and solve for the two values of the given variable.

1. What are the values for which 2x 3 < 7 ?

A) x < 2
B) x > 5
C) 5 < x < 2
D) 2 < x < 5

2. If |7a 10| > 4, then which of the following CANNOT be a value of a ?

A) 5
B) 3
C) 2
D) 4

3. (NO CALCULATOR) For what value of a is |a 2| + 3 equal to 0 ?

A) 0
B) 1
C) 2
D) There is no such value of a

146 146
PSAT Foundations of Arithmetic and Algebra: Percentage

Foundations of Arithmetic and Algebra: Percentage

A percentage is a number or ratio expressed as a fraction of 100.

Identify: Questions that involves percentages, or give before and after numbers.

Set Up: To determine percent change, plug any elements you know into the percent
change formula.

Percent Change Formula


Final Original
Percent Change = x 100%
Original

1. What number is 45 percent less than 600 ?

2. Janet goes to the store to buy herself a new dress, and she finds that there is a 20% off sale
going on. Additionally, Janet has a 20% discount coupon. If Janet buys the dress that is on
sale and also uses her coupon, what will her total discount be?

A) 20%
B) 38%
C) 36%
D) 40%

3. Steven works for a company that mines copper and iron. Last year, his company mines 1200
tons of copper and 1000 tons of iron. This year, the yield, by tonnage, of copper declined by 20
percent, and the yield, by tonnage, of iron increased by 40 percent. What percent of the
original tonnage is represented by the amount of copper and iron mined this year? (Round to
the nearest tenth of a percent.)

A) 92.7%
B) 106.8%
C) 107.3%
D) 112.7%

147 147
PSAT Foundations of Arithmetic and Algebra Notes

148 148
PSAT Foundations of Arithmetic and Algebra Review

1. (NO CALCULATOR) If (x2)n = x14 , what is the value of n ?

2. (NO CALCULATOR) What is the greatest common factor of 6y, 12y2, and 81y3 ?

A) 3y
B) 3y2
C) 6y2
D) 9x3

2x
3. + 4 < 8 is equivalent to which of the following expressions?
5

A) 15 < x < 15

B) 30 < x < 10
15 35
C) <x<
2 2
16
D) < x < 10
5

4. Toms factory produces toothpicks and coasters. Last month, his factory produced 1,200 pounds of
toothpicks and 800 pounds of coasters. This month, the production, by weight, of toothpicks declined by 40
percent, and the production, by weight, of coasters increased 40 percent. What was the percent decline in
the total weight of the two items produced by the factory?

4a
5. If 2a b = 6, what is the value of
2b

A) 26
B) 44
C) 8
D) The value cannot be determined from the information given.

149 149
SAT Foundations of Arithmetic and Algebra Answers

Workbook Answers Review Answers

Terms and Operations


1) 7

1) B 2) A

2) B 3) B

3) 20 4) 8%

Exponents 5) A

1) D

2) 9

3) 3

Absolute Value

1) D

2) C

3) D

Percentage

1) 330

2) C

3) C

150 150
PSAT Average, Median, and Mode

PSAT Math:
Average, Median,
and Mode

Important Formulas

Average (Arithmetic Mean) Formula


Sum
Average =
Total Number

Weighted Average Formula


Sum of Weighted Terms
Weighted Average =
Total Number

151 151
PSAT Average, Median, and Mode: Average (Arithmetic Mean)

Average, Median, and Mode: Average (Arithmetic Mean)

Identify: If you see the term average (arithmetic mean) in a math question, it is an
average question.

Set Up: Immediately write down the average formula. Plug whatever information is
given in the question into the formula and solve for whatever is missing in the equation.
Repeat as necessary.

Average Formula Weighted Average Formula


Sum Sum of Weighted Terms
Average = Weighted Average =
Total Number Total Number

1. Allen and three friends play in a softball league. This season, Allen struck out 45 times,
Bob struck out 72 times, Chad struck out 23 times, and Derek struck out 61 times. What is
the average number of strikeouts for these players, to the nearest 0.1 strikeouts?

2. The local city orchestra has 11 violin players. The five male violin players average 73
inches tall and the six female violin players average 67 inches tall. What is the average
height, in inches, of the 11 violin players? (Rounded to the nearest tenth of an inch.)

3. If a is the average (arithmetic mean) of 3n and 6, b is the average of 4n and 6, and c is the
average of 5n and 18, what is the average of a, b, and c in terms of n ?

A) 2n + 5
B) 2n + 10
C) 3n + 8
D) 6n + 15

152 152
PSAT Average, Median, and Mode: Median and Mode

Average, Median, and Mode: Median and Mode

Identify: You see the term median or mode.

Set Up: Median is the middle number in a set arranged from least to greatest. Mode
is/are the number(s) that appear(s) most frequently in a set.

1. A researcher chose 240 students at random from each of two high schools and asked each
student how many cousins he or she has. The results are shown in the table below. What is
the median number of cousins for all students surveyed?

Cousins Survey
Number of Cousins Dedham High School Norwood High School
2 80 70
3 40 60
4 65 45
5 55 65

A) 2
B) 3
C) 3.5
D) 4

2. (NO CALCULATOR) The median of a set of data containing five items was found. Six
data items were added to the set. Three of these items were greater than the original
median, and the other three items were less than the original median. Which of the
following statements must be true about the median of the new data set?

A) It is the average of the three new lower values.


B) It is the same as the original median.
C) It is the average of the three new higher values.
D) It is greater than the original median.

153 153
PSAT Average, Median, and Mode Notes

154 154
PSAT Average, Median, and Mode Review

Questions 1 and 2 relate to the below data

The histogram below shows the distribution of players in a local euchre league.

25

20
Frequency

15

10

0
20-29 30-39 40-49 50-59 60-69 70-79
Age (years)

1. Which of the following could be the median age of the 71 players in the league?

A) 22 years old
B) 41 years old
C) 50 years old
D) 63 years old

2. (NO CALCULATOR) Which of the following could NOT be the mode of the 71 players in the league?

A) 26 years old
B) 32 years old
C) 55 years old
D) 64 years old

3. (NO CALCULATOR) In Taylors karate class, the mean age of all the female members is 7 years, and
the mean age of all male members is 9 years. Which of the following must be true about the mean age y of
the combined group of male and female members of the karate club?

A) y = 7
B) y = 9
C) 7 < y < 9
D) y > 9

155 155
PSAT Average, Median, and Mode Review

Questions 4 and 5 relate to the below data

The histogram below shows the distribution of players in the Page Turners book club.
5

4
Frequency

0
18 19 20 21 22 23 24 25 26 27 28

Age (years)

4. What is the average (arithmetic mean) age of the members of the book club?

A) 21.9 years old


B) 22.7 years old
C) 23.9 years old
D) 25.1 years old

5. What is the median age of the 28 members of the book club?

A) 22.5 years old


B) 23 years old
C) 23.5 years old
D) 24 years old

6. A student receives grades on tests of between 0 and 100, inclusive. In the first five grades the student
received, the average (arithmetic mean) of the scores was 95. What is the least grade the student can receive for
the sixth test and still be able to have an average of at least 90 for the first 10 tests?

156 156
PSAT Average, Median, and Mode Answers

Workbook Answers Review Answers

Average (Arithmetic Mean)


1) C

1) 50.3 2) A

2) 69.7 3) C

3) A 4) C

Median and Mode 5) D

6) 25
1) B

2) B

157 157
PSAT Functions

PSAT Math:
Functions

158 158
PSAT Functions: f(x)

Functions: f(x)

Identify: Any question that includes function notation with numbers or variables
such as f(4), g(t), or h(6).

Set Up: The number in the parentheses is the number to plug in to the function wherever
you see a variable.

1. (NO CALCULATOR) If h(x) = 3x + 20, what is the value of h(4)?

g(4)
2. (NO CALCULATOR) If f(x) = x2 + x + 5 and g(x) = x , then what is the value of ?
f(1)

1
f(x) = 3x + a

3. In the function above, a is a constant. If f(6) = 12, what is the value of f(9)

A) 12
B) 16
C) 1
D) 7

4. (NO CALCULATOR) A function y satisfies y(3) = 6 and y(6) = 12. A function z satisfies
z(6) = 3 and z(12) = 3. What is the value of z(y(6)) ?

A) 3
B) 6
C) 9
D) 12

159 159
PSAT Functions: f(x a)

Functions: f(x a)

Identify: Any question that includes function notation with a variable AND a
number and/or a second variable in the original function such as f(x 3), g(t + 4),
or h(a + b)

Set Up: If the original input is an equation (for example, x 1), first solve for the variable
by setting the two values in parentheses equal to one another and then input the new value
into the equation.

1. (NO CALCULATOR) If the function f is defined by f(x + 7) = 4x + 10, what is the value
of f(7)?

A) 7
B) 10
C) 14
D) 38

2. If the function h is defined by h(z 2) = 2z2 100, what is the value of h(7)?

A) 34
B) 2
C) 0
D) 62

3. If the function z is defined by z( 2) = 4 + 54, what is the value of z(10)?

160 160
PSAT Functions: Graphs

Functions: Graphs

Identify: Problems involving a graph of variables (x) and functions f(x).

Set Up: The input to the function (what is inside the parentheses) is also an x-
coordinate on the graph. The output of the function is the corresponding y-
coordinate on the graph.

2
y = f(x)
1
y = g(x)
x
-2 -1 O 1 2

Questions 1, 2 and 3 relate to the above graph

1. (NO CALCULATOR) The graph of y = f(x) is shown above. What is f(1) ?

A) 1
B) 0
C) 1
D) 2

2. (NO CALCULATOR) The graph of y = g(x) is shown above. If g(q) = 1, which of the following
is a possible value of q?

A) 1.5
B) 0.5
C) 0
D) 1.5

3. (NO CALCULATOR) Graphs of the functions f and g are shown in the xy-plane above. For
which of the following values of x does f(x) + g(x) = 2

A) 2
B) 1
C) 1
D) 2

161 161
PSAT Functions: Tables

Functions: Tables

Identify: Problems involving a table of variables (x) and functions f(x).

Set Up: Pick a variable (x) from the table and plug it into each of the answer
choice functions. The output value should equal the corresponding f(x) from the table.

x 2 4 6 8
f(x) 2 0 2 4

1. (NO CALCULATOR) The table above gives values of the linear function f(x) for selected
values of x. Which of the following defines f(x) ?

A) f(x) = x + 4
B) f(x) = x 4
C) f(x) = 2x 8
D) f(x) = 4x 16

x 1 2 3 4
y 3 4 5 6

2. (NO CALCULATOR) The table above gives values of the linear function f(x) for selected
values of x. Which of the following defines y ?

A) y = x3
B) y = 2x
C) y = 3x
D) y=x+2

162 162
PSAT Functions Notes

163 163
PSAT Functions Review

1. (NO CALCULATOR) If f(x) = 7x 12, then f(7) =

r(p) = p2 + p
s(p) = p2 p

2. (NO CALCULATOR) Which of the following is equivalent to s(t + 1) ?

A) r(t)
B) r(t) + 1
C) r(t) 1
D) s(t) + 1

15
10
5

15 10 5 O 5 10 15 x
5
10
15

3. (NO CALCULATOR) The figure above shows the graph of the function f. If f(q) = 5, which of the following
is closest to q ?

A) 13
B) 7
C) 1
D) 11

4. (NO CALCULATOR) If f(x + 5) = x + 4, then f(4) =

5. Consider the function c(t) = 2t3 + 3. What is the value of c(c(2)) ?

164 164
SAT Functions: Answers

Workbook Answers Review Answers

Functions: f(x)
1) 37

1) 32 2) A

2 3) A
2) 7
4) 3
3) D
5) 4,391
4) A

Functions: f(x a)

1) B

2) D

3) 102

Functions: Graphs

1) D

2) B

3) A

Functions: Tables

1) B

2) D

165 165
PSAT Quadratics and Polynomials

PSAT Math:
Quadratics and
Polynomials

Important Formulas
Quadratic Formula

2
b b 4ac
x=
2a

166 166
PSAT Quadratics and Polynomials: Quadratics

Quadratics and Polynomials: Quadratics

Identify: Look for a quadratic equation or binomials (two things in parentheses being
multiplied together).

Set Up: Keep the FOIL rules in mind: First, Outer, Inner, Last.

Look for patterns, like these three commonly tested quadratics:

(x + y)2 = (x +y)(x + y) = x2 + 2xy + y2


(x y)2 = (x y)(x y) = x2 2xy + y2
(x + y)(x y) = x2 y2

1. (NO CALCULATOR) If x2 y2 = 36 and x + y = 9, what is the value of (x y) ?

2. (NO CALCULATOR) 16a4 + 24a2b2 + 9b4

Which of the following is equivalent to the expression shown above?

A) (4a2 + 3b2)2
B) (2a + 3b)4
C) (2a2 + 3b2)2
D) (4a + 3b)4

3. (NO CALCULATOR) If (ax + 4)(bx + 5) = 8x2 + cx + 20 for all values of x, and a + b = 6,


what are the two possible values for c ?

A) 6 and 20
B) 12 and 20
C) 26 and 28
D) 28 and 38

167 167
PSAT Quadratics and Polynomials: Polynomial Division and Polynomial Factors

Quadratics and Polynomials: Polynomial Division

Identify: A question has a polynomial divided by a second algebraic equation.

Set Up: When dividing, focus on the left-most part of the divisor. This is the part of
the equation that will go into the dividend.

7x2 3x 4
1. (NO CALCULATOR) =?
x+ 2

22
A) 7x 17 + x + 2

22
B) 7x 17 + x + 2

30
C) 7x 17 x + 2

30
D) 7x 17 + x + 2

Quadratics and Polynomials: Polynomial Factors

Identify: A question references the factor(s) of a polynomial equation.

Set Up: For a number and/or equation to be a factor of a polynomial, it must divide the
polynomial evenly (divide the polynomial with no remainder).

1. (NO CALCULATOR) The function f is defined by a polynomial. Some values of x and f(x)
are shown in the table above. Which of the following must be a factor of f(x) ?

x 1 3 5 7
f(x) 4 2 1 0

A) x 1
B) x 3
C) x 5
D) x 7

168 168
PSAT Quadratics and Polynomials: Polynomial Solving - Quadratic Formula

Quadratics and Polynomials: Polynomial Solving Quadratic Formula

Identify: A question that requires finding the solutions to a polynomial equation or


asks for the zero of a polynomial function.

Set Up: If possible, convert the equation into the form ax2 + bx + c = 0 and then plug the
given information into the Quadratic Formula.
Quadratic Formula

2
b b 4ac
x=
2a

A zero (or root) of a polynomial function is a number that, when plugged in for the
variable, makes the function equal to zero.

1. (NO CALCULATOR) Which of the following is a zero of the function f(y) = y2 + 6y 6 ?

A) 3 + 15

B) 3 + 15
C) 6 + 15

D) 6 15

3x2 + 2x 8 = 0

2. If a and b are two solutions of the equation above, and a > b, which of the following is the
value of a b ?

169 169
PSAT Quadratics and Polynomials: Polynomial Solving - Completing the Square

Quadratics and Polynomials: Polynomial Solving Completing the


Square

Identify: A question which requires finding the solutions to a polynomial equation with
complex factors.

Set Up: Polynomials can be solved by completing the square.

Completing the square steps:

Step 1: Divide all terms by the coefficient of x2.

Step 2: Move the number term to the right side of the equation.

Step 3: Take half of the coefficient of the x-term, and square it. Add this square to both sides of
the equation. Convert the left-hand side to squared form, and simplify the right-hand side.

Step 4: Take the square root on both sides of the equation.

Step 5: Subtract the number that remains on the left side of the equation to find x.

x2 + y2 + 8x + 10y 215 = 0

1. The equation of a circle in the xy-plane is shown above. What is the diameter of the circle?

A) 32
B) 64
C) 90
D) 128

170 170
PSAT Quadratics and Polynomials: Higher-Order Equations

Quadratics and Polynomials: Higher-Order Equations

Identify: A question will involve the solution to a polynomial formula where the
highest power is greater than 2.

Set Up: If the highest power is even, let x2 = a, solve for a, then replace a with x2 and
solve to get the answers to the original equation. If the highest power is odd, factor out
an x, and then follow the steps for an even power.

A zero (or root) of a polynomial function is a number that, when plugged in for the
variable, makes the function equal to zero. A polynomial of degree n will have n roots,
some of which may be multiple roots (they repeat).

1. (NO CALCULATOR) What are the roots of the equation x4 = 20x2 64 ?

A) x = 0, x = 4
B) x = 0, x = 2
C) x = 4, x = 8
D) x = 2, x = 4

2. What are the zeroes of the equation 3x5 = 36x3 96x ?

A) x = 0, x = 2 2, x = 2
B) x = 0, x = 2 2, x = 2
C) x = 0, x = 2, x = 3
D) x = 0, x = 2, x = 3

171 171
PSAT Quadratics and Polynomials Notes

172 172
PSAT Quadratics and Polynomials Review

1. If x2 y2 = 100 and x + y = 4, what is the value of (x y) ?

2. What are the solutions of 7a 34 = a2 ?

3. (NO CALCULATOR) What is the sum of the zeroes of the function (F(x) = 0) when
F(x) = (x2 + 6x + 5)(x + 10) ?

x3 1
4. (NO CALCULATOR) x + 2 = ?
9
A) x2 2x + 4 +
x2
9
B) x2 2x 4 + x + 2

9
C) x2 + 2x + 4 +
x+2
9
D) x2 2x + 4 +
x+2

x2 + y2 4x + 12y = 41

5. The equation of a circle in the xy-plane is shown above. What is the diameter of the circle?

A) 6
B) 9
C) 18
D) 36

173 173
PSAT Quadratics and Polynomials Answers

Workbook Answers Review Answers

Quadratics
1) 25

1) 4
7 185
2) a =
2) A 2

3) C 3) 16

4) D
Polynomial Division
5) C
1) D

Polynomial Factors

1) D

Quadratic Formula

1) B

10
2) 3

Completing the Square

1) A

Higher-Order Equations

1) D

2) A

174 174
PSAT Ratio, Proportion, and Unit Conversion

PSAT Math:
Ratio, Proportion, and
Unit Conversion

Important Formulas
Ratio Formula Direct Proportional Relationship
One Part One Part One Part
Ratio = =
Another Part Whole Thing Whole Thing

Probability
Desired Outcomes
Probability =
Total Outcomes

175 175
PSAT Ratio, Proportion, and Unit Conversion: Ratios and Proportional Relationships

Ratios and Proportional Relationships

Identify: If you see a question that defines a ratio and/or a proportional relationship.

Set Up: Write down the ratio and/or proportional relationship formulas and fill in
the provided information to solve the question. For proportional relationships, cross
multiply to solve for the unknown variable. Make sure your units are consistent.

Ratio Formula Direct Proportional Relationship


One Part One Part One Part
Ratio = =
Another Part Whole Thing Whole Thing

1. (NO CALCULATOR) Kates parents will allow her to buy six cookies for every 4 times she
takes out the trash this month. If Kate takes out the trash 10 times this months, how many
cookies will she be able to purchase?

2. A tennis instructor charges $70 per lesson, plus an additional fee for the use of the court.
The additional fee varies directly with the square root of the amount of time, in minutes, the
court is used. If a lesson plus 120 minutes of court time costs $150, approximately what is the
total amount charged for a lesson with 90 minutes of court usage? (Round to the nearest cent.)

A survey of 33,443 randomly selected teenagers aged 13 through 16 in the United States was
conducted to gather data on social media usage. The data are shown in the table below.
Use Social Media Do Not Use Social Media Total
Ages 13 and 14 12,145 8,232 20,377
Ages 15 and 16 8,545 4,521 13,066
Total 20,690 12,753 33,443

3. Based on the data, how many times more likely is it for 13 year old or a 14 year old to use
social media than it is for a 15 year old or a 16 year old to use social media? (Round to the
nearest hundredth.)

176 176
PSAT Ratio, Proportion, and Unit Conversion: Ratios and Proportional Relationships

Ratios and Proportional Relationships (Colons)

Identify: If you see a question that defines a ratio and/or proportional relationship
between two or more things expressed with a colon.

Set Up: Represent each number in the ratio as the coefficient of a variable. Each
coefficient represents a portion of the overall total:

Proportion of Overall Total Relationship


An individual coefficent
Proportion =
The sum of the coefficents

1. If the degree measures of the angles of a quadrilateral are in the ratio 1:2:3:4, by how
many degrees does the measure of the largest angle exceed the measure of the smallest
angle?

2. Triangle ABC above is isosceles with AB = AC and BC = 120. The ratio of ED to FD is 4:6.
What is the length of BD ?

177 177
PSAT Ratio, Proportion, and Unit Conversion: Probability

Probability

Identify: The question will mention probability, often in reference to data


presented in a chart or graph.

Set Up: List the elements you know, and plug them into the probability formula:

Probability
Desired Outcomes
Probability =
Total Outcomes

1. In a survey of 1,000 drivers, all but 350 claimed that they always wear a seat belt while
driving. If one of the surveys participants is randomly chosen, what is the probability that
this person claimed to always wear a seat belt while driving?

Listen to Podcasts Do Not Listen to Podcasts Total

Ages 20 to 24 11,145 5,232 16,377


Ages 25 to 29 4,673 3,950 8,623
Total 18,818 9,182 24,000

2. A survey of 24,000 randomly selected American aged 20 through 29 in the United States
was conducted to gather data on podcast listenership. The data are shown in the table above.

Based on the data, if a person from the survey is picked at random, what is the probability
that the person will either be a person of ages 20-24 who listens to podcasts or a person of ages
25-29 who does not listen to podcasts

A) 0.39
B) 0.47
C) 0.63
D) 0.74

178 178
PSAT Ratio, Proportion, and Unit Conversion: Unit Conversion

Unit Conversion

Identify: You are being asked a question which asks for a conversion between different
units.

Set Up: Use relevant conversions to cancel units between the provided unit(s) and the
desired unit(s).

1. (NO CALCULATOR) A certain juice drink carton contains 5 pints. What is the total
volume, in gallons, of the contents of a crate that contains 56 of these juice drink cartons?
(8 pints = 1 gallon)

A) 12
B) 20
C) 27
D) 35

2. A local restaurant serves soda to customers by the pint. Each week, the restaurant receives
a 20 gallon shipment of soda syrup. How many customers can be served which this shipment?
(1 gallon = 128 ounces; 1 pint = 16 ounces)

A) 100
B) 128
C) 144
D) 160

3. A typical image taken by Jims digital camera is 12.5 megabits in size. Jim can upload his
photo to the internet at a rate of 15 kilobits per second for a maximum of 8 hours each day. If
1 megabit equals 1,024 kilobits, what is the maximum number of typical images that Jim can
upload to the internet each day? (Round to the nearest whole number.)

179 179
PSAT Ratio, Proportion, and Unit Conversion Notes

180 180
PSAT Ratio, Proportion, and Unit Conversion Review

1. In her last two track meets, Heather has run 800 meters and 1500 meters. How many inches did Heather run
during those two meets? (Round to the nearest inch.) (1 meter 39.37 inches)

2. (NO CALCULATOR) A bowl is filled with jellybeans of three colors: orange, purple, and green. Suppose that
if John reaches into the bowl and picks a jellybean at random, the probability that he chooses a green jellybean
1
is 4 . Which of the following could NOT be the total number of jellybeans in the bowl?

A) 20
B) 36
C) 42
D) 56

3. According to a certain mixture, 45 pounds of mulch are needed to cover 60 square feet of a flower bed. At this
rate, how many pounds of mulch are needed to cover 780 square feet of a flower bed?

4. In a mixture of cake and ice cream, the ratio by weight of cake to ice cream is 3:11 per pound of mixture. How
many pounds of cake will there be in 2 pounds of this mixture?

5. Ryan traveled in his car at an average speed of 55 kilometers per hour for 12 hours and consumed fuel at a
rate of 40 kilometers per gallon. How many gallons of fuel did the car use for the entire 12-hour trip?

A) 16.5
B) 18
C) 20.5
D) 24

6. There are 150 green marbles and 100 yellow marbles in a bag that contains 250 marbles. If only yellow
marbles are added to the bag so that the probability of randomly drawing a yellow marble from the bag becomes
3
5 , how many yellow marbles are in the bag?

A) 125
B) 175
C) 225
D) 250

181 181
PSAT Ratio, Proportion, and Unit Conversion Answers

Workbook Answers Review Answers

Ratios and Proportional Relationships


1) 90,551

1) 15 2) C

2) $139.28 3) 585

3) 0.91
3
4) 7
Ratios and Proportional Relationships
(Colon) 5) A

6) C
1) 108

2) 48

Probability

13
1) 20

2) C

Unit Conversion

1) D

2) D

3) 34

182 182
PSAT Solving Linear Equations

PSAT Math:
Solving Linear
Equations

183 183
PSAT Solving Linear Equations: Slope-Intercept Form

Solving Linear Equations: Slope-Intercept Form

Identify: The question will give you at least one point, a slope, and/or a y- or x-
intercept of a line.

Set Up: Write down the slope-intercept form and use it to solve the question. Parallel
lines have the same slope. Perpendicular lines have slopes which are negative reciprocals.

Slope-Intercept Form
y = mx + b
m = slope
b = y-intercept

1. (NO CALCULATOR) In the standard (x, y) coordinate plane, what is the y-intercept of the
line given by the equation 4x + 7y = 14 ?

1
2. In the (x, y) coordinate plane, a line passes through the point (4, 6) and has a slope of 3.
What is the x-coordinate of a point on the line having a y-coordinate of 3?

Solving Linear Equations: Slope-Intercept Form Application

Identify: The question asks for an interpretation of an equation in slope-intercept form (y =


mx + b).

Set Up: The slope of a line (m) represents average (arithmetic mean) rate of change.
The y-intercept (b) represents the value of y when x equals 0.

1. (NO CALCULATOR) The mean height of trees (in feet), y, at a local park can be estimated
using the equation y = 63.232x + 13.435, where x represents the number of years since 2010
and x 5. Which of the following statements is the best interpretation of the number 13.435 in
the context of this problem?

A) The estimated mean height of trees (in feet) in 2010.


B) The estimated mean height of trees (in feet) in 2015.
C) The estimated yearly decrease in the mean height of trees (in feet).
D) The estimated yearly increase in the mean height of trees (in feet).

184 184
PSAT Solving Linear Equations: Distance, Midpoint, and Slope

Solving Linear Equations: Distance and Midpoint Formulas

Identify: You are asked to find the distance between two points, or the midpoint of
two points.

Set Up: Immediately write down the distance or midpoint formula. Plug whatever
information is given in the question into the formula and solve for whatever is missing in
the equation.
Distance Formula Midpoint Formula
x1 + x2 y1 + y2
Midpoint =
Distance = x2 x1 2 + y2 y1 2 , 2
2

1. What is the distance, in units, between the points with standard (x, y) coordinates (2, 3)
and (6, 6) ?

2. (NO CALCULATOR) What is the midpoint of the points with standard (x, y) coordinates
(5, 7) and (9, 3) ?

A) (2, 5)
B) (4, 4)
C) (4, 4)
D) (5, 2)

Solving Linear Equations: Slope

Identify: You will be given two points on a line in the xy-coordinate plane and be asked
about the slope.

Set Up: Write down the slope formula and use it to solve for the missing information.
Slope Equation
change in y (rise) y2 y1
slope (m) = =
change in x (run) x2 x1

1. A line in a coordinate plane has points of (1, w) and (3, 8). The slope of the line is 1.5.
What is the value of w?

185 185
PSAT Solving Linear Equations Notes

186 186
PSAT Solving Linear Equations Review

1. (NO CALCULATOR) In the standard (x, y) coordinate plane, what is the slope of the line
given by the equation 5x + 4y = 17 ?

2. In the xy-coordinate plane, what is the distance between the points with coordinates (4, 2)
and (2, 6)?

3. What is the slope of a line perpendicular to the line that passes through (2, 5) and (7, 9) in
the standard (x, y) coordinate plane?

9
4. (NO CALCULATOR) A line in the xy-plane passes through the origin and has a slope of 2.
Which of the following points lies on the line?

A) (0, 9)
B) (0, 2)
C) (2, 12)
D) (4, 18)

5. (NO CALCULATOR) The amount of time it takes students at a local elementary school to
run one mile (in minutes), y, can be represented by the equation y = 0.642x + 18.125, where x
represents the age of the student and 6 < x < 10. Which of the following statements is the best
interpretation of the number 0.642 in the context of this problem?

A) The estimated mile time of students (in minutes) in 2005.


B) The estimated mile time of students (in minutes) in 2009.
C) The estimated yearly decrease in the mile time of students (in minutes).
D) The estimated yearly increase in the mile time of students (in minutes).

6. (NO CALCULATOR) In the system of linear equations below, d is a constant. If the system
has infinite solutions, what is the value of d ?
1 1
3x 8y = 5
5
dx 8y = 25

187 187
SAT Solving Linear Equations Answers

Workbook Answers Review Answers

Slope-Intercept Form
5
1) 4
1) 2
2) 20
2) 5

Slope-Intercept Form Application 9


3) 14

1) A 4) D

Distance and Midpoint Formulas 5) C

1) 5 5
6) 3
2) A

Slope Formula

1) 2

188 188
PSAT Solving Non-Linear Equations

PSAT Math:
Solving Non-Linear
Equations

189 189
PSAT Solving Non-Linear Equations: Single Equation Questions

Solving Non-Linear Equations: Single Equation Questions

Identify: A question (before or after translation from a word problem) has one equals
sign and/or inequality or you are solving for the value of an equation.

Set Up: Follow the below steps (where necessary):

1) Group like terms (variables with the same exponent and/or whole numbers).
2) Multiply by a common denominator and/or cross-multiply to remove fractions.
3) Solve for the variable.

1. (NO CALCULATOR) The inequality 5(x 3) < 7(x + 1) is equivalent to which of the
following inequalities?

A) x > 11
B) x > 8
C) x > 2
D) x > 11

7b2 + 2 x = 0

2. If b > 0 and x = 4 in the equation above, what is the value of b ?

A) 1
B) 2
C) 2
D) 2 2

s = 80 + 10.5x

3. One end of a spring is attached to a ceiling. When an object of mass x kilograms is attached
to the other end of the spring, the spring stretches to a length of s centimeters as shown in the
equation above. What is x when s is 290 ?

A) 14.8
B) 20.0
C) 37.0
D) 75.2

190 190
PSAT Solving Non-Linear Equations: Systems of Equations Questions

Solving Non-Linear Equations: Systems of Equations Questions

Identify: There are two or more equals signs and/or inequalities.

Set Up: Follow the below steps (where necessary):

1) Group like terms (variables with the same exponent and/or whole numbers).
2) Add or subtract given equations (if possible).
3) If you cannot add or subtract, then substitute between equations.

1. (NO CALCULATOR) If g = 2, h = 5, i = 7, and j = 4 what is the value of gi + hj ?

a 8b
2. (NO CALCULATOR) If = 4, what is the value of a ?
b

3. If k = 5 4 and 2k = 4x2 , and x > 0, what is the value of x ?

2x 2y = y + 4
6y + 8x = 4x

x
4. Solve for y:
.

191 191
PSAT Solving Non-Linear Equations: Rewriting Equations

Solving Non-Linear Equations: Rewriting Equation Questions

Identify: A question that relates to manipulating equations to create new


expressions. These questions will often ask for one variable in terms of another.

Set Up: Use algebraic operations and/or substitution to convert the provided
expression into the desired equation. The phrase in terms of translates to solve
for the first variable. Thus, if a question asks to solve a in terms of b, you should solve
the equation for a. The b variable will appear on the right side of the equation.

1. (NO CALCULATOR) If 2b + 1 = 2x 2, what is x in terms of b ?


1
A) x = b + 2
b+3
B) x = 2
3
C) x = b + 2

D) x = 2b

2. (NO CALCULATOR) A circle has area D and radius t. Which of the following represents t
in terms of D ?
D
A) t = 2
D
B) t =
D
C) t = 2
D
D) t =

4
3. (NO CALCULATOR) If a1 = 3 , were a > 0 and x > 0, which of the following equations

gives x in terms of a ?
1
A) x =
a
1
B) x = a3

C) x = a3

D) x = 64a3

192 192
PSAT Solving Non-Linear Equations Notes

193 193
PSAT Solving Non-Linear Equations Review

4x x + 8
=
3 4

1. (NO CALCULATOR) In the equation above, what is the value of x ?

2
2. (NO CALCULATOR) If = 5d where d 0 and c 1, what is d in terms of c ?
c1

A) d = 5c 5

2
B) d =
5c 5
1
C) d = 2c 2

2
D) d = 5 c + 2

2 4
3. If a b = 6, what is the value of 14a 20b ?
5 7

A) 41
B) 72
C) 120
D) 210

2y
4. (NO CALCULATOR) Which of the following is equivalent to (x 2y) (2xy)

2y
A) 1 x

y2
B) + 2xy
x
x x
C) y + y2

D) 2xy

194 194
PSAT Solving Non-Linear Equations Answers

Workbook Answers Review Answers

Single Equation Questions 24


1) 13

1) A 2) B
2) B 3) D
3) B 4) A

Systems of Equations

1) 34

2) 2

3) 10

3
4) 2

Rewriting Equations

1) C

2) B

3) D

195 195
PSAT Translating Word Problems

PSAT Math:
Translating Word
Problems

196 196
PSAT Translating Word Problems: Translation Table

Translating Word Problems: Translation Table

Use the following chart to assist in translating word problems into algebra:
.

Term Translation
Each,
Groups of (a number)
Number of Times, Variable (x , y, etc.)
Per, Some
Equals, Is =
And, Plus, Sum +
Difference
Product x

For every, Per (ratio)


Exceed(s)
Greater Than >

Meet or exceed

Greater Than or Equal To
Less Than <
Less Than or Equal To

197 197
PSAT Translating Word Problems: Linear Equations

Translating Word Problems: Linear Equations

Identify: A word problem that mentions a linear model. A linear model relates to the
formula y = mx + b, where m is the slope (the rate of change) and b is the y-
intercept (the amount when x = 0).

Set Up: Plug the given information into slope intercept form (y = mx + b) and then
solve for the requested information.

1. (NO CALCULATOR) Yesterday, Daniel ran a half marathon. At 11 a.m., Daniel had
finished four miles and at 1 p.m. Daniel had finished 10 miles. What was Daniels speed in
miles per hour from 11 a.m. to 1 p.m.?

2. (NO CALCULATOR) Tammy is a member of a sorority which has a yearly membership fee
of $500 plus the requirement to donate $15 for each social function that is held during the
year. Which of following functions gives Tammys cost, in dollars, for a year in which the
sorority holds f social functions?

A) S(f) = 500 + 15f


B) S(f) = 500f + 15
C) S(f) = 15f
D) S(f) = 515f

3. As Nathan traveled into the Sahara desert, the air temperature increased at a constant
rate. If at 10 miles into the desert the temperature was 97 degrees Fahrenheit, and at 210
miles into the desert the temperature was 107 degrees Fahrenheit, which of the following
linear models best describes the relationship between temperature t and distance traveled into
the desert d ?

A) t = 0.05d + 96.5
B) t = 0.05d + 97
C) t = 10d + 96.5
D) t = 10d + 200

198 198
PSAT Translating Word Problems: Non-Linear Equations

Translating Word Problems: Non-Linear Equations

Identify: After translating a word problem equation there is one equation and/or you are
solving for the value of a variable.

Set Up: Follow the below steps after translation (where necessary):

1) Group terms into a single equation with an equals or inequality sign.


2) Plug any information provided in the question into the equation.
3) Solve for the equation and/or for the requested variable.

1. A local store charges $2.50 for one loaf of bread and $8.25 for a pound of roast beef. If Jim
bought two loaves of bread and three pounds of roast beef, how much money did Jim spend?

2. (NO CALCULATOR) The number of boys that played lacrosse in a local town this year is
three times the number of boys that played lacrosse in the same town last year. If 150 boys
played lacrosse this year and y boys played last year, which of the following equations is true?

A) 150y = 3
B) 3y = 150
y
C) 3 = 150

D) y + 150 = 3

3. (NO CALCULATOR) Sean is renting a car for a cross country road trip. The car rental
company charges a one-time flat fee of $20.00 plus $49.99 per day. A tax of 11% is also applied
to the daily cost for insurance. Which of the following represents Seans total cost, in dollars,
for renting a car for x days?

A) 1.11(49.99x) + 20x
B) 1.11(69.99x)
C) 1.11(49.99x + 20)
D) 1.11(49.99x) + 20

199 199
PSAT Translating Word Problems: System of Equations

Translating Word Problems: System of Equations

Identify: After translating a word problem equation there are multiple equations
and/or inequalities.

Set Up: Follow the below steps (where necessary):

1) Group like terms (variables with the same exponent and/or whole numbers).
2) Add, subtract, or substitute between the given equation.
3) Solve for the equations and/or for the requested variable

1. (NO CALCULATOR) John exercises by doing sit ups, a, in groups of ten and push ups, b,
in groups of 20. If last week John did 20 total groups of exercise for a total of 450 completed
movements, which system of equations would yield the number of push ups John completed?

A) a + b = 450
20a + 10b = 20

B) a + b = 20
20a + 10b = 450

C) a + b = 450
10a + 20b = 20

D) a + b = 20
10a + 20b = 450

2. The sum of three numbers is 930. One of the numbers, x, is 50% less than the sum of the
other two numbers. What is the value of x ?

3. Mr. Hoffman has a beaker containing q milliliters of solution to distribute to the students in
his chemistry class. If he gives each student 4 milliliters of solution, he will have 30 milliliters
left over. In order to give each student 6 milliliters of solution, he will need an additional 8
milliliters. How many students are in the class?

200 200
PSAT Translating Word Problems Notes

201 201
PSAT Translating Word Problems Review

1. (NO CALCULATOR) In a number game, players receive points in increments of either three or five. If a
player scores 47 total points, and 35 of those points come in increments of 5, how many increments of three did
the player score?

2. (NO CALCULATOR) Thomas subscribes to a magazine that charges a yearly fee of $25 plus $2 extra for
each special issue that is released during the year. Which of the following functions gives Thomass cost, in
dollars, for a year in which the magazine releases m special issues?

A) F(m) = 25m
B) F(m) = 25m + 2
C) F(m) = 25+ 2m
D) F(m) = 25m + 2m

3. (NO CALCULATOR) A homeowner purchased 5,000lbs. of stone to build a new walkway. Some of the
stones weighed 25lbs. and some of the stones weighed 10lbs. The 25lbs. stones were ordered at a ratio of 3:2 to
the 10lbs. stones. Solving which of the following equations yields the number of 25lbs. stones.

A) 25x + 10y = 5000


3x = 2y

B) 25x + 10y = 5000


2x = 3y

C) 3x + 2y = 5000
25x = 10y

D) 2x + 3y = 5000
10x = 25y

4. A local city has two jugglers that perform at birthday parties. Julio charges a flat $250 fee plus $10 per
minute spent performing and Erin charges a flat $130 fee plus $25 per minute spent performing. If j represents
the number of minutes performed, what are all values of j for which Erins total charge is greater than Julios
total charge?

A) j > 8
B) 6 j 8
C) 4 j 6
D) j < 4

5. In Blueberry City, houses are built to accommodate groups of 2 and 4 people. This year, 1,200 people live in
Blueberry City in a total of 500 homes. In Blueberry City, how many people live in houses of 2 people?

202 202
PSAT Translating Word Problems Answers

Workbook Answers Review Answers

Linear Equations 1) 4

2) C
1) 3 mph
3) B
2) A
4) A
3) A
5) 800
Non-Linear Equations

1) $29.75

2) B

3) D

System of Equations

1) D

2) 310

3) 11

203 203
PSAT Circles

PSAT Math:
Circles

Important Formulas
Area Formula Circumference Formula

Area = r2 Circumference = 2r

Sector Formula Arc Formula


Central Angle Central Angle
Sector = Area( ) Arc = Circumference( )
360 360

204 204
PSAT Circles: Circumference and Area

Circles: Circumference and Area

Identify: Any question that provides a circumference and/or area of a circle.

Set Up: Write down the responding formula to find missing information (often the
radius). Repeat as necessary.

Circumference Formula Area Formula

Circumference = 2r Area = r2

1. A certain circle has an area of 16 square inches. How many inches long is its diameter?

2. If circle A has a circumference of 120, and circle B has a circumference equal to the length
of a 30 arc of circle A, what is the area of circle B ?

A) 10
B) 12
C) 25
D) 50

120 B
O

3. In the figure above, point O is the center of the circle, line segments AB and BC are tangent
to the circle at points A and C, respectively, and the segments intersect at point B as shown. If
the circumference of the circle is 36, what is the length of minor arc AC ?

205 205
PSAT Circles: Arc and Sector

Circles: Arc and Sector

Identify: Any question with a sector, a central angle, or an arc.

Set Up: Use the arc and sector formulas.

Arc Formula Sector Formula


Central Angle Central Angle
Arc = Circumference( ) Sector = Area( )
360 360

1. In a circle with a radius of 30 inches, what is the length, in inches, of an arc on the circle
with a central angle measure of 120?

A) 4
B) 10
C) 20
D) 30

2. In a circle with a diameter of 10 inches, what is the area, in square inches, of a sector of the
circle with a central angle measure of 72?

A) 5
B) 10
C) 20
D) 30

2
3. In a circle with center O, central angle AOB has a measure of 3 radians. The area of the
sector formed by central angle AOB is what fraction of the area of the circle?

206 206
PSAT Circles: Chords

Circles: Chords

Identify: Any question with a chord. A chord is a line segment whose endpoints both lie on a
circle.

Set Up: Use the Pythagorean Theorem to solve for the missing information.

1. In the circle above, F is the center. Chord DE is 24 feet long and is 5 feet from G. What is
the distance from point G to point D ?

2. The area of the above circle is 64, the distance from point C to point D is 6, and
AB is the circles longest chord. What is the distance from point D to point A ?

A) 6
B) 6 2
C) 10
D) 10 2

3. A circle has a radius of r inches, and chord CD is parallel to the diameter AB. If the
length of CD is 49 of the length of AB, what is the distance between the chord and the
diameter in terms of r? (round to the nearest hundredth.)
A) 0.43r
B) 0.57r
C) 0.76r
D) 0.90r

207 207
PSAT Circles: Inscribed Circles

Circles: Inscribed Circles

Identify: Any question which references an inscribed circle.

Set Up: A length of a square is the same as the diameter of an inscribed circle. The
diameter of a circle is the same length as the diagonal of an inscribed square.
.

1. A circle is inscribed within a square. The square has an area of 144 square units. What is
the circumference of the inscribed circle?

A) 6
B) 12
C) 36
D) 144

2. A circle is inscribed in a square. If the area of the inscribed circle is 81, what is
the area of the square?

A) 81
B) 81
C) 324
D) 324

3. A square is inscribed in a circle. If the area of the inscribed square is 16, what is
the area of the circle?

A) 8
B) 8 2
C) 16
D) 16 2

208 208
PSAT Circles Notes

209 209
PSAT Circles Review

1. In a circle with a radius of 12 inches, what is the length, in inches, of an arc on the circle
with a central angle measure of 180 ?

2. In a circle with a diameter of 20 inches, what is the length, in inches, of a sector of the circle
with a central angle measure of 90 ?

3. A circle is inscribed within a square. The circle has an area of 400 square units. What is
the perimeter of the square?

A) 40
B) 80
C) 160
D) 1,600

4. The area of circle A is four times greater than of the area of circle B. If the circumference of
circle B is 8, what is the circumference of circle A?

A) 4
B) 8
C) 16
D) 32

5. In the circle below, B is the center. Chord AD is 800 feet long and is 300 meters from C.
What is the distance from point A to point C ?

210 210
PSAT Circles: Answers

Workbook Answers Review Answers

Circles: Circumference and Area


1) 12

1) 13 2) 25

2) C 3) C

3) 6 4) C

Circles: Arc and Sector 5) 500

1) C

2) A

1
3) 3

Circles: Chords

1) 13

2) C

3) D

Circles: Inscribed Circles

1) B

2) C

3) A

211 211
PSAT Geometry in Three Dimensions

PSAT Math:
Geometry in Three
Dimensions

Important Formulas
Cone Volume Sphere Volume
1 4
Volume = r2 h Volume = r3
3 3

Cylinder Surface Area Cylinder Volume

Surface Area = 2r2 + 2rh Volume = r2 h

Rectangular Prism Surface Area Rectangular Prism Volume

Surface Area = 2wl + 2hl + 2hw Volume = lwh

Rectangular Pyramid Volume


1
Volume = 3lwh

212 212
PSAT Cones and Cylinders

Cones

Identify: Any question that references the volume of a cone or the distance from the
apex of a cone to a point along the base.

Set Up: Write down the cone volume formula to find the missing information for a
volume question. The distance from the apex of the cone to a point along the base is
the hypotenuse of a right triangle.

Cone Volume Formula


1
Volume = r2 h
3

1. A cone has a height of 60 and a base diameter of 30. What is the volume of this cone?

A) 1,500
B) 4,500
C) 9,000
D) 18,000

Cylinders

Identify: Any question that references the surface area or volume of a cylinder.

Set Up: Write down the appropriate formula to find the missing information. Repeat as
necessary.

Cylinder Surface Area Formula Cylinder Volume Formula

Surface Area = 2r2 + 2rh Volume = r2 h

1. A cylinder has a volume of 2,160 and a height of 15. What is the radius of this cylinder?

213 213
PSAT Rectangular Prisms and Spheres

Rectangular Prisms

Identify: Any question that references the surface area or volume of a rectangular
prism.

Set Up: Write down the appropriate formula to find the missing information. Repeat as
necessary.

Rectangular Prism Surface Area Formula Rectangular Prism Volume Formula

Surface Area = 2wl + 2hl + 2hw Volume = lwh

1. A rectangular prism has a height of 25, a width of 8, and a length which is twice as long as
its height. What is the volume of this rectangular prism?

A) 2,500
B) 3,200
C) 5,000
D) 10,000

Spheres

Identify: Any question that references the volume of a sphere.

Set Up: Write down the sphere volume formula to find the missing information. Repeat
as necessary.

Sphere Volume Formula


4
Volume = r3
3

1. A sphere has a volume of 7,776. What is the diameter of the sphere?

214 214
PSAT Rectangular Pyramids

Rectangular Pyramids

Identify: Any question that references the volume of a rectangular pyramid or the
distance from the apex of a rectangular pyramid to a point along the base.

Set Up: Write down the appropriate formula to find the missing information for a
volume question. The distance from the apex of a rectangular pyramid to a point along the
base is the hypotenuse of a right triangle.

Rectangular Pyramid Volume Formula


1
Volume = lwh
3

1. A rectangular pyramid has a height of 10, a length of 20, and a volume of 1,800. What is the
width of this rectangular pyramid?

215 215
PSAT Geometry in Three Dimensions Notes

216 216
PSAT Geometry in Three Dimensions Review

1. A sphere has a diameter is 42. What is the volume of the sphere?

2. The height h of a certain cylinder is equal to its radius, If the diameter of the cylinder is 30, then what is the
volume of this cylinder?

3. A local town is building both a new science museum and a new library with walls each made out of stone.
Each building is using stones that are 1 foot by 1 foot by 6 inches. The dimensions of the walls of the science
museum are 80 feet by 60 feet by 3 feet, and the dimensions of the walls of the library are 60 feet by 60 feet by 2
feet. If each building has four walls, by what factor will the number of stones for the walls of the library be
greater than the number of stones for the walls of the science museum?

4. What is the volume of the largest sphere that can be inscribed inside a cube of volume 1,728 ?

A) 144
B) 288
C) 576
D) 2,304

5. A cone with height h sits inside a cube of the same height. If the sides of the cylinder touch each of the four
sides of the box, what is the volume of the cone in terms of r ?

1
A) 2 h2

B) 2h2

h3
C) 4

h3
D) 12

217 217
PSAT Geometry in Three Dimensions Answers

Workbook Answers Review Answers

Cones
1) 12,348

1) B 2) 3,375

Cylinders 3) 0.5

4) B
1) 12
5) D
Rectangular Prisms

1) D

Spheres

1) 36

Rectangular Pyramids

1) 27

218 218
PSAT Graphing

PSAT Math:
Graphing

Important Formulas

Circle Graphing Formula


Circle Graphing Formula: (x h)2 + (y k)2 = r2

Center: (h, k)
Radius: r

Parabola Graphing Formulas

Graphing Formula: y = ax2 + bx + c


Vertex form: y = a(x h)2 + k, where (h, k) is the vertex

If: a > 0; parabola opens up (u-shaped). If: a < 0; parabola opens down (n-shaped)
b
x-coordinate of the vertex (highest or lowest point of the curve): 2a
A parabolas axis of symmetry passes vertically through the vertex.

219 219
PSAT Graphing: Circles

Graphing: Circles

Identify: Look for questions that reference the graphing formula of a circle.

Set Up: Plug in the elements from the question into the circle equation.

Circle Graphing Formula


Circle Graphing Formula: (x h)2 + (y k)2 = r2

Center: (h, k)
Radius: r

1. What is the radius of a circle with the equation (x 4)2 + (y + 8)2 36 = 0?

2. The Santa Monica carnival recently purchased a new Ferris Wheel. The wheel has a
diameter of 220 feet.

Which of the following is the equation of the circle formed by a rider traveling along the
circumference of the Ferris Wheel?

A) x2 + y2 = 2202
B) (x + y)2 = 220
C) x2 + y2 = 1102
D) x + y = 1102

220 220
PSAT Graphing: Parabolas

Graphing: Parabolas

Identify: Look for questions that have a diagram of a parabola or reference the graphing
formula(s) of a parabola.

Set Up: Plug in the elements from the question into the relevant parabola equation(s).

Parabola Graphing Formulas


Graphing Formula: y = ax2 + bx + c
Vertex form: y = a(x h)2 + k, where (h, k) is the vertex

If: a > 0; parabola opens up (u-shaped). If: a < 0; parabola opens down (n-shaped)
b
x-coordinate of the vertex (highest or lowest point of the curve):
2a
A parabolas axis of symmetry passes vertically through the vertex.

1. The graph of y = (3x 3) (x + 3) is a parabola in the xy-plane. In which of the following


equivalent equations do the x- and y-coordinates of the vertex of the parabola appear as
constants or coefficients?

A) y = 3x2 + 12x 9
B) y = 3(x + 1) + 6
C) y = 3(x + 1)2 + (12)
D) y = (x 3)(3x 3)

2. The xy-plane above shows one of the two points of intersection of the graphs of a linear
function and a quadratic function. The shown point of intersection has coordinates (a, b). If
the vertex of the graph of the quadratic function is at (2, 0), what is the value of a? (Round
to the nearest thousandth.)

A) 0.432
B) 1.076
C) 1.576
D) 2.458

221 221
PSAT Graphing Notes

222 222
PSAT Graphing Review

1. (NO CALCULATOR) What is the diameter of the circle represented by the below equation?

(x + 8)2 + (y 11)2 = 432

y = x2 2x + 24

2. The equation above represents a parabola in the xy-plane. Which of the following equivalent forms of the
equation displays the x-intercepts of the parabola as constants or coefficients?

A) y 6 = x2 6x
B) y + 4 = (x 2)2
C) y = x(x 4) + 8
D) y = (x + 4)(x 6)

3. (NO CALCULATOR) Which of the following is an equation of a circle in the xy-plane with center (0, 2) and a
radius with endpoint (4, 5)?

A) x2 + (y 2)2 = 25
B) x2 + (y 5)2 = 25
C) (x 2)2 + y2 = 5
D) (x 4)2 + y2 = 5

4. The graph of y = (2x + 2) (x 4) is a parabola in the xy-plane. In which of the following equivalent equations
do the x- and y-coordinates of the vertex of the parabola appear as constants or coefficients?
3 50
A) y = 2(x )2 + ( )
2 4
3 50
B) y = 2(x ) + ( )
2 4
3 50
C) y = 2(x 2)3 + ( 2 )

D) y = (x 2)(2x 8)

223 223
PSAT Graphing Answers

Workbook Answers Review Answers

Graphing Circles
1) 86

1) 6 2) D

2) C 3) A

Graphing Parabolas 4) B

1) C

2) B

224 224
PSAT Lines and Angles

PSAT Math:
Lines and Angles

Important Formulas
Rule of 180
Sum of the angles in any triangle = 180 degrees
Sum of the angles on a straight line = 180 degrees

225 225
PSAT Intersecting Lines

Lines and Angles: Intersecting Lines

Identify: A question has two parallel lines intersected by a third line.

Set Up: For parallel lines intersected by a third line, the formed acute angles are
congruent, the formed obtuse angles are congruent, and acute and obtuse angles
are supplementary.

1. In the figure above, x||y and a = 110. What is the value of b ?

2. (NO CALCULATOR) Lines w, x, y, and z are shown below, and yz. Which of the
following is the set of all angles that must be supplementary to angle a ?

A) {9, 10}
B) {2, 3, 9, 10}
C) {9, 10, 13, 14}
D) {2, 3, 5, 7, 9, 10, 13, 14}

226 226
PSAT Rule of 180

Lines and Angles: Rule of 180

Identify: If you see angles on a straight line, and you are being asked for an angle
measurement.

Set Up: Use the Rule of 180 to solve for the unknown information.

1. Given the triangle shown below with exterior angles that measure a, b, and c as shown,
what is the sum of a, b, and c?

2. (NO CALCULATOR) In the figure above, lines a, b, and c intersect at a point. If j + k = x


+ y, which of the following must be true?

I. j = l
II. k = z
III. l = z

A) I and II only
B) I and III only
C) II and III only
D) I, II, and III

227 227
PSAT Lines and Angles Notes

228 228
PSAT Lines and Angles Review

1. In the figure above, XWY measures 35, WXY measures 105, and points X, Y, and Z are collinear.
What is the measure of WYZ ?

2. In the figure above, if r is parallel to s and b = 96, then a + c =

A) 168
B) 180
C) 192
D) Cannot be determined

3. The measure of NOP in the figure above is 150. The measures of the three angles are given in terms of
x, in degrees. What is the measure of QOP ?

4. In the figure above, line m is parallel to line n. Transversals u and v intersect at point B on m and
intersect n at points R and S, respectively. Point Z is on n, the measure of BSZ is 130, and the measure of
RBS is 80. How many of the angles formed by rays of m, n, u, and v have a measure of 50?

229 229
SAT Lines and Angles Answers

Workbook Answers Review Answers

Intersecting Lines
1) 140

1) 70 2) A

2) B 3) 120

Rule of 180 4) 8

1) 360

2) B

230 230
PSAT Quadrilaterals and Polygons

PSAT Math:
Quadrilaterals and
Polygons
Important Formulas
Rhombuses
Area Formula Perimeter Formula Angle Relationship
Area = bh Perimeter = 2b + 2h
(In a square, b and h are equal) (In a square, b and h are equal)

Trapezoids
Area Formula Angle Relationship
b1 + b2
Area = ( 2 )h

Polygons
Sum of Interior Angles = 180 (n 2)
n = number of sides

231 231
PSAT Quadrilaterals and Polygons: Rhombus Questions

Quadrilaterals and Polygons: Rhombus Questions

Identify: A question references the area, angles, and/or perimeter of a non-trapezoid


quadrilateral.

Set Up: Write down the appropriate formula(s) and/or relationship(s) to find the
missing information.

Area Formula Perimeter Formula Angle Relationship


Area = bh Perimeter = 2b + 2h
(In a square, b and h are equal) (In a square, b and h are equal)

1. In the above parallelogram, if x = 140, what is 2y ?

2. Jan is planning to fence a 6-foot by 9-foot rectangular plot of ground to use as a garden. She
intends to plant a 1-foot-wide border of flowers along the inside of the entire perimeter.

What is the minimum number of feet of fence Jan would need to enclose the garden if there will be a 2-
foot-wide opening on one side of the plot for people to enter the garden?

A) 13
B) 15
C) 28
D) 30

3. Tammy puts a fence around her rectangular garden. The garden has a width that is 4 feet
more than 2 times its length. What is the perimeter of Tammys fence if the area of her garden
is 1,550 square feet? (round to the nearest foot)

232 232
PSAT Quadrilaterals and Polygons: Trapezoid Questions

Quadrilaterals and Polygons: Trapezoid Questions

Identify: Any question that references the area and/or angles of a trapezoid.

Set Up: Write down the appropriate formula and/or relationship to find the missing
information.

Area Formula Angle Relationship


b1 + b2
Area = ( 2 )h

1. What is the area of a trapezoid with bases of length 8 and 6 and a height of 20 ?

2. In the above parallelogram, if x = 120, what is y + 100 ?

3. (NO CALCULATOR) In quadrilateral ABCD above, BC is parallel to AD, and AB = CD. If


BC and AD were each halved and BE was decreased by 50 percent, how would the area of
ABCD change?

A) The area of ABCD would be decreased by 75 percent.


B) The area of ABCD would be decreased by 50 percent.
C) The area of ABCD would not change.
D) The area of ABCD would be multiplied by 2.

233 233
PSAT Quadrilaterals and Polygons: Polygons

Quadrilaterals and Polygons: Polygons

Identify: Look for regular shapes with greater than four sides. Regular shapes have
sides that are all equal and interior (inside) angles that are all equal.

Set Up: For area problems, try to break the polygon into smaller figures such as
triangles. For angle problems, use the Sum of Interior Angles formula.

Sum of Interior Angles Formula


Sum of Interior Angles = 180 (n 2)
n = number of sides

1. In the diagram above, ABCDE is a regular polygon. What is the value of x ?

2. The figure above shows a regular hexagon with sides of length z and a square with sides of
length z. If the area of the hexagon is 60 3 square inches, what is the area, in square inches,
of the square?

A) 40
B) 32
C) 16 3
D) 8 3

234 234
PSAT Polygons and Quadrilaterals Notes

235 235
PSAT Polygons and Quadrilaterals Review

Questions 1 and 2 relate to the below information

A local town is currently constructing a new park that is 80 feet by 100 feet. This park will be all grass,
except for a rectangular track that runs along the inside border of the park and is 15 feet wide.

1. What is the area, in square feet, of the parks grassy area?

2. What is the minimum number of feet of fence the city would need to enclose the park (which includes the
track) if there will be two separate openings for access that are each four feet wide?

Questions 3 relates to the below information

The perimeter of a rectangle is P units, the area is A units, and the width is w units. The length of the
rectangle is x units longer than the width.

3. (NO CALCULATOR) Which of the following equations expresses x in terms of P and w ?

p
A) 2 2w

p
B) 4w
2

C) P 2w

D) P 4w

4. A circular fountain with a diameter of 30 meters is to be placed entirely within a rectangular plaza that is
40 meters by 60 meters. Bricks will be laid on the entire plaza around the fountain (but not under it), making
it accessible to pedestrians. What is the approximate area, in square meters, of the plaza that will be brick?

A) 400 square feet


B) 1,400 square feet
C) 1,700 square feet
D) 2,300 square feet

236 236
PSAT Polygons and Quadrilaterals Answers

Workbook Answers Review Answers

Rhombus Questions
1) 3,500

1) 80 2) 352

2) C 3) A

3) 169 4) C

Trapezoid Questions

1) 140

2) 160

3) A

Polygon Questions

1) 126

2) A

237 237
PSAT Triangles

PSAT Math:
Triangles

Important Formulas
Area Formula Pythagorean Theorem
1
Area = bh a2 + b2 = c2
2

Rule of 180
Sum of the angles in any triangle = 180 degrees
Sum of the angles on a straight line = 180 degrees

238 238
PSAT Triangles: Angles (Rule of 180)

Triangles: Angles (Rule of 180)

Identify: If you see a triangle or intersecting lines, and you are being asked for an
angle measurement.

Set Up: Use the Rule of 180 to solve for the unknown angle(s).

Rule of 180
Sum of the angles in any triangle = 180 degrees
Sum of the angles on a straight line = 180 degrees

1. Two angles of a triangle are 23 and 71. What is the other angle in this triangle?

2. In the circle above with center O, if w = 54, what fraction of the circles total circumference
is minor arc AB ?

3. Two isosceles triangles are show above. If 180 b = 3a and a = 30, what is the value of c ?

239 239
PSAT Triangles: Area

Triangles: Area

Identify: If the area of a triangle is referenced, you are being asked a triangle area
question.

Set Up: Immediately write down the triangle area formula. Plug whatever
information is given in the question (triangle area, base, and/or height) into the formula
and solve for whatever is missing in the equation. Repeat as necessary.

Triangle Area Formula


1
Area = 2bh

1. A right isosceles triangle has an area of 162. What is the length of each of its legs?

2. In the figure above, XY = YZ, and XZ is a diameter of the circle, having a length of 10
inches. What is the area of XYZ, in square inches?

3. In the figure above, the circle has a center O and radius r. Segments NL and NM are
tangent to the circle. If P is the midpoint of segment NL and the measure of angle LOP equals
the measure of angle OPL, what is the combined area, in terms of r, of the triangles MNO and
LNO ?

A) r 5
B) 2r 5
C) r2
D) 2r2

240 240
PSAT Triangles: Side Lengths (Pythagorean Theorem)

Triangles: Side Lengths (Pythagorean Theorem)

Identify: If you are solving for the side of a right triangle or need to find the length of
any diagonal line, you are being asked a right triangle side length question.

Set Up: Immediately write down the Pythagorean Theorem. Plug whatever
information is given in the question (hypotenuse and/or legs) into the formula and solve for
whatever is missing in the equation. Repeat as necessary.

Pythagorean Theorem

a2 + b2 = c2

1. Consider a right triangle with a hypotenuse of length 13, and legs (9 x) and 12. What is
the value of x?

2. In pentagon FGHIJ above, what is the value of a2 + b2 ?

3. A right triangle has legs of lengths j and k and a hypotenuse of length l. If j2 + k2 + l2 =


1058, what is the value of 3l ?

A) 24
B) 33
C) 48
D) 69

241 241
PSAT Triangles: Special Right Triangles

Triangles: Special Right Triangles

Identify: If you see a right triangle with the angles 30-60-90 or 45-45-90.

Set Up: Use special right triangle ratios to solve the problem.

30-60-90 45-45-90

1. In the figure above, RSTV is a square and UV = 7. What is the area of RSTV ?

2. In quadrilateral MNOP above, the bisector of <MNO is perpendicular to MO at point Q. If


QO = 6 and NO = 12, what is the measure of angle NMQ ?

A) 30
B) 50
C) 55
D) 60

242 242
PSAT Triangles: Similar Triangles

Triangles: Similar Triangles

Identify: If you see triangles with similar angle measures.

Set Up: If triangles have similar angle measures, then their side lengths are
proportional.
.

If: Then: And:


a=d
A B C
b=e = = ABC ~ DEF
D E F
c =f

1. The sides of a triangle are 11, 14, and 19. What is the length of the longest side of a
similar triangle whose shortest side is 15 ? (Round your answer to the nearest tenth.)

2. The lengths represented by AB, EB, BD, and CD in the above diagram are 600 feet,
800 feet, 450 feet, and 550 feet, respectively. Segments AC and DE intersect at B, and
AEB and CDB have the same measure. What is the value of x ?

243 243
PSAT Triangles Notes

244 244
PSAT Triangles Review

1. To get to the bookstore from his house, Jacob must drive his car 12 miles north and then 5
miles west. What is the shortest distance, in miles, between the bookstore and Jacobs house?

2. If a triangle has a height of 15 and a base of 40, what is the area of the triangle?

3. In the above triangle, if ABO is 34, what is v ?

4. (NO CALCULATOR) In the circle shown above, E is the center and lies on AD and BC.
Which of the following statements is true?

A) EDC measures 76
B) ED is perpendicular to CD
C) AB BE
D) CD AE

5. (NO CALCULATOR) In the figure above STU ~ VUW. Which of the following must be
true?

A) STU = VUW
B) ST | SU
C) SU UV
D) STU = UVW

245 245
PSAT Triangles Answers

Workbook Answers Review Answers

Triangles: Angles (Rule of 180)


1) 13

1) 86 2) 300

1 3) 112
2) 5
4) A
3) 135
5) D
Triangle: Area

1) 18

2) 25

3) D

Triangles: Sides Lengths


(Pythagorean Theorem)

1) 4

2) 21

3) D

Triangles: Special Right Triangles

1) 147

2) D

Triangles: Similar Triangles

1) 25.9

2) 978

246 246
PSAT Advanced Topics (Math)

PSAT Math:
Advanced Topics

Important Formulas

Compound Interest Formula


r
Compound Interest: A = P(1 + n)nt
A = Amount accumulated
P = Principal
r = Interest rate
n = Compounding per period
t = Number of periods

247 247
PSAT Advanced Concepts (Math): Complex Numbers

Advanced Concepts (Math): Complex Numbers (i)

Identify: The question will include the complex number i.

Set Up: Use i = to solve the question.

1. (NO CALCULATOR) What value is equivalent to 14 8i2 ?

2. Which of the following is equal to (5 2i)(7 + 4i) = ? (Note: i = 1)

A) 27
B) 43
C) 27 + 6i
D) 43 + 6i

4 6i
3. Which of the following complex numbers is equivalent to ? (Note: i = 1)
6 3i

14 8i
A) 15 + 15

14 8i
B) 15 15

2 24i
C) 3 + 33

2 24i
D) 3 33

248 248
PSAT Advanced Concepts (Math): Compound Interest

Advanced Concepts (Math): Compound Interest

Identify: The question will ask about compound interest.

Set Up: Use the compound interest formula to solve the question.

Compound Interest Formula


r
Compound Interest: A = P(1 + n)nt
A = Amount accumulated
P = Principal
r = Interest rate
n = Compounding per period
t = Number of periods

1. Eric opened a bank account that earns 4 percent interest compounded annually. His initial
deposit was $100 and he uses the expression $100(x)t to find the value of the account after t
years. What is the value of x in the expression?

2. Erics friend Nathan found an account that earns 6 percent interest compounded annually.
Nathan made an initial deposit of $200 into his account and at the same time Eric made a
deposit of $100 into his account. After ten years, how much more money will Nathans initial
deposit have earned than Erics initial deposit?

A) $114.45
B) $178.76
C) $210.15
D) $305.34

3. If you deposit $2,500 into an account paying 8% annual interest compounded quarterly, how
much money will be in the account after 4 years? (Round your answer to the nearest dollar
and ignore the dollar sign when gridding in your response.)

249 249
PSAT Advanced Concepts (Math): Radians

Advanced Concepts (Math): Radians

Identify: Question with angles determined in terms of .

Set Up: Use radians = 180 degrees to solve the question.

1. What degree measurement is equivalent to 15.5 ?

2. If two interior angles of a triangle have measures of 15 and 45 degrees respectively, then
what is the measure of the third angle, in radians?

2
A) 3

B) 2

C) 3

D) 6


3. If two interior angles of a triangle have measures of 9 and 12 radians respectively, then
what is the measure of the third angle, in degrees?

250 250
PSAT Advanced Concepts: Trigonometry: Sine, Cosine, and Tangent

Trigonometry: Sine, Cosine, and Tangent

Identify: Look for references to sine, cosine, or tangent in the question.

Set Up: Use the SOH-CAH-TOA relationships to solve the question.

SOH-CAH-TOA
opposite adjacent opposite
sin = cos = tan =
hypotenuse hypotenuse adjacent

1. (NO CALCULATOR) For the above right triangle, what is cos B ?

a
2. (NO CALCULATOR) Given the triangle ABC above, which of the following is equal to ?
b

A) cos A
B) cos B
C) tan A
D) tan B

3. In triangle LMN, the measure of M is 90, MN = 8, and LN = 10. Triangle QRS is similar
to triangle QRS, where vertices Q, R, and S correspond to vertices L, M, and N, respectively,
and each side of triangle QRS is four times the length of the corresponding side of triangle
LMN. What is the value of tan Q ?

251 251
PSAT Advanced Concepts (Math): Unit Circle

Advanced Concepts (Math): Unit Circle

Identify: Look for trigonometric identities (sine,


cosine, or tangent) and radian measures in the
question.

Set Up: Use unit circle values or draw a unit


circle and graph the information provided in the
question to solve the question.

Cosine and sine are co-functions of each other. First number: Cosine
This mean that complementary angles of cosine and Second number: Sine
sine are equal. Tangent = sine/cosine

b
1
1. (NO CALCULATOR) In the above right triangle sin a = 3. What is cos (90 a) ?

2. (NO CALCULATOR) In the xy-plane above, O is the center of the circle, and the measure

of AOB is radians. What is the value of b ?
b

252 252
PSAT Advanced Concepts (Math): Notes

253 253
PSAT Advanced Concepts (Math): Review

1. (NO CALCULATOR) What is the value of cos() ?

2. At a time when the suns rays are striking the level ground at an angle of 35, a statue casts a
shadow that is 81 feet long. To the nearest foot, how many feet tall is the statue? (Note: sin 35
0.57, cos 35 0.82, tan 35 0.70)

3. For i = 1 , (3 i)2 = ?

A) 3i
B) 10i
C) 8 6i
D) 10 6i


4. (NO CALCULATOR) What is the value of (sin(4))((cos(3)) ?

A) 1

B) 2

2
C) 2

2
D) 4

5. (NO CALCULATOR) In a triangle, one angle measures 90, and the other angles measure x
2
and y. If sin x = 3 , What is cos y ?

254 254
PSAT Advanced Concepts (Math): Answers

Workbook Answers Unit Circle

Complex Numbers
1
1) 3
1) 22
2) 3
2) D

3) B
Review Answers

1) 1
Compound Interest
2) 57

1) 1.04 3) C

2) C 4) D

3) 3,432
2
5) 3

Radians

1) 2,790

2) A

3) 145

Trigonometry: SOH-CAH-TOA

3
1) 5

2) C

4
3) 3

255 255
PSAT Interpreting Equations

PSAT Math:
Interpreting Equations

256 256
PSAT Interpreting Equations: Linear Equations

Interpreting Equations: Linear Equations

Identify: A question asks about an equation model that can be rewritten in the form
y = mx + b .

Set Up: Identify the units represented by x and y (the answer must relate to these
units) and then plug numbers in for x to identify trends in the data.

W = 450 + 20m

1. (NO CALCULATOR) The equation above is used to model the relationship between
weight, W, of a grizzly bear and the bears age, m, in months. According to the model, what is
the meaning of the 450 in the equation?

A) For every increase in one month of age, a grizzly bear weighs an additional 450 pounds.
B) For every increase in twenty months of age, a grizzly bear weighs an additional 450
pounds.
C) At birth, a grizzly bear weighs 450 pounds.
D) A grizzly bear gains 20 pounds of weight each month until it weighs 450 pounds.

s = 1,000 40x

2. (NO CALCULATOR) A radio station is running a promotion where a number of songs are
given away each day. The equation above can be used to model the number of songs, s, that
remain to be given away x days after the promotion began. What does it mean that (25, 0) is a
solution to this equation?

A) During the promotion, 40 songs are given away each day.


B) It takes 40 days during the promotion to give away 1,000 songs.
C) There are 40 songs available at the start of the promotion.
D) The song promotion will last for 1,000 days.
.

257 257
PSAT Interpreting Equations: Non-Linear Equations

Interpreting Equations: Non-Linear Equations

Identify: An equation model that CANNOT be rewritten in the form y = mx + b .

Set Up: Translate the information in question into a formula on your own and
then compare your equation to the one given. If there are two things performing a
task at rates that are different magnitudes (one thing is twice as fast as the other, for
example), then the faster thing is represented by a variable and the slower thing is
represented by a multiple of that variable.

1 1 1
6+8=p

1. (NO CALCULATOR) A father and son are painting a shed together. The father can
complete the paint job in six hours, and the son can complete the paint job in eight hours. The
1
equation above describes the situation described. Which the following describes what 8
represents in this equation?

A) The portion of the shed completed by the father.


B) The portion of the shed completed by the father and son.
C) The portion of the shed completed by the son in one hour.
D) The portion of the shed completed by the father and son in one hour.

1 2 1
x+x=5

2. (NO CALCULATOR) John and Jim are working together to move a pile of mulch onto
their mothers flower bed. John can shovel half as fast as Jim, and together they can complete
the job in 5 hours. The equation above represents the situation described. Which of the
1
following describes what the expression x represents in the equation?

A) The time, in hours, it takes Jim to complete the job alone.


B) The portion of the job that John would complete in one hour.
C) The portion of the job that Jim would complete in one hour.
D) The portion of the job that John would complete in two hours.

258 258
PSAT Interpreting Equations Notes

259 259
PSAT Interpreting Equations Review

1. (NO CALCULATOR) Sams Shoe Emporium has volunteered to make a contribution to charity. The
contribution will include a flat donation and a matching amount for each additional business that donates.
The overall contribution of Sams Shoe Emporium is modeled by the equation C = 1,000m + 50,000, where m
represents the number of additional companies that donate.

Which of the following statements is true?

A) For every additional business that contributes, Sams Shoe Emporium will contribute an additional
50,000 dollars.
B) Each additional business will contribute 1,000 dollars.
C) For every additional business that contributes, Sams Shoe Emporium will contribute an additional
1,000 dollars.
D) Sams Shoe Emporium will donate at least 1,000 dollars.

2. (NO CALCULATOR) A construction company estimates the price of a job, in dollars, using the
expression 400 + 25xy, where x is the number of builders who will be working and y is the total number of
days the job will take using x builders. Which of the following is the best interpretation of the number 25 in
the expression?

A) A minimum of 25 builders will work on each job.


B) The price of every job increases by $25 every day.
C) Each builder works 12 hours a day.
D) The company charges $25 per day for each builder.

3. (NO CALCULATOR) Robert paints houses during the summer. If Roberts payment for each house
painted is modeled by the equation H = 500 8p , where p represents time spent painting, in hours, which of
the following statements is true?

A) For every increase in painting of eight hours, Robert receives 500 more dollars in payment.
B) For every increase in painting of eight hours, Robert receives 8 less dollars in payment.
C) For every increase in painting of one hour, Robert receives 500 more dollars in payment.
D) For every increase in painting of one hour, Robert receives 8 less dollars in payment.

1 1 1
+ =
4 7 w

4. (NO CALCULATOR) Rose and Edith are working together to create a quilt. Rose could knit the quilt in
seven hours and Edith could knit the quilt in four hours. The equation above describes the situation
1
described. Which the following describes what 7 represents in this equation?

A) The portion of the quilt completed by Edith.


B) The portion of the quilt completed by Rose.
C) The portion of the quilt completed by Edith in one hour.
D) The portion of the quilt completed by Rose in one hour.

260 260
PSAT Interpreting Equations Answers

Workbook Answers Review Answers

Interpreting Linear Equations


1) C

1) C 2) D

2) A 3) D

Interpreting Non-Linear Equations 4) D

1) C

2) B

261 261
PSAT Plug and Chug & Backsolving

PSAT Math:
Plug and Chug &
Backsolving

262 262
PSAT Plug and Chug & Backsolving: Plug and Chug

Plug and Chug & Backsolving: Plug and Chug

Identify: Problems with variable(s) in the question and variable(s) in the answer choices.
The phrase in terms of often indicates a plug and chug question.

Set Up: Pick number(s) for the variable(s) and solve the question as though the variable(s)
is/are your number(s). Plug your number(s) into the answer choices and look for the same
value. If more than one answer choice works, pick different number(s) to eliminate the
remaining choice(s).

1. (NO CALCULATOR) If 2a 4 = 2b, then 4b 11 =

A) a+6
B) 3a
C) 4a 11
D) 4a 19

2. (NO CALCULATOR) Katrina went to the food store to buy her cereal for the week. All
brands of cereal were discounted 42% off the marked price. Which of the following is an
expression for the discounted price on a marked price of y dollars?

A) 0.42y
B) y 42y
C) y 0.42
D) y 0.42y

3. (NO CALCULATOR) If 6x = 3n 3 and y = 4 n, which of the following expresses y in


terms of x?

4x
A) y = 3
18 2x
B) y = 3
C) y = 4 3x

D) y = 3 2x

263 263
PSAT Plug and Chug & Backsolving: Backsolving

Plug and Chug & Backsolving: Backsolving

Identify: Problems with variable(s) in the question and numbers in the answer choices.

Set Up: Plug the answer choices into the question as the variable(s). Look for the answer
that generates the desired value.

1. If x + 24 = 40, what are the possible values for x?

A) 64 and 16
B) 64 and 16
C) 16 and 26
D) 18 and 64

2. A town committee recently voted for whether or not to expand a local park. It was revealed
that 60 percent of the committee voted for the expansion. Which of the following could be the
total number of committee members?

A) 82
B) 94
C) 110
D) 162

3. (NO CALCULATOR) Which ordered pair (x, y) satisfies the system of equations shown
below?

2x + 4y = 10
4x 2y = 20

A) (5, 0)
B) (5, 0)
C) (0, 2.5)
D) (2.5, 0)

264 264
PSAT Plug and Chug & Backsolving Notes

265 265
PSAT Plug and Chug & Backsolving Review

11 x
1. (NO CALCULATOR) If = , what does x equal?
x2 121

A) 1
B) 11
C) 11
D) 121

2. Which ordered pair (x, y) satisfies the system of equations shown below?

3x + 2y = 12
5x 4y = 42

A) (4, 0)
B) (0, 4)
C) (6, 3)
D) (3, 6)

3. The students at Hawthorne Elementary School are either right-handed or left-


handed. A survey reveals that exactly 15 percent of the students at Hawthorne are
left-handed. Which of the following could be the total number of left-handed students
at Hawthorne Elementary School?

A) 70
B) 84
C) 92
D) 120

4. (NO CALCULATOR) If x 0, y is a real number, y3 = 4x, and y2 = 8x, then y = ?

1
A) 2

B) 1

C) 2

D) 4

266 266
PSAT Plug and Chug & Backsolving Answers

Workbook Answers Review Answers

Plug and Chug


1) C

1) D 2) C

2) D 3) D

3) D 4) A

Backsolving

1) A

2) C

3) B

267 267
PSAT Statistics

PSAT Math:
Statistics

Important Formulas
Exponential Growth
x
y = a(2) b

y = Final Amount
x = Duration
a = Original Amount
b = Doubling Time

268 268
PSAT Statistics: Exponential Growth

Statistics: Exponential Growth

Identify: A question which references exponential growth (periodic doubling).

Set Up: Write down the exponential growth formula and plug in the given
information to solve for the unknown quantity.

Exponential Growth
x
y = a(2) b

y = Final Amount
x = Duration
a = Original Amount
b = Doubling Time

1. A scientist set up a one year experiment to study the mitosis of cells. The cells in the
experiment doubled in number every 6 months. If at the end of the experiment there were 80
cells, how many cells were there at the beginning of the experiment?

2. From 1770 to 1850, the population of a country doubled in size every twenty years. If the
population of the country was 320,000 in 1850, what was the population of the country in
1770?

3. (NO CALCULATOR) From 2000 to 2010, a city worker measured the amount of pollution
in a local river every 24 months. The pollution of pollution in January 2000 was 150 ppm, and
had doubled at every subsequent measuring. If Z represents the pollution at the end of 2010,
then which of the following equations models pollution over time (in ppm)?

A) Z = 150(2)120
24

B) Z = 150(2)120
5

C) Z = 150(2)24(5)
D) Z = 150(2)105

269 269
PSAT Statistics: Interpreting Charts and Graphs

Statistics: Interpreting Charts and Graphs

Identify: The question will describe a survey, poll, experiment, or other situation,
using a chart, table, and/or graph.

Set Up: Review the titles of any tables, and the headings of any axes. Make sure that
you are using data from the correct location(s).

Questions 1 and 2 relate to the below data

The table below shows how 2,461 residents of three different cities voted in a recent election.

Candidate A Candidate B Candidate C Total


City A 234 232 123 589
City B 156 453 454 1,063
City C 343 123 343 809
Total 733 808 920 2,461

1. What percentage of all votes cast in City A went to Candidates A and C? (Round to the
nearest whole number.)

2. What percentage of all votes cast in the three cities did Candidate C receive? (Round to the
nearest tenth of a percent.)

100

90
Guests

80

70

60
4PM 5PM 6PM 7PM 8PM 9PM 10PM 11PM
Time
3. The line graph above shows the hourly numbers of guests from 4PM to 11PM at a
local restaurant. According to the graph, what was the greatest change (in absolute
value) in hourly guests between two consecutive hours?

A) 7 guests
B) 10 guests
C) 16 guests
D) 21 guests

270 270
PSAT Statistics: Line of Best Fit

Statistics: Line of Best Fit

Identify: A question will reference the line of best fit for a given collection of data
(usually arranged in a scatter plot).

Set Up: The line of best fit is a line on a graph shows the general direction that a
group of points seems to be heading. The slope (rise divided by run) of a line of best fit
describes the trend of the collected data.

Questions 1, 2, and 3 relate to the below data

The below scatterplot shows the relationship between the ages and weights of 15 dogs
in a local neighborhood.
60
Weight (kilograms)

50
40
30
20
10
0
2 4 6 8 10 12
Age

1. (NO CALCULATOR) Based on the line of best fit, which of the below answers is
closest to the predicted weight for a dog that is 11 years old?

A) 27 kilograms
B) 35 kilograms
C) 45 kilograms
D) 50 kilograms

2. (NO CALCULATOR) Based on the line of best fit, which of the below answers is
the closest to the predicted age for a dog that weighs 25 kilograms?

A) 6 years old
B) 7 years old
C) 8 years old
D) 9 years old

3. (NO CALCULATOR) What is the approximate slope of the line of best fit for the
above data?

A) 4
B) 1
C) 2
D) 4

271 271
PSAT Statistics: Margin of Error

Statistics: Margin of Error

Identify: A question will reference some form of data collection and the terms margin of
error, sample/sample error or association.

Set Up: To decrease the margin of error (the amount of random sampling error) or increase
the association (the strength of the connection between different data points) in the results
of a survey, additional information must align with the population in the original
survey.

1. (NO CALCULATOR) Jason surveyed 100 Massachusetts lawyers about their opinions on
tort reform. Which of the below actions will most likely result in a lower margin of error for
the survey?

A) Asking 200 lawyers from around the world about their opinions on tort reform.
B) Asking 500 people from across all professions about their opinions on tort reform.
C) Asking 200 Massachusetts lawyers about their opinions on tort reform.
D) Asking 500 lawyers from across the United Stated about their opinions on tort reform.

2. (NO CALCULATOR) A researcher wanted to know if there is an association between the


amount of rain and the number of daffodils in California. She studied rainfall and daffodil
data for 10 years and found convincing evidence of a positive association between rainfall and
daffodils in California. Which of the following conclusions is well supported by the data?

A) There is a positive association between the amount of rainfall and the number of
daffodils in the United States.
B) There is a positive association between the amount of rainfall and the number of
daffodils in California.
C) Using the number of daffodils and the amount of rain as defined in the study, an
increase in the number of daffodils is caused by an increase in the amount of rain in
the United States.
D) Using the number of daffodils and the amount of rain as defined in the study, an
increase in the number of daffodils is caused by an increase in the amount of rain
around the world.

272 272
PSAT Statistics Notes

273 273
PSAT Statistics Review

Questions 1 and 2 relate the following scenario

A student at a local university created a program to double a given number at a regular interval of two hours. The
student then input 12 as the initial number and set the experiment to run for 3 days.

1. (NO CALCULATOR) If N represents the number at the end of the experiment, then which of the following
equations models the experiment? (1 day = 24 hours.)

A) N = 12(12)36
B) N = 12(2)36
C) N = 12(2)12
D) N = 12(12)12

2. If the student ran the experiment again for 2 days and ended with the number 117,440,512, what was the
original number for this experiment (1 day = 24 hours)?

3. (NO CALCULATOR) Ms. Douglas asked fifteen students in her tenth grade French class what activity they
would like to do for a field trip. Which of the below actions will most likely result in a more accurate sample of the
preferred field trip of her students?

A) Asking 25 students in all of her classes what activity they would like to do for a field trip.
B) Asking 25 of her tenth grade French students what activity they would like to do for a field trip.
C) Asking 35 of her fellow teachers what activity they would like to do for a field trip.
D) Asking 35 of her ninth, tenth, and eleventh grade French students what activity they would like to do
for a field trip.

4. (NO CALCULATOR) A recent study of the commute times for 2,500 people around the world showed that as
people live closer to a downtown area, their commute time increases. Which of the following conclusions is
supported by the data?

A) There is a positive association between proximity to a downtown area and commute time in the United
States.
B) There is a positive association between proximity to a downtown area and commute time around the world.
C) There is a negative association between proximity to a downtown area and commute time in the United
States.
D) There is a negative association between proximity to a downtown area and commute time around the world.

274 274
PSAT Statistics Review

Questions 5 and 6 relate to the below data

The below scatterplot shows the growth and achievement percentiles of Ms. Frazees second grade class on a
recent standardized assessment.

100

80

60

40

20

0
0 20 40 60 80 100
5. (NO CALCULATOR) Based on the above data, which of the following would be the most accurate
statement?

A) The data shows a positive association that is linear.


B) The data shows a positive association that is not linear.
C) The data shows a negative association that is linear.
D) The data shows a negative association that is not linear.

6. Which linear equation most accurately represents the line of best fit for this data (not shown)?

A) y = 1.5x + 135
B) y = 1.5x + 160
C) y = 1.5x + 180
D) y = 1.5x + 135

275 275
PSAT Statistics Review
Questions 7, 8, 9, and 10 relate to the below data

The table below shows how 2,461 residents of three different cities voted in a recent national election.

1st Graders 2nd Graders 3rd Graders Total


Blue 7 14 31 52
Red 12 24 17 53
Green 23 21 8 52
Total 42 59 56 157

7. What percentage of 1st and 2nd graders chose blue or red as their favorite color? (Round to the nearest
tenth.)

8. What percentage of 1st and 3rd graders chose green as their favorite color? (Round to the neatest tenth.)

9. How many times as likely is it for blue to be a 1st graders favorite color than for blue to be a 2nd graders
favorite color?

A) 0.70 times as likely


B) 0.87 times as likely
C) 1.30 times as likely
D) 1.42 times as likely

10. How many times as likely is it for red to be a 2nd graders favorite color than for red to be a 3rd graders
favorite color?

A) 0.71 times as likely


B) 0.89 times as likely
C) 1.02 times as likely
D) 1.34 times as likely

276 276
PSAT Statistics Answers

Workbook Answers Review Answers

Statistics: Exponential Growth


1) B
1) 20
2) 7
2) 20,000 3) B

3) A 4) D

5) C
Statistics: Interpreting Charts and
Graphs 6) A

7) 56.4
1) 61
8) 31.6
2) 37.4 9) A

3) C 10) D

Statistics: Line of Best Fit

1) C

2) A

3) D

Statistics: Margin of Error

1) C

2) B

277 277

You might also like